219
យុវសិស ឈ ទុយ IMO Shortlist 1959-2009 Problem

យុវសិស្ ឈឹ ទីុយ IMO Shortlist 1959-2009 · IMO Shortlist 1959 1 Prove that the fraction 21n+4 14n+3 ... Let α be the acute angel subtending, from A, that

  • Upload
    others

  • View
    18

  • Download
    0

Embed Size (px)

Citation preview

Page 1: យុវសិស្ ឈឹ ទីុយ IMO Shortlist 1959-2009 · IMO Shortlist 1959 1 Prove that the fraction 21n+4 14n+3 ... Let α be the acute angel subtending, from A, that

យវសស ឈ ទយ

IMO Shortlist 1959-2009

Problem

Page 2: យុវសិស្ ឈឹ ទីុយ IMO Shortlist 1959-2009 · IMO Shortlist 1959 1 Prove that the fraction 21n+4 14n+3 ... Let α be the acute angel subtending, from A, that

IMO Shortlist 1959

1 Prove that the fraction21n + 414n + 3

is irreducible for every natural number n.

2 For what real values of x is √x +

√2x− 1 +

√x−

√2x− 1 = A

given

a) A =√

2;

b) A = 1;

c) A = 2,

where only non-negative real numbers are admitted for square roots?

3 Let a, b, c be real numbers. Consider the quadratic equation in cos x

a cos x2 + b cos x + c = 0.

Using the numbers a, b, c form a quadratic equation in cos 2x whose roots are the same asthose of the original equation. Compare the equation in cos x and cos 2x for a = 4, b = 2,c = −1.

4 Construct a right triangle with given hypotenuse c such that the median drawn to the hy-potenuse is the geometric mean of the two legs of the triangle.

5 An arbitrary point M is selected in the interior of the segment AB. The square AMCDand MBEF are constructed on the same side of AB, with segments AM and MB as theirrespective bases. The circles circumscribed about these squares, with centers P and Q,intersect at M and also at another point N . Let N ′ denote the point of intersection of thestraight lines AF and BC.

a) Prove that N and N ′ coincide;

b) Prove that the straight lines MN pass through a fixed point S independent of the choiceof M ;

c) Find the locus of the midpoints of the segments PQ as M varies between A and B.

6 Two planes, P and Q, intersect along the line p. The point A is given in the plane P , andthe point C in the plane Q; neither of these points lies on the straight line p. Constructan isosceles trapezoid ABCD (with AB ‖ CD) in which a circle can be inscribed, and withvertices B and D lying in planes P and Q respectively.

http://www.artofproblemsolving.com/This file was downloaded from the AoPS Math Olympiad Resources Page Page 1

Page 3: យុវសិស្ ឈឹ ទីុយ IMO Shortlist 1959-2009 · IMO Shortlist 1959 1 Prove that the fraction 21n+4 14n+3 ... Let α be the acute angel subtending, from A, that

IMO Shortlist 1959

1 Prove that the fraction21n + 414n + 3

is irreducible for every natural number n.

2 For what real values of x is √x +

√2x− 1 +

√x−

√2x− 1 = A

given

a) A =√

2;

b) A = 1;

c) A = 2,

where only non-negative real numbers are admitted for square roots?

3 Let a, b, c be real numbers. Consider the quadratic equation in cos x

a cos x2 + b cos x + c = 0.

Using the numbers a, b, c form a quadratic equation in cos 2x whose roots are the same asthose of the original equation. Compare the equation in cos x and cos 2x for a = 4, b = 2,c = −1.

4 Construct a right triangle with given hypotenuse c such that the median drawn to the hy-potenuse is the geometric mean of the two legs of the triangle.

5 An arbitrary point M is selected in the interior of the segment AB. The square AMCDand MBEF are constructed on the same side of AB, with segments AM and MB as theirrespective bases. The circles circumscribed about these squares, with centers P and Q,intersect at M and also at another point N . Let N ′ denote the point of intersection of thestraight lines AF and BC.

a) Prove that N and N ′ coincide;

b) Prove that the straight lines MN pass through a fixed point S independent of the choiceof M ;

c) Find the locus of the midpoints of the segments PQ as M varies between A and B.

6 Two planes, P and Q, intersect along the line p. The point A is given in the plane P , andthe point C in the plane Q; neither of these points lies on the straight line p. Constructan isosceles trapezoid ABCD (with AB ‖ CD) in which a circle can be inscribed, and withvertices B and D lying in planes P and Q respectively.

http://www.artofproblemsolving.com/This file was downloaded from the AoPS Math Olympiad Resources Page Page 1

Page 4: យុវសិស្ ឈឹ ទីុយ IMO Shortlist 1959-2009 · IMO Shortlist 1959 1 Prove that the fraction 21n+4 14n+3 ... Let α be the acute angel subtending, from A, that

IMO Shortlist 1960

1 Determine all three-digit numbers N having the property that N is divisible by 11, andN

11is equal to the sum of the squares of the digits of N .

2 For what values of the variable x does the following inequality hold:

4x2

(1−√

2x + 1)2< 2x + 9 ?

3 In a given right triangle ABC, the hypotenuse BC, of length a, is divided into n equal parts (nand odd integer). Let α be the acute angel subtending, from A, that segment which containsthe mdipoint of the hypotenuse. Let h be the length of the altitude to the hypotenuse fo thetriangle. Prove that:

tanα =4nh

(n2 − 1)a.

4 Construct triangle ABC, given ha, hb (the altitudes from A and B), and ma, the medianfrom vertex A.

5 Consider the cube ABCDA′B′C ′D′ (with face ABCD directly above face A′B′C ′D′).

a) Find the locus of the midpoints of the segments XY , where X is any point of AC and Yis any piont of B′D′;

b) Find the locus of points Z which lie on the segment XY of part a) with ZY = 2XZ.

6 Consider a cone of revolution with an inscribed sphere tangent to the base of the cone. Acylinder is circumscribed about this sphere so that one of its bases lies in the base of the cone.let V1 be the volume of the cone and V2 be the volume of the cylinder.

a) Prove that V1 6= V2;

b) Find the smallest number k for which V1 = kV2; for this case, construct the angle subtendedby a diamter of the base of the cone at the vertex of the cone.

7 An isosceles trapezoid with bases a and c and altitude h is given.

a) On the axis of symmetry of this trapezoid, find all points P such that both legs of thetrapezoid subtend right angles at P ;

b) Calculate the distance of p from either base;

c) Determine under what conditions such points P actually exist. Discuss various cases thatmight arise.

http://www.artofproblemsolving.com/This file was downloaded from the AoPS Math Olympiad Resources Page Page 1

Page 5: យុវសិស្ ឈឹ ទីុយ IMO Shortlist 1959-2009 · IMO Shortlist 1959 1 Prove that the fraction 21n+4 14n+3 ... Let α be the acute angel subtending, from A, that

IMO Shortlist 1961

1 Solve the system of equations:x + y + z = a

x2 + y2 + z2 = b2

xy = z2

where a and b are constants. Give the conditions that a and b must satisfy so that x, y, z aredistinct positive numbers.

2 Let a, b, c be the sides of a triangle, and S its area. Prove:

a2 + b2 + c2 ≥ 4S√

3

In what case does equality hold?

3 Solve the equation cosn x− sinn x = 1 where n is a natural number.

4 Consider triangle P1P2P3 and a point p within the triangle. Lines P1P, P2P, P3P intersectthe opposite sides in points Q1, Q2, Q3 respectively. Prove that, of the numbers

P1P

PQ1,P2P

PQ2,P3P

PQ3

at least one is ≤ 2 and at least one is ≥ 2

5 Construct a triangle ABC if AC = b, AB = c and ∠AMB = w, where M is the midpoint ofthe segment BC and w < 90. Prove that a solution exists if and only if

b tanw

2≤ c < b

In what case does the equality hold?

6 Consider a plane ε and three non-collinear points A,B, C on the same side of ε; suppose theplane determined by these three points is not parallel to ε. In plane ε take three arbitrarypoints A′, B′, C ′. Let L,M,N be the midpoints of segments AA′, BB′, CC ′; Let G be thecentroid of the triangle LMN . (We will not consider positions of the points A′, B′, C ′ suchthat the points L,M,N do not form a triangle.) What is the locus of point G as A′, B′, C ′

range independently over the plane ε?

http://www.artofproblemsolving.com/This file was downloaded from the AoPS Math Olympiad Resources Page Page 1

Page 6: យុវសិស្ ឈឹ ទីុយ IMO Shortlist 1959-2009 · IMO Shortlist 1959 1 Prove that the fraction 21n+4 14n+3 ... Let α be the acute angel subtending, from A, that

IMO Shortlist 1962

1 Find the smallest natural number n which has the following properties:

a) Its decimal representation has a 6 as the last digit.

b) If the last digit 6 is erased and placed in front of the remaining digits, the resulting numberis four times as large as the original number n.

2 Determine all real numbers x which satisfy the inequality:

√3− x−

√x + 1 >

12

3 Consider the cube ABCDA′B′C ′D′ (ABCD and A′B′C ′D′ are the upper and lower bases,repsectively, and edges AA′, BB′, CC ′, DD′ are parallel). The point X moves at a constantspeed along the perimeter of the square ABCD in the direction ABCDA, and the point Ymoves at the same rate along the perimiter of the square B′C ′CB in the direction B′C ′CBB′.Points X and Y begin their motion at the same instant from the starting positions A and B′,respectively. Determine and draw the locus of the midpionts of the segments XY .

4 Solve the equation cos2 x + cos2 2x + cos2 3x = 1

5 On the circle K there are given three distinct points A,B, C. Construct (using only a straight-edge and a compass) a fourth point D on K such that a circle can be inscribed in the quadri-lateral thus obtained.

6 Consider an isosceles triangle. let R be the radius of its circumscribed circle and r be theradius of its inscribed circle. Prove that the distance d between the centers of these two circleis

d =√

R(R− 2r)

7 The tetrahedron SABC has the following property: there exist five spheres, each tangent tothe edges SA, SB, SC,BC,CA, AB, or to their extensions.

a) Prove that the tetrahedron SABC is regular.

b) Prove conversely that for every regular tetrahedron five such spheres exist.

http://www.artofproblemsolving.com/This file was downloaded from the AoPS Math Olympiad Resources Page Page 1

Page 7: យុវសិស្ ឈឹ ទីុយ IMO Shortlist 1959-2009 · IMO Shortlist 1959 1 Prove that the fraction 21n+4 14n+3 ... Let α be the acute angel subtending, from A, that

IMO Shortlist 1963

1 Find all real roots of the equation√x2 − p + 2

√x2 − 1 = x

where p is a real parameter.

2 Point A and segment BC are given. Determine the locus of points in space which are verticesof right angles with one side passing through A, and the other side intersecting segment BC.

3 In an n-gon A1A2 . . . An, all of whose interior angles are equal, the lengths of consecutivesides satisfy the relation

a1 ≥ a2 ≥ · · · ≥ an.

Prove that a1 = a2 = . . . = an.

4 Find all solutions x1, x2, x3, x4, x5 of the system

x5 + x2 = yx1

x1 + x3 = yx2

x2 + x4 = yx3

x3 + x5 = yx4

x4 + x1 = yx5

where y is a parameter.

5 Prove that cos π7 − cos 2π

7 + cos 3π7 = 1

2

6 Five students A,B, C, D, E took part in a contest. One prediction was that the contestantswould finish in the order ABCDE. This prediction was very poor. In fact, no contestantfinished in the position predicted, and no two contestants predicted to finish consecutivelyactually did so. A second prediction had the contestants finishing in the order DAECB. Thisprediction was better. Exactly two of the contestants finished in the places predicted, andtwo disjoint pairs of students predicted to finish consecutively actually did so. Determine theorder in which the contestants finished.

http://www.artofproblemsolving.com/This file was downloaded from the AoPS Math Olympiad Resources Page Page 1

Page 8: យុវសិស្ ឈឹ ទីុយ IMO Shortlist 1959-2009 · IMO Shortlist 1959 1 Prove that the fraction 21n+4 14n+3 ... Let α be the acute angel subtending, from A, that

IMO Shortlist 1964

2 Suppose a, b, c are the sides of a triangle. Prove that

a2(b + c− a) + b2(a + c− b) + c2(a + b− c) ≤ 3abc

3 A circle is inscribed in a triangle ABC with sides a, b, c. Tangents to the circle parallel to thesides of the triangle are contructe. Each of these tangents cuts off a triagnle from 4ABC.In each of these triangles, a circle is inscribed. Find the sum of the areas of all four inscribedcircles (in terms of a, b, c).

4 Seventeen people correspond by mail with one another-each one with all the rest. In theirletters only three different topics are discussed. each pair of correspondents deals with onlyone of these topics. Prove that there are at least three people who write to each other aboutthe same topic.

5 Supppose five points in a plane are situated so that no two of the straight lines joining themare parallel, perpendicular, or coincident. From each point perpendiculars are drawn to allthe lines joining the other four points. Determine the maxium number of intersections thatthese perpendiculars can have.

6 In tetrahedron ABCD, vertex D is connected with D0, the centrod if 4ABC. Line parallelto DD0 are drawn through A,B and C. These lines intersect the planes BCD,CAD andABD in points A2, B1, and C1, respectively. Prove that the volume of ABCD is one thirdthe volume of A1B1C1D0. Is the result if point Do is selected anywhere within 4ABC?

http://www.artofproblemsolving.com/This file was downloaded from the AoPS Math Olympiad Resources Page Page 1

Page 9: យុវសិស្ ឈឹ ទីុយ IMO Shortlist 1959-2009 · IMO Shortlist 1959 1 Prove that the fraction 21n+4 14n+3 ... Let α be the acute angel subtending, from A, that

IMO Shortlist 1965

1 Determine all values of x in the interval 0 ≤ x ≤ 2π which satisfy the inequality

2 cos x ≤√

1 + sin 2x−√

1− sin 2x ≤√

2.

2 Consider the sytem of equations

a11x1 + a12x2 + a13x3 = 0

a21x1 + a22x2 + a23x3 = 0

a31x1 + a32x2 + a33x3 = 0

with unknowns x1, x2, x3. The coefficients satisfy the conditions:

a) a11, a22, a33 are positive numbers;

b) the remaining coefficients are negative numbers;

c) in each equation, the sum ofthe coefficients is positive.

Prove that the given system has only the solution x1 = x2 = x3 = 0.

3 Given the tetrahedron ABCD whose edges AB and CD have lengths a and b respectively.The distance between the skew lines AB and CD is d, and the angle between them is ω.Tetrahedron ABCD is divided into two solids by plane ε, parallel to lines AB and CD. Theratio of the distances of ε from AB and CD is equal to k. Compute the ratio of the volumesof the two solids obtained.

4 Find all sets of four real numbers x1, x2, x3, x4 such that the sum of any one and the productof the other three is equal to 2.

5 Consider 4OAB with acute angle AOB. Thorugh a point M 6= O perpendiculars are drawnto OA and OB, the feet of which are P and Q respectively. The point of intersection of thealtitudes of 4OPQ is H. What is the locus of H if M is permitted to range over

a) the side AB;

b) the interior of 4OAB.

6 In a plane a set of n points (n ≥ 3) is give. Each pair of points is connected by a segment.Let d be the length of the longest of these segments. We define a diameter of the set to beany connecting segment of length d. Prove that the number of diameters of the given set isat most n.

http://www.artofproblemsolving.com/This file was downloaded from the AoPS Math Olympiad Resources Page Page 1

Page 10: យុវសិស្ ឈឹ ទីុយ IMO Shortlist 1959-2009 · IMO Shortlist 1959 1 Prove that the fraction 21n+4 14n+3 ... Let α be the acute angel subtending, from A, that

IMO Shortlist 1966

1 Given n > 3 points in the plane such that no three of the points are collinear. Does thereexist a circle passing through (at least) 3 of the given points and not containing any other ofthe n points in its interior ?

2 Given n positive real numbers a1, a2, . . . , an such that a1a2 · · · an = 1, prove that

(1 + a1)(1 + a2) · · · (1 + an) ≥ 2n.

3 A regular triangular prism has the altitude h, and the two bases of the prism are equilateraltriangles with side length a. Dream-holes are made in the centers of both bases, and thethree lateral faces are mirrors. Assume that a ray of light, entering the prism through thedream-hole in the upper base, then being reflected once by any of the three mirrors, quits theprism through the dream-hole in the lower base. Find the angle between the upper base andthe light ray at the moment when the light ray entered the prism, and the length of the wayof the light ray in the interior of the prism.

4 Given 5 points in the plane, no three of them being collinear. Show that among these 5points, we can always find 4 points forming a convex quadrilateral.

5 Prove the inequality

tanπ sinx

4 sinα+ tan

π cos x

4 cos α> 1

for any x, α with 0 ≤ x ≤ π2 and π

6 < y < π3 .

6 Let m be a convex polygon in a plane, l its perimeter and S its area. Let M (R) be the locusof all points in the space whose distance to m is ≤ R, and V (R) is the volume of the solidM (R) .

a.) Prove that

V (R) =43πR3 +

π

2lR2 + 2SR.

Hereby, we say that the distance of a point C to a figure m is ≤ R if there exists a point Dof the figure m such that the distance CD is ≤ R. (This point D may lie on the boundary ofthe figure m and inside the figure.)

additional question:

b.) Find the area of the planar R-neighborhood of a convex or non-convex polygon m.

c.) Find the volume of the R-neighborhood of a convex polyhedron, e. g. of a cube or of atetrahedron.

Note by Darij: I guess that the ”R-neighborhood” of a figure is defined as the locus of allpoints whose distance to the figure is ≤ R.

http://www.artofproblemsolving.com/This file was downloaded from the AoPS Math Olympiad Resources Page Page 1

Page 11: យុវសិស្ ឈឹ ទីុយ IMO Shortlist 1959-2009 · IMO Shortlist 1959 1 Prove that the fraction 21n+4 14n+3 ... Let α be the acute angel subtending, from A, that

IMO Shortlist 1966

7 For which arrangements of two infinite circular cylinders does their intersection lie in a plane?

8 We are given a bag of sugar, a two-pan balance, and a weight of 1 gram. How do we obtain1 kilogram of sugar in the smallest possible number of weighings?

9 Find x such that trigonometric

sin 3x cos(60 − x) + 1sin(60 − 7x)− cos(30 + x) + m

= 0

where m is a fixed real number.

10 How many real solutions are there to the equation x = 1964 sin x− 189 ?

11 Does there exist an integer z that can be written in two different ways as z = x! + y!, wherex, y are natural numbers with x ≤ y ?

12 Find digits x, y, z such that the equality√xx · · ·x︸ ︷︷ ︸n times

− yy · · · y︸ ︷︷ ︸n times

= zz · · · z︸ ︷︷ ︸n times

holds for at least two values of n ∈ N, and in that case find all n for which this equality istrue.

13 Let a1, a2, . . . , an be positive real numbers. Prove the inequality

(n

2

) ∑i<j

1aiaj

≥ 4

∑i<j

1ai + aj

2

14 What is the maximal number of regions a circle can be divided in by segments joining npoints on the boundary of the circle ?

Posted already on the board I think...

15 Given four points A, B, C, D on a circle such that AB is a diameter and CD is not a diameter.Show that the line joining the point of intersection of the tangents to the circle at the pointsC and D with the point of intersection of the lines AC and BD is perpendicular to the lineAB.

16 We are given a circle K with center S and radius 1 and a square Q with center M and side 2.Let XY be the hypotenuse of an isosceles right triangle XY Z. Describe the locus of pointsZ as X varies along K and Y varies along the boundary of Q.

http://www.artofproblemsolving.com/This file was downloaded from the AoPS Math Olympiad Resources Page Page 2

Page 12: យុវសិស្ ឈឹ ទីុយ IMO Shortlist 1959-2009 · IMO Shortlist 1959 1 Prove that the fraction 21n+4 14n+3 ... Let α be the acute angel subtending, from A, that

IMO Shortlist 1966

17 Let ABCD and A′B′C ′D′ be two arbitrary parallelograms in the space, and let M, N, P, Qbe points dividing the segments AA′, BB′, CC ′, DD′ in equal ratios.

a.) Prove that the quadrilateral MNPQ is a parallelogram.

b.) What is the locus of the center of the parallelogram MNPQ, when the point M moveson the segment AA′ ?

(Consecutive vertices of the parallelograms are labelled in alphabetical order.

18 Solve the equation 1sin x + 1

cos x = 1p where p is a real parameter. Discuss for which values of p

the equation has at least one real solution and determine the number of solutions in [0, 2π)for a given p.

19 Construct a triangle given the radii of the excircles.

20 Given three congruent rectangles in the space. Their centers coincide, but the planes they liein are mutually perpendicular. For any two of the three rectangles, the line of intersection ofthe planes of these two rectangles contains one midparallel of one rectangle and one midparallelof the other rectangle, and these two midparallels have different lengths. Consider the convexpolyhedron whose vertices are the vertices of the rectangles.

a.) What is the volume of this polyhedron ?

b.) Can this polyhedron turn out to be a regular polyhedron ? If yes, what is the conditionfor this polyhedron to be regular ?

21 Prove that the volume V and the lateral area S of a right circular cone satisfy the inequality(6V

π

)2

≤(

2S

π√

3

)3

When does equality occur?

22 Let P and P ′ be two parallelograms with equal area, and let their sidelengths be a, b and a′,b′. Assume that a′ ≤ a ≤ b ≤ b′, and moreover, it is possible to place the segment b′ such thatit completely lies in the interior of the parallelogram P.

Show that the parallelogram P can be partitioned into four polygons such that these fourpolygons can be composed again to form the parallelogram P ′.

23 Three faces of a tetrahedron are right triangles, while the fourth is not an obtuse triangle.

(a) Prove that a necessary and sufficient condition for the fourth face to be a right triangleis that at some vertex exactly two angles are right.

(b) Prove that if all the faces are right triangles, then the volume of the tetrahedron equalsone -sixth the product of the three smallest edges not belonging to the same face.

http://www.artofproblemsolving.com/This file was downloaded from the AoPS Math Olympiad Resources Page Page 3

Page 13: យុវសិស្ ឈឹ ទីុយ IMO Shortlist 1959-2009 · IMO Shortlist 1959 1 Prove that the fraction 21n+4 14n+3 ... Let α be the acute angel subtending, from A, that

IMO Shortlist 1966

24 There are n ≥ 2 people at a meeting. Show that there exist two people at the meeting whohave the same number of friends among the persons at the meeting. (It is assumed that if Ais a friend of B, then B is a friend of A; moreover, nobody is his own friend.)

25 Prove thattan 730′ =

√6 +

√2−

√3− 2.

26 Prove the inequality

a.) (a1 + a2 + ... + ak)2 ≤ k

(a2

1 + a22 + ... + a2

k

),

where k ≥ 1 is a natural number and a1, a2, ..., ak are arbitrary real numbers.

b.) Using the inequality (1), show that if the real numbers a1, a2, ..., an satisfy the inequality

a1 + a2 + ... + an ≥√

(n− 1)(a2

1 + a22 + ... + a2

n

),

then all of these numbers a1, a2, . . . , an are non-negative.

27 Given a point P lying on a line g, and given a circle K. Construct a circle passing throughthe point P and touching the circle K and the line g.

28 In the plane, consider a circle with center S and radius 1. Let ABC be an arbitrary trianglehaving this circle as its incircle, and assume that SA ≤ SB ≤ SC. Find the locus of

a.) all vertices A of such triangles;

b.) all vertices B of such triangles;

c.) all vertices C of such triangles.

29 A given natural number N is being decomposed in a sum of some consecutive integers.

a.) Find all such decompositions for N = 500.

b.) How many such decompositions does the number N = 2α3β5γ (where α, β and γ arenatural numbers) have? Which of these decompositions contain natural summands only?

c.) Determine the number of such decompositions (= decompositions in a sum of consecutiveintegers; these integers are not necessarily natural) for an arbitrary natural N.

Note by Darij: The 0 is not considered as a natural number.

30 Let n be a positive integer, prove that :

(a) log10(n + 1) > 310n + log10 n;

(b) log n! > 3n10

(12 + 1

3 + · · ·+ 1n − 1

).

http://www.artofproblemsolving.com/This file was downloaded from the AoPS Math Olympiad Resources Page Page 4

Page 14: យុវសិស្ ឈឹ ទីុយ IMO Shortlist 1959-2009 · IMO Shortlist 1959 1 Prove that the fraction 21n+4 14n+3 ... Let α be the acute angel subtending, from A, that

IMO Shortlist 1966

31 Solve the equation |x2 − 1| + |x2 − 4| = mx as a function of the parameter m. Which pairs(x,m) of integers satisfy this equation ?

32 The side lengths a, b, c of a triangle ABC form an arithmetical progression (such that b−a =c− b). The side lengths a1, b1, c1 of a triangle A1B1C1 also form an arithmetical progression(with b1 − a1 = c1 − b1). [Hereby, a = BC, b = CA, c = AB, a1 = B1C1, b1 = C1A1,c1 = A1B1.] Moreover, we know that ]CAB = ]C1A1B1.

Show that triangles ABC and A1B1C1 are similar.

Note by Darij: I have changed the wording of the problem since the conditions given in theoriginal were not sufficient, at least not in the form they were written.

33 Given two internally tangent circles; in the bigger one we inscribe an equilateral triangle.From each of the vertices of this triangle, we draw a tangent to the smaller circle. Prove thatthe length of one of these tangents equals the sum of the lengths of the two other tangents.

34 Find all pairs of positive integers (x; y) satisfying the equation 2x = 3y + 5.

35 Let ax3 + bx2 + cx + d be a polynomial with integer coefficients a, b, c, d such that ad is anodd number and bc is an even number. Prove that (at least) one root of the polynomial isirrational.

36 Let ABCD be a quadrilateral inscribed in a circle. Show that the centroids of triangles ABC,CDA, BCD, DAB lie on one circle.

37 Show that the four perpendiculars dropped from the midpoints of the sides of a cyclic quadri-lateral to the respective opposite sides are concurrent.

Note by Darij: A cyclic quadrilateral is a quadrilateral inscribed in a circle.

38 Two concentric circles have radii R and r respectively. Determine the greatest possible numberof circles that are tangent to both these circles and mutually nonintersecting. Prove that thisnumber lies between 3

2 ·√

R+√

r√R−

√r− 1 and 63

20 ·R+rR−r .

39 Consider a circle with center O and radius R, and let A and B be two points in the plane ofthis circle.

a.) Draw a chord CD of the circle such that CD is parallel to AB, and the point of theintersection P of the lines AC and BD lies on the circle.

b.) Show that generally, one gets two possible points P (P1 and P2) satisfying the conditionof the above problem, and compute the distance between these two points, if the lengthsOA = a, OB = b and AB = d are given.

http://www.artofproblemsolving.com/This file was downloaded from the AoPS Math Olympiad Resources Page Page 5

Page 15: យុវសិស្ ឈឹ ទីុយ IMO Shortlist 1959-2009 · IMO Shortlist 1959 1 Prove that the fraction 21n+4 14n+3 ... Let α be the acute angel subtending, from A, that

IMO Shortlist 1966

40 For a positive real number p, find all real solutions to the equation√x2 + 2px− p2 −

√x2 − 2px− p2 = 1.

41 Given a regular n-gon A1A2...An (with n ≥ 3) in a plane. How many triangles of the kindAiAjAk are obtuse ?

42 Given a finite sequence of integers a1, a2, ..., an for n ≥ 2. Show that there exists a subsequenceak1 , ak2 , ..., akm , where 1 ≤ k1 ≤ k2 ≤ ... ≤ km ≤ n, such that the number a2

k1+a2

k2+ ...+a2

km

is divisible by n.

Note by Darij: Of course, the 1 ≤ k1 ≤ k2 ≤ ... ≤ km ≤ n should be understood as1 ≤ k1 < k2 < ... < km ≤ n; else, we could take m = n and k1 = k2 = ... = km, so that thenumber a2

k1+ a2

k2+ ... + a2

km= n2a2

k1will surely be divisible by n.

43 Given 5 points in a plane, no three of them being collinear. Each two of these 5 points arejoined with a segment, and every of these segments is painted either red or blue; assume thatthere is no triangle whose sides are segments of equal color.

a.) Show that:

(1) Among the four segments originating at any of the 5 points, two are red and two are blue.

(2) The red segments form a closed way passing through all 5 given points. (Similarly for theblue segments.)

b.) Give a plan how to paint the segments either red or blue in order to have the condition(no triangle with equally colored sides) satisfied.

44 What is the greatest number of balls of radius 1/2 that can be placed within a rectangularbox of size 10× 10× 1 ?

45 An alphabet consists of n letters. What is the maximal length of a word if we know that anytwo consecutive letters a, b of the word are different and that the word cannot be reduced toa word of the kind abab with a 6= b by removing letters.

46 Let a, b, c be reals and

f(a, b, c) =∣∣∣∣ |b− a||ab|

+b + a

ab− 2

c

∣∣∣∣ +|b− a||ab|

+b + a

ab+

2c

Prove that f(a, b, c) = 4 max 1a , 1

b ,1c.

47 Consider all segments dividing the area of a triangle ABC in two equal parts. Find the lengthof the shortest segment among them, if the side lengths a, b, c of triangle ABC are given.How many of these shortest segments exist ?

http://www.artofproblemsolving.com/This file was downloaded from the AoPS Math Olympiad Resources Page Page 6

Page 16: យុវសិស្ ឈឹ ទីុយ IMO Shortlist 1959-2009 · IMO Shortlist 1959 1 Prove that the fraction 21n+4 14n+3 ... Let α be the acute angel subtending, from A, that

IMO Shortlist 1966

48 For which real numbers p does the equation x2 + px + 3p = 0 have integer solutions ?

49 Two mirror walls are placed to form an angle of measure α. There is a candle inside theangle. How many reflections of the candle can an observer see?

50 Solve the equation 1sin x + 1

cos x = 1p where p is a real parameter. Discuss for which values of p

the equation has at least one real solution and determine the number of solutions in [0, 2π)for a given p.

51 Consider n students with numbers 1, 2, . . . , n standing in the order 1, 2, . . . , n. Upon a com-mand, any of the students either remains on his place or switches his place with anotherstudent. (Actually, if student A switches his place with student B, then B cannot switch hisplace with any other student C any more until the next command comes.)

Is it possible to arrange the students in the order n, 1, 2, . . . , n− 1 after two commands ?

52 A figure with area 1 is cut out of paper. We divide this figure into 10 parts and color themin 10 different colors. Now, we turn around the piece of paper, divide the same figure on theother side of the paper in 10 parts again (in some different way). Show that we can color thesenew parts in the same 10 colors again (hereby, different parts should have different colors)such that the sum of the areas of all parts of the figure colored with the same color on bothsides is ≥ 1

10 .

53 Prove that in every convex hexagon of area S one can draw a diagonal that cuts off a triangleof area not exceeding 1

6S.

54 We take 100 consecutive natural numbers a1, a2, ..., a100. Determine the last two digits ofthe number a8

1 + a82 + ... + a8

100.

55 Given the vertex A and the centroid M of a triangle ABC, find the locus of vertices B suchthat all the angles of the triangle lie in the interval [40, 70].

56 In a tetrahedron, all three pairs of opposite (skew) edges are mutually perpendicular. Provethat the midpoints of the six edges of the tetrahedron lie on one sphere.

57 Is it possible to choose a set of 100 (or 200) points on the boundary of a cube such that thisset is fixed under each isometry of the cube into itself? Justify your answer.

58 In a mathematical contest, three problems, A,B, C were posed. Among the participants therwere 25 students who solved at least one problem each. Of all the contestants who did notsolve problem A, the number who solved B was twice the number who solved C. The numberof students who solved only problem A was one more than the number of students who solvedA and at least one other problem. Of all students who solved just one problem, half did notsolve problem A. How many students solved only problem B?

http://www.artofproblemsolving.com/This file was downloaded from the AoPS Math Olympiad Resources Page Page 7

Page 17: យុវសិស្ ឈឹ ទីុយ IMO Shortlist 1959-2009 · IMO Shortlist 1959 1 Prove that the fraction 21n+4 14n+3 ... Let α be the acute angel subtending, from A, that

IMO Shortlist 1966

59 Let a, b, c be the lengths of the sides of a triangle, and α, β, γ respectively, the angles oppositethese sides. Prove that if

a + b = tanγ

2(a tanα + b tanβ)

the triangle is isosceles.

60 Prove that the sum of the distances of the vertices of a regular tetrahedron from the centerof its circumscribed sphere is less than the sum of the distances of these vertices from anyother point in space.

61 Prove that for every natural number n, and for every real number x 6= kπ2t (t = 0, 1, . . . , n; k

any integer)1

sin 2x+

1sin 4x

+ · · ·+ 1sin 2nx

= cot x− cot 2nx

62 Solve the system of equations

|a1 − a2|x2 + |a1 − a3|x3 + |a1 − a4|x4 = 1

|a2 − a1|x1 + |a2 − a3|x3 + |a2 − a4|x4 = 1

|a3 − a1|x1 + |a3 − a2|x2 + |a3 − a4|x4 = 1

|a4 − a1|x1 + |a4 − a2|x2 + |a4 − a3|x3 = 1

where a1, a2, a3, a4 are four different real numbers.

63 Let ABC be a triangle, and let P , Q, R be three points in the interiors of the sides BC, CA,AB of this triangle. Prove that the area of at least one of the three triangles AQR, BRP ,CPQ is less than or equal to one quarter of the area of triangle ABC.

Alternative formulation: Let ABC be a triangle, and let P , Q, R be three points on thesegments BC, CA, AB, respectively. Prove that

min |AQR| , |BRP | , |CPQ| ≤ 14 · |ABC|,

where the abbreviation |P1P2P3| denotes the (non-directed) area of an arbitrary triangleP1P2P3.

http://www.artofproblemsolving.com/This file was downloaded from the AoPS Math Olympiad Resources Page Page 8

Page 18: យុវសិស្ ឈឹ ទីុយ IMO Shortlist 1959-2009 · IMO Shortlist 1959 1 Prove that the fraction 21n+4 14n+3 ... Let α be the acute angel subtending, from A, that

IMO Shortlist 1967

Bulgaria

1 Prove that all numbers of the sequence

1078113

,110778111

3,111077781111

3, . . .

are exact cubes.

2 Prove that

13n2 +

12n +

16≥ (n!)

2n ,

and let n ≥ 1 be an integer. Prove that this inequality is only possible in the case n = 1.

3 Prove the trigonometric inequality cos x < 1− x2

2 + x4

16 , when x ∈(0, π

2

).

4 Suppose medians ma and mb of a triangle are orthogonal. Prove that:

a.) Using medians of that triangle it is possible to construct a rectangular triangle.

b.) The following inequality:5(a2 + b2 − c2) ≥ 8ab,

is valid, where a, b and c are side length of the given triangle.

5 Solve the system of equations:

x2 + x− 1 = yy2 + y − 1 = zz2 + z − 1 = x.

6 Solve the system of equations:

|x + y|+ |1− x| = 6|x + y + 1|+ |1− y| = 4.

http://www.artofproblemsolving.com/This file was downloaded from the AoPS Math Olympiad Resources Page Page 1

Page 19: យុវសិស្ ឈឹ ទីុយ IMO Shortlist 1959-2009 · IMO Shortlist 1959 1 Prove that the fraction 21n+4 14n+3 ... Let α be the acute angel subtending, from A, that

IMO Shortlist 1967

Democratic Republic Of Germany

1 Find whether among all quadrilaterals, whose interiors lie inside a semi-circle of radius r,there exist one (or more) with maximum area. If so, determine their shape and area.

2 Which fractionsp

q, where p, q are positive integers < 100, is closest to

√2? Find all digits

after the point in decimal representation of that fraction which coincide with digits in decimalrepresentation of

√2 (without using any table).

3 Suppose tanα =p

q, where p and q are integers and q 6= 0. Prove that the number tanβ for

which tan 2β = tan 3α is rational only when p2 + q2 is the square of an integer.

4 Prove the following statement: If r1 and r2 are real numbers whose quotient is irrational,then any real number x can be approximated arbitrarily well by the numbers of the formzk1,k2 = k1r1 + k2r2 integers, i.e. for every number x and every positive real number p twointegers k1 and k2 can be found so that |x− (k1r1 + k2r2)| < p holds.

http://www.artofproblemsolving.com/This file was downloaded from the AoPS Math Olympiad Resources Page Page 2

Page 20: យុវសិស្ ឈឹ ទីុយ IMO Shortlist 1959-2009 · IMO Shortlist 1959 1 Prove that the fraction 21n+4 14n+3 ... Let α be the acute angel subtending, from A, that

IMO Shortlist 1967

Great Britain

1 Let k, m, n be natural numbers such that m + k + 1 is a prime greater than n + 1. Letcs = s(s + 1). Prove that

(cm+1 − ck)(cm+2 − ck) . . . (cm+n − ck)

is divisible by the product c1c2 . . . cn.

2 If x is a positive rational number show that x can be uniquely expressed in the form x =∑nk=1

akk! where a1, a2, . . . are integers, 0 ≤ an ≤ n − 1, for n > 1, and the series terminates.

Show that x can be expressed as the sum of reciprocals of different integers, each of which isgreater than 106.

3 The n points P1, P2, . . . , Pn are placed inside or on the boundary of a disk of radius 1 in sucha way that the minimum distance Dn between any two of these points has its largest possiblevalue Dn. Calculate Dn for n = 2 to 7. and justify your answer.

http://www.artofproblemsolving.com/This file was downloaded from the AoPS Math Olympiad Resources Page Page 3

Page 21: យុវសិស្ ឈឹ ទីុយ IMO Shortlist 1959-2009 · IMO Shortlist 1959 1 Prove that the fraction 21n+4 14n+3 ... Let α be the acute angel subtending, from A, that

IMO Shortlist 1967

Hungary

1 In a sports meeting a total of m medals were awarded over n days. On the first day onemedal and 1

7 of the remaining medals were awarded. On the second day two medals and 17

of the remaining medals were awarded, and so on. On the last day, the remaining n medalswere awarded. How many medals did the meeting last, and what was the total number ofmedals ?

2 In the space n ≥ 3 points are given. Every pair of points determines some distance. Supposeall distances are different. Connect every point with the nearest point. Prove that it isimpossible to obtain (closed) polygonal line in such a way.

3 Without using tables, find the exact value of the product:

P =7∏

k=1

cos(

15

).

4 Let k1 and k2 be two circles with centers O1 and O2 and equal radius r such that O1O2 = r.Let A and B be two points lying on the circle k1 and being symmetric to each other withrespect to the line O1O2. Let P be an arbitrary point on k2. Prove that

PA2 + PB2 ≥ 2r2.

5 Prove that for an arbitrary pair of vectors f and g in the space the inequality

af2 + bfg + cg2 ≥ 0

holds if and only if the following conditions are fulfilled:

a ≥ 0, c ≥ 0, 4ac ≥ b2.

6 Three disks of diameter d are touching a sphere in their centers. Besides, every disk touchesthe other two disks. How to choose the radius R of the sphere in order that axis of the wholefigure has an angle of 60 with the line connecting the center of the sphere with the point ofthe disks which is at the largest distance from the axis ? (The axis of the figure is the linehaving the property that rotation of the figure of 120 around that line brings the figure inthe initial position. Disks are all on one side of the plane, passing through the center of thesphere and orthogonal to the axis).

http://www.artofproblemsolving.com/This file was downloaded from the AoPS Math Olympiad Resources Page Page 4

Page 22: យុវសិស្ ឈឹ ទីុយ IMO Shortlist 1959-2009 · IMO Shortlist 1959 1 Prove that the fraction 21n+4 14n+3 ... Let α be the acute angel subtending, from A, that

IMO Shortlist 1967

Italy

1 A0B0C0 and A1B1C1 are acute-angled triangles. Describe, and prove, how to construct thetriangle ABC with the largest possible area which is circumscribed about A0B0C0 (so BCcontains B0, CA contains B0, and AB contains C0) and similar to A1B1C1.

2 Let ABCD be a regular tetrahedron. To an arbitrary point M on one edge, say CD, corre-sponds the point P = P (M) which is the intersection of two lines AH and BK, drawn fromA orthogonally to BM and from B orthogonally to AM . What is the locus of P when Mvaries ?

3 Which regular polygon can be obtained (and how) by cutting a cube with a plane ?

4 Find values of the parameter u for which the expression

y =tan(x− u) + tan(x) + tan(x + u)

tan(x− u) tan(x) tan(x + u)

does not depend on x.

http://www.artofproblemsolving.com/This file was downloaded from the AoPS Math Olympiad Resources Page Page 5

Page 23: យុវសិស្ ឈឹ ទីុយ IMO Shortlist 1959-2009 · IMO Shortlist 1959 1 Prove that the fraction 21n+4 14n+3 ... Let α be the acute angel subtending, from A, that

IMO Shortlist 1967

Mongolia

1 Given m + n numbers: ai, i = 1, 2, . . . ,m, bj , j = 1, 2, . . . , n, determine the number of pairs(ai, bj) for which |i− j| ≥ k, where k is a non-negative integer.

2 An urn contains balls of k different colors; there are ni balls of i− th color. Balls are selectedat random from the urn, one by one, without replacement, until among the selected balls mballs of the same color appear. Find the greatest number of selections.

3 Determine the volume of the body obtained by cutting the ball of radius R by the trihedronwith vertex in the center of that ball, it its dihedral angles are α, β, γ.

4 In what case does the system of equations

x + y + mz = ax + my + z = bmx + y + z = c

have a solution? Find conditions under which the unique solution of the above system is anarithmetic progression.

5 Faces of a convex polyhedron are six squares and 8 equilateral triangles and each edge is acommon side for one triangle and one square. All dihedral angles obtained from the triangleand square with a common edge, are equal. Prove that it is possible to circumscribe a spherearound the polyhedron, and compute the ratio of the squares of volumes of that polyhedronand of the ball whose boundary is the circumscribed sphere.

6 Prove the identityn∑

k=0

(nk

) (tan x

2

)2k(

1 + 2k

(1−tan2 x2 )

k

)= sec2n x

2 + secn x

for any natural number n and any angle x.

http://www.artofproblemsolving.com/This file was downloaded from the AoPS Math Olympiad Resources Page Page 6

Page 24: យុវសិស្ ឈឹ ទីុយ IMO Shortlist 1959-2009 · IMO Shortlist 1959 1 Prove that the fraction 21n+4 14n+3 ... Let α be the acute angel subtending, from A, that

IMO Shortlist 1967

Poland

1 Prove that a tetrahedron with just one edge length greater than 1 has volume at most 18 .

2 Prove this proposition: Center the sphere circumscribed around a tetrahedron which coincideswith the center of a sphere inscribed in that tetrahedron if and only if the skew edges of thetetrahedron are equal.

3 Prove that for arbitrary positive numbers the following inequality holds

1a

+1b

+1c≤ a8 + b8 + c8

a3b3c3.

4 Does there exist an integer such that its cube is equal to 3n2 + 3n + 7, where n is an integer.

5 Show that a triangle whose angles A, B, C satisfy the equality

sin2 A + sin2 B + sin2 C

cos2 A + cos2 B + cos2 C= 2

is a rectangular triangle.

6 A line l is drawn through the intersection point H of altitudes of acute-angle triangles. Provethat symmetric images la, lb, lc of l with respect to the sides BC, CA, AB have one point incommon, which lies on the circumcircle of ABC.

http://www.artofproblemsolving.com/This file was downloaded from the AoPS Math Olympiad Resources Page Page 7

Page 25: យុវសិស្ ឈឹ ទីុយ IMO Shortlist 1959-2009 · IMO Shortlist 1959 1 Prove that the fraction 21n+4 14n+3 ... Let α be the acute angel subtending, from A, that

IMO Shortlist 1967

Romania

1 Decompose the expression into real factors:

E = 1− sin5(x)− cos5(x).

2 The equationx5 + 5λx4 − x3 + (λα− 4)x2 − (8λ + 3)x + λα− 2 = 0

is given. Determine α so that the given equation has exactly (i) one root or (ii) two roots,respectively, independent from λ.

3 Suppose that p and q are two different positive integers and x is a real number. Form theproduct (x + p)(x + q). Find the sum S(x, n) =

∑(x + p)(x + q), where p and q take values

from 1 to n. Does there exist integer values of x for which S(x, n) = 0.

4 (i) Solve the equation:

sin3(x) + sin3

(2π

3+ x

)+ sin3

(4π

3+ x

)+

34

cos 2x = 0.

(ii) Supposing the solutions are in the form of arcs AB with one end at the point A, thebeginning of the arcs of the trigonometric circle, and P a regular polygon inscribed in thecircle with one vertex in A, find:

1) The subsets of arcs having the other end in B in one of the vertices of the regular dodecagon.2) Prove that no solution can have the end B in one of the vertices of polygon P whose numberof sides is prime or having factors other than 2 or 3.

5 If x, y, z are real numbers satisfying relations

x + y + z = 1 and arctan x + arctan y + arctan z =π

4,

prove that x2n+1 + y2n+1 + z2n+1 = 1 holds for all positive integers n.

6 Prove the following inequality:

k∏i=1

xi ·k∑

i=1

xn−1i ≤

k∑i=1

xn+k−1i ,

where xi > 0, k ∈ N, n ∈ N.

http://www.artofproblemsolving.com/This file was downloaded from the AoPS Math Olympiad Resources Page Page 8

Page 26: យុវសិស្ ឈឹ ទីុយ IMO Shortlist 1959-2009 · IMO Shortlist 1959 1 Prove that the fraction 21n+4 14n+3 ... Let α be the acute angel subtending, from A, that

IMO Shortlist 1967

Socialists Republic Of Czechoslovakia

1 The parallelogram ABCD has AB = a,AD = 1, ∠BAD = A, and the triangle ABD has allangles acute. Prove that circles radius 1 and center A,B, C, D cover the parallelogram if andonly

a ≤ cos A +√

3 sinA.

2 Find all real solutions of the system of equations:

n∑k=1

xik = ai

for i = 1, 2, . . . , n.

3 Circle k and its diameter AB are given. Find the locus of the centers of circles inscribed inthe triangles having one vertex on AB and two other vertices on k.

4 The square ABCD has to be decomposed into n triangles (which are not overlapping) andwhich have all angles acute. Find the smallest integer n for which there exist a solution ofthat problem and for such n construct at least one decomposition. Answer whether it ispossible to ask moreover that (at least) one of these triangles has the perimeter less than anarbitrarily given positive number.

5 Let n be a positive integer. Find the maximal number of non-congruent triangles whose sideslengths are integers ≤ n.

6 Given a segment AB of the length 1, define the set M of points in the following way: itcontains two points A,B, and also all points obtained from A,B by iterating the followingrule: With every pair of points X, Y the set M contains also the point Z of the segment XYfor which Y Z = 3XZ.

http://www.artofproblemsolving.com/This file was downloaded from the AoPS Math Olympiad Resources Page Page 9

Page 27: យុវសិស្ ឈឹ ទីុយ IMO Shortlist 1959-2009 · IMO Shortlist 1959 1 Prove that the fraction 21n+4 14n+3 ... Let α be the acute angel subtending, from A, that

IMO Shortlist 1967

Soviet Union

1 Let a1, . . . , a8 be reals, not all equal to zero. Let

cn =8∑

k=1

ank

for n = 1, 2, 3, . . .. Given that among the numbers of the sequence (cn), there are infinitelymany equal to zero, determine all the values of n for which cn = 0.

2 Is it possible to find a set of 100 (or 200) points on the boundary of a cube such that this setremains fixed under all rotations which leave the cube fixed ?

3 Find all x for which, for all n,n∑

k=1

sin kx ≤√

32

.

4 In a group of interpreters each one speaks one of several foreign languages, 24 of them speakJapanese, 24 Malaysian, 24 Farsi. Prove that it is possible to select a subgroup in whichexactly 12 interpreters speak Japanese, exactly 12 speak Malaysian and exactly 12 speakFarsi.

5 A linear binomial l(z) = Az +B with complex coefficients A and B is given. It is known thatthe maximal value of |l(z)| on the segment −1 ≤ x ≤ 1 (y = 0) of the real line in the complexplane z = x + iy is equal to M. Prove that for every z

|l(z)| ≤ Mρ,

where ρ is the sum of distances from the point P = z to the points Q1 : z = 1 and Q3 : z = −1.

6 On the circle with center 0 and radius 1 the point A0 is fixed and points A1, A2, . . . , A999, A1000

are distributed in such a way that the angle ∠A00Ak = k (in radians). Cut the circle at pointsA0, A1, . . . , A1000. How many arcs with different lengths are obtained. ?

http://www.artofproblemsolving.com/This file was downloaded from the AoPS Math Olympiad Resources Page Page 10

Page 28: យុវសិស្ ឈឹ ទីុយ IMO Shortlist 1959-2009 · IMO Shortlist 1959 1 Prove that the fraction 21n+4 14n+3 ... Let α be the acute angel subtending, from A, that

IMO Shortlist 1967

Sweden

1 Determine all positive roots of the equation xx = 1√2.

2 Let n and k be positive integers such that 1 ≤ n ≤ N + 1, 1 ≤ k ≤ N + 1. Show that:

minn6=k

| sinn− sin k| < 2N

.

3 The function ϕ(x, y, z) defined for all triples (x, y, z) of real numbers, is such that there aretwo functions f and g defined for all pairs of real numbers, such that

ϕ(x, y, z) = f(x + y, z) = g(x, y + z)

for all real numbers x, y and z. Show that there is a function h of one real variable, such that

ϕ(x, y, z) = h(x + y + z)

for all real numbers x, y and z.

4 A subset S of the set of integers 0 - 99 is said to have property A if it is impossible to fill acrossword-puzzle with 2 rows and 2 columns with numbers in S (0 is written as 00, 1 as 01,and so on). Determine the maximal number of elements in the set S with the property A.

5 In the plane a point O is and a sequence of points P1, P2, P3, . . . are given. The distancesOP1, OP2, OP3, . . . are r1, r2, r3, . . . Let α satisfies 0 < α < 1. Suppose that for every n thedistance from the point Pn to any other point of the sequence is ≥ rα

n . Determine the exponentβ, as large as possible such that for some C independent of n

rn ≥ Cnβ, n = 1, 2, . . .

6 In making Euclidean constructions in geometry it is permitted to use a ruler and a pair ofcompasses. In the constructions considered in this question no compasses are permitted,but the ruler is assumed to have two parallel edges, which can be used for constructing twoparallel lines through two given points whose distance is at least equal to the breadth of therule. Then the distance between the parallel lines is equal to the breadth of the ruler. Carrythrough the following constructions with such a ruler. Construct:

a) The bisector of a given angle. b) The midpoint of a given rectilinear line segment. c) Thecenter of a circle through three given non-collinear points. d) A line through a given pointparallel to a given line.

http://www.artofproblemsolving.com/This file was downloaded from the AoPS Math Olympiad Resources Page Page 11

Page 29: យុវសិស្ ឈឹ ទីុយ IMO Shortlist 1959-2009 · IMO Shortlist 1959 1 Prove that the fraction 21n+4 14n+3 ... Let α be the acute angel subtending, from A, that

IMO Shortlist 1968

1 Two ships sail on the sea with constant speeds and fixed directions. It is known that at 9 : 00the distance between them was 20 miles; at 9 : 35, 15 miles; and at 9 : 55, 13 miles. At whatmoment were the ships the smallest distance from each other, and what was that distance ?

2 Find all triangles whose side lengths are consecutive integers, and one of whose angles is twiceanother.

3 Prove that every tetrahedron has a vertex whose three edges have the right lengths to forma triangle.

4 Let a, b, c be real numbers with a non-zero. It is known that the real numbers x1, x2, . . . , xn

satisfy the n equations:ax2

1 + bx1 + c = x2

ax22 + bx2 + c = x3

. . . . . . . . . . . .

ax2n + bxn + c = x1

Prove that the system has zero, one or more than one real solutions if (b − 1)2 − 4ac isnegative, equal to zero or positive respectively.

5 Let hn be the apothem (distance from the center to one of the sides) of a regular n-gon(n ≥ 3) inscribed in a circle of radius r. Prove the inequality

(n + 1)hn + 1− nhn > r.

Also prove that if r on the right side is replaced with a greater number, the inequality willnot remain true for all n ≥ 3.

6 If ai (i = 1, 2, . . . , n) are distinct non-zero real numbers, prove that the equation

a1

a1 − x+

a2

a2 − x+ · · ·+ an

an − x= n

has at least n− 1 real roots.

7 Prove that the product of the radii of three circles exscribed to a given triangle does notexceed A = 3

√3

8 times the product of the side lengths of the triangle. When does equalityhold?

8 Given an oriented line ∆ and a fixed point A on it, consider all trapezoids ABCD one ofwhose bases AB lies on ∆, in the positive direction. Let E,F be the midpoints of AB andCD respectively. Find the loci of vertices B,C,D of trapezoids that satisfy the following:

http://www.artofproblemsolving.com/This file was downloaded from the AoPS Math Olympiad Resources Page Page 1

Page 30: យុវសិស្ ឈឹ ទីុយ IMO Shortlist 1959-2009 · IMO Shortlist 1959 1 Prove that the fraction 21n+4 14n+3 ... Let α be the acute angel subtending, from A, that

IMO Shortlist 1968

(i) |AB| ≤ a (a fixed);

(ii) |EF | = l (l fixed);

(iii) the sum of squares of the nonparallel sides of the trapezoid is constant.

[hide=”Remark”]

Remark. The constants are chosen so that such trapezoids exist.

9 Let ABC be an arbitrary triangle and M a point inside it. Let da, db, dc be the distancesfrom M to sides BC, CA, AB; a, b, c the lengths of the sides respectively, and S the area ofthe triangle ABC. Prove the inequality

abdadb + bcdbdc + cadcda ≤4S2

3.

Prove that the left-hand side attains its maximum when M is the centroid of the triangle.

10 Consider two segments of length a, b (a > b) and a segment of length c =√

ab.

(a) For what values of a/b can these segments be sides of a triangle ?

(b) For what values of a/b is this triangle right-angled, obtuse-angled, or acute-angled ?

11 Find all solutions (x1, x2, ..., xn) of the equation

1 +1x1

+x1 + 1x1x2

+(x1 + 1)(x2 + 1)

x12x3+ · · ·+ (x1 + 1)(x2 + 1) · · · (xn−1 + 1)

x1x2 · · ·xn= 0

12 If a and b are arbitrary positive real numbers and m an integer, prove that(1 +

a

b

)m+

(1 +

b

a

)m≥ 2m+1.

13 Given two congruent triangles A1A2A3 and B1B2B3 (AiAk = BiBk), prove that there exists aplane such that the orthogonal projections of these triangles onto it are congruent and equallyoriented.

14 A line in the plane of a triangle ABC intersects the sides AB and AC respectively at pointsX and Y such that BX = CY . Find the locus of the center of the circumcircle of triangleXAY.

15 Let n be a natural number. Prove that⌊n + 20

21

⌋+

⌊n + 21

22

⌋+ · · ·+

⌊n + 2n−1

2n

⌋= n.

[hide=”Remark”]For any real number x, the number bxc represents the largest integer smalleror equal with x.

http://www.artofproblemsolving.com/This file was downloaded from the AoPS Math Olympiad Resources Page Page 2

Page 31: យុវសិស្ ឈឹ ទីុយ IMO Shortlist 1959-2009 · IMO Shortlist 1959 1 Prove that the fraction 21n+4 14n+3 ... Let α be the acute angel subtending, from A, that

IMO Shortlist 1968

16 A polynomial p(x) = a0xk + a1x

k−1 + · · ·+ ak with integer coefficients is said to be divisibleby an integer m if p(x) is divisible by m for all integers x. Prove that if p(x) is divisible bym, then k!a0 is also divisible by m. Also prove that if a0, k,m are non-negative integers forwhich k!a0 is divisible by m, there exists a polynomial p(x) = a0x

k + · · ·+ ak divisible by m.

17 Given a point O and lengths x, y, z, prove that there exists an equilateral triangle ABC forwhich OA = x,OB = y, OC = z, if and only if x + y ≥ z, y + z ≥ x, z + x ≥ y (the pointsO,A,B, C are coplanar).

18 If an acute-angled triangle ABC is given, construct an equilateral triangle A′B′C ′ in spacesuch that lines AA′, BB′, CC ′ pass through a given point.

19 We are given a fixed point on the circle of radius 1, and going from this point along thecircumference in the positive direction on curved distances 0, 1, 2, . . . from it we obtain pointswith abscisas n = 0, 1, 2, . . . . respectively. How many points among them should we take toensure that some two of them are less than the distance 1

5 apart ?

20 Given n (n ≥ 3) points in space such that every three of them form a triangle with one anglegreater than or equal to 120, prove that these points can be denoted by A1, A2, . . . , An insuch a way that for each i, j, k, 1 ≤ i < j < k ≤ n, angle AiAjAk is greater than or equal to120.

21 Let a0, a1, . . . , ak (k ≥ 1) be positive integers. Find all positive integers y such that

a0|y, (a0 + a1)|(y + a1), . . . , (a0 + an)|(y + an).

22 Find all natural numbers n the product of whose decimal digits is n2 − 10n− 22.

23 Find all complex numbers m such that polynomial

x3 + y3 + z3 + mxyz

can be represented as the product of three linear trinomials.

25 Given k parallel lines l1, . . . , lk and ni points on the line li, i = 1, 2, . . . , k, find the maximumpossible number of triangles with vertices at these points.

26 Let f be a real-valued function defined for all real numbers, such that for some a > 0 we have

f(x + a) =12

+√

f(x)− f(x)2

for all x. Prove that f is periodic, and give an example of such a non-constant f for a = 1.

http://www.artofproblemsolving.com/This file was downloaded from the AoPS Math Olympiad Resources Page Page 3

Page 32: យុវសិស្ ឈឹ ទីុយ IMO Shortlist 1959-2009 · IMO Shortlist 1959 1 Prove that the fraction 21n+4 14n+3 ... Let α be the acute angel subtending, from A, that

IMO Shortlist 1969

1 (BEL1) A parabola P1 with equation x2−2py = 0 and parabola P2 with equation x2 +2py =0, p > 0, are given. A line t is tangent to P2. Find the locus of pole M of the line t withrespect to P1.

2 (BEL2)(a) Find the equations of regular hyperbolas passing through the points A(α, 0), B(β, 0),and C(0, γ). (b) Prove that all such hyperbolas pass through the orthocenter H of the triangleABC. (c) Find the locus of the centers of these hyperbolas. (d) Check whether this locuscoincides with the nine-point circle of the triangle ABC.

3 (BEL3) Construct the circle that is tangent to three given circles.

4 (BEL4) Let O be a point on a nondegenerate conic. A right angle with vertex O intersectsthe conic at points A and B. Prove that the line AB passes through a fixed point located onthe normal to the conic through the point O.

5 (BEL5) Let G be the centroid of the triangle OAB. (a) Prove that all conics passing throughthe points O,A,B, G are hyperbolas. (b) Find the locus of the centers of these hyperbolas.

6 (BEL6) Evaluate(cos π

4 + i sin π4

)10 in two different ways and prove that(

101

)−

(103

)+

12

(105

)= 24

7 (BUL1) Prove that the equation√

x3 + y3 + z3 = 1969 has no integral solutions.

8 (BUL2) Find all functions f defined for all x that satisfy the condition xf(y) + yf(x) =(x + y)f(x)f(y), for all x and y. Prove that exactly two of them are continuous.

9 (BUL3) One hundred convex polygons are placed on a square with edge of length 38cm. Thearea of each of the polygons is smaller than πcm2, and the perimeter of each of the polygonsis smaller than 2πcm. Prove that there exists a disk with radius 1 in the square that does notintersect any of the polygons.

10 (BUL4) Let M be the point inside the right-angled triangle ABC(∠C = 90) such that∠MAB = ∠MBC = ∠MCA = φ. Let Ψ be the acute angle between the medians of AC andBC. Prove that sin(φ+Ψ)

sin(φ−Ψ) = 5.

11 (BUL5) Let Z be a set of points in the plane. Suppose that there exists a pair of points thatcannot be joined by a polygonal line not passing through any point of Z. Let us call such apair of points unjoinable. Prove that for each real r > 0 there exists an unjoinable pair ofpoints separated by distance r.

12 (CZS1) Given a unit cube, find the locus of the centroids of all tetrahedra whose vertices lieon the sides of the cube.

http://www.artofproblemsolving.com/This file was downloaded from the AoPS Math Olympiad Resources Page Page 1

Page 33: យុវសិស្ ឈឹ ទីុយ IMO Shortlist 1959-2009 · IMO Shortlist 1959 1 Prove that the fraction 21n+4 14n+3 ... Let α be the acute angel subtending, from A, that

IMO Shortlist 1969

13 (CZS2) Let p be a prime odd number. Is it possible to find p− 1 natural numbers n + 1, n +2, ..., n + p − 1 such that the sum of the squares of these numbers is divisible by the sum ofthese numbers?

14 (CZS3) Let a and b be two positive real numbers. If x is a real solution of the equationx2 + px+ q = 0 with real coefficients p and q such that |p| ≤ a, |q| ≤ b, prove that |x| ≤ 1

2(a+√a2 + 4b) Conversely, if x satisfies the above inequality, prove that there exist real numbers

p and q with |p| ≤ a, |q| ≤ b such that x is one of the roots of the equation x2 + px + q = 0.

15 (CZS4) Let K1, · · · ,Kn be nonnegative integers. Prove that K1!K2! · · ·Kn! ≥[

Kn

]!n, where

K = K1 + · · ·+ Kn

16 (CZS5) A convex quadrilateral ABCD with sides AB = a,BC = b, CD = c,DA = d andangles α = ∠DAB, β = ∠ABC, γ = ∠BCD, and δ = ∠CDA is given. Let s = a+b+c+d

2 andP be the area of the quadrilateral. Prove that P 2 = (s−a)(s−b)(s−c)(s−d)−abcd cos2 α+γ

2

17 (CZS6) Let d and p be two real numbers. Find the first term of an arithmetic progressiona1, a2, a3, · · · with difference d such that a1a2a3a4 = p. Find the number of solutions in termsof d and p.

18 (FRA1) Let a and b be two nonnegative integers. Denote by H(a, b) the set of numbers n ofthe form n = pa + qb, where p and q are positive integers. Determine H(a) = H(a, a). Provethat if a 6= b, it is enough to know all the sets H(a, b) for coprime numbers a, b in order toknow all the sets H(a, b). Prove that in the case of coprime numbers a and b, H(a, b) containsall numbers greater than or equal to ω = (a− 1)(b− 1) and also ω

2 numbers smaller than ω

19 (FRA2) Let n be an integer that is not divisible by any square greater than 1. Denote by xm

the last digit of the number xm in the number system with base n. For which integers x is itpossible for xm to be 0? Prove that the sequence xm is periodic with period t independentof x. For which x do we have xt = 1. Prove that if m and x are relatively prime, then0m, 1m, ..., (n− 1)m are different numbers. Find the minimal period t in terms of n. If n doesnot meet the given condition, prove that it is possible to have xm = 0 6= x1 and that thesequence is periodic starting only from some number k > 1.

20 (FRA3) A polygon (not necessarily convex) with vertices in the lattice points of a rectangulargrid is given. The area of the polygon is S. If I is the number of lattice points that are strictlyin the interior of the polygon and B the number of lattice points on the border of the polygon,find the number T = 2S −B − 2I + 2.

21 (FRA4) A right-angled triangle OAB has its right angle at the point B. An arbitrary circlewith center on the line OB is tangent to the line OA. Let AT be the tangent to the circledifferent from OA (T is the point of tangency). Prove that the median from B of the triangleOAB intersects AT at a point M such that MB = MT.

http://www.artofproblemsolving.com/This file was downloaded from the AoPS Math Olympiad Resources Page Page 2

Page 34: យុវសិស្ ឈឹ ទីុយ IMO Shortlist 1959-2009 · IMO Shortlist 1959 1 Prove that the fraction 21n+4 14n+3 ... Let α be the acute angel subtending, from A, that

IMO Shortlist 1969

22 (FRA5) Let α(n) be the number of pairs (x, y) of integers such that x + y = n, 0 ≤ y ≤ x,and let β(n) be the number of triples (x, y, z) such thatx + y + z = n and 0 ≤ z ≤ y ≤ x.Find a simple relation between α(n) and the integer part of the number n+2

2 and the relationamong β(n), β(n− 3) and α(n). Then evaluate β(n) as a function of the residue of n modulo6. What can be said about β(n) and 1 + n(n+6)

12 ? And what about (n+3)2

6 ? Find the numberof triples (x, y, z) with the property x + y + z ≤ n, 0 ≤ z ≤ y ≤ x as a function of the residueof n modulo 6.What can be said about the relation between this number and the number(n+6)(2n2+9n+12)

72 ?

23 (FRA6) Consider the integer d = ab−1c , where a, b, and c are positive integers and c ≤ a.

Prove that the set G of integers that are between 1 and d and relatively prime to d (thenumber of such integers is denoted by φ(d)) can be partitioned into n subsets, each of whichconsists of b elements. What can be said about the rational number φ(d)

b ?

24 (GBR1) The polynomial P (x) = a0xk + a1x

k−1 + · · · + ak, where a0, · · · , ak are integers, issaid to be divisible by an integer m if P (x) is a multiple of m for every integral value of x.Show that if P (x) is divisible by m, then a0 · k! is a multiple of m. Also prove that if a, k, mare positive integers such that ak! is a multiple of m, then a polynomial P (x) with leadingterm axkcan be found that is divisible by m.

25 (GBR2) Let a, b, x, y be positive integers such that a and b have no common divisor greaterthan 1. Prove that the largest number not expressible in the form ax + by is ab − a − b. IfN(k) is the largest number not expressible in the form ax + by in only k ways, find N(k).

26 (GBR3) A smooth solid consists of a right circular cylinder of height h and base-radius r,surmounted by a hemisphere of radius r and center O. The solid stands on a horizontal table.One end of a string is attached to a point on the base. The string is stretched (initially beingkept in the vertical plane) over the highest point of the solid and held down at the point Pon the hemisphere such that OP makes an angle α with the horizontal. Show that if α issmall enough, the string will slacken if slightly displaced and no longer remain in a verticalplane. If then pulled tight through P , show that it will cross the common circular section ofthe hemisphere and cylinder at a point Q such that ∠SOQ = φ, S being where it initiallycrossed this section, and sinφ = r tan α

h .

27 (GBR4) The segment AB perpendicularly bisects CD at X. Show that, subject to restric-tions, there is a right circular cone whose axis passes through X and on whose surface lie thepoints A,B, C, D. What are the restrictions?

28 (GBR5) Let us define u0 = 0, u1 = 1 and for n ≥ 0, un+2 = aun+1 + bun, a and b beingpositive integers. Express un as a polynomial in a and b. Prove the result. Given that b isprime, prove that b divides a(ub − 1).

http://www.artofproblemsolving.com/This file was downloaded from the AoPS Math Olympiad Resources Page Page 3

Page 35: យុវសិស្ ឈឹ ទីុយ IMO Shortlist 1959-2009 · IMO Shortlist 1959 1 Prove that the fraction 21n+4 14n+3 ... Let α be the acute angel subtending, from A, that

IMO Shortlist 1969

29 (GDR1) Find all real numbers λ such that the equation sin4 x − cos4 x = λ(tan4 x − cot4 x)(a) has no solution, (b) has exactly one solution, (c) has exactly two solutions, (d) has morethan two solutions (in the interval (0, π

4 ).

30 (GDR2)IMO1 Prove that there exist infinitely many natural numbers a with the followingproperty: The number z = n4 + a is not prime for any natural number n.

31 (GDR3) Find the number of permutations a1, · · · , an of the set 1, 2, ..., n such that |ai −ai+1| 6= 1 for all i = 1, 2, ..., n− 1. Find a recurrence formula and evaluate the number of suchpermutations for n ≤ 6.

32 (GDR4) Find the maximal number of regions into which a sphere can be partitioned by ncircles.

33 (GDR5) Given a ring G in the plane bounded by two concentric circles with radii R and R2 ,

prove that we can cover this region with 8 disks of radius 2R5 . (A region is covered if each of

its points is inside or on the border of some disk.)

34 (HUN1) Let a and b be arbitrary integers. Prove that if k is an integer not divisible by 3,then (a + b)2k + a2k + b2k is divisible by a2 + ab + b2

35 (HUN2) Prove that 1 + 123 + 1

33 + · · ·+ 1n3 < 5

4

36 (HUN3) In the plane 4000 points are given such that each line passes through at most 2 ofthese points. Prove that there exist 1000 disjoint quadrilaterals in the plane with vertices atthese points.

37 (HUN4)IMO2 If a1, a2, ..., an are real constants, and if y = cos(a1 +x)+2 cos(a2 +x)+ · · ·+n cos(an +x) has two zeros x1 and x2 whose difference is not a multiple of π, prove that y = 0.

38 (HUN5) Let r and m(r ≤ m) be natural numbers and Ak = 2k−12m π. Evaluate 1

m2

m∑k=1

m∑l=1

sin(rAk) sin(rAl) cos(rAk−

rAl)

39 (HUN6) Find the positions of three points A,B, C on the boundary of a unit cube such thatminAB,AC,BC is the greatest possible.

40 (MON1) Find the number of five-digit numbers with the following properties: there are twopairs of digits such that digits from each pair are equal and are next to each other, digitsfrom different pairs are different, and the remaining digit (which does not belong to any ofthe pairs) is different from the other digits.

http://www.artofproblemsolving.com/This file was downloaded from the AoPS Math Olympiad Resources Page Page 4

Page 36: យុវសិស្ ឈឹ ទីុយ IMO Shortlist 1959-2009 · IMO Shortlist 1959 1 Prove that the fraction 21n+4 14n+3 ... Let α be the acute angel subtending, from A, that

IMO Shortlist 1969

41 (MON2) Given reals x0, x1, α, β, find an expression for the solution of the system

xn+2 − αxn+1 − βxn = 0, n = 0, 1, 2, . . .

42 (MON3) Let Ak(1 ≤ k ≤ h) be n−element sets such that each two of them have a nonemptyintersection. Let A be the union of all the sets Ak, and let B be a subset of A such that foreach k(1 ≤ k ≤ h) the intersection of Ak and B consists of exactly two different elements ak

and bk. Find all subsets X of the set A with r elements satisfying the condition that for atleast one index k, both elements ak and bk belong to X.

43 (MON4) Let p and q be two prime numbers greater than 3. Prove that if their difference is2n, then for any two integers m and n, the number S = p2m+1 + q2m+1 is divisible by 3.

44 (MON5) Find the radius of the circle circumscribed about the isosceles triangle whose sidesare the solutions of the equation x2 − ax + b = 0.

45 Given n > 4 points in the plane, no three collinear. Prove that there are at least (n−3)(n−4)2

convex quadrilaterals with vertices amongst the n points.

46 (NET1) The vertices of an (n + 1)−gon are placed on the edges of a regular n−gon so thatthe perimeter of the n−gon is divided into equal parts. How does one choose these n + 1points in order to obtain the (n + 1)−gon with (a) maximal area; (b) minimal area?

47 C is a point on the semicircle diameter AB, between A and B. D is the foot of the perpen-dicular from C to AB. The circle K1 is the incircle of ABC, the circle K2 touches CD,DAand the semicircle, the circle K3 touches CD,DB and the semicircle. Prove that K1,K2 andK3 have another common tangent apart from AB.

48 (NET3) Let x1, x2, x3, x4, and x5 be positive integers satisfying

x1 + x2 + x3 + x4 + x5 = 1000,

x1 − x2 + x3 − x4 + x5 > 0,

x1 + x2 − x3 + x4 − x5 > 0,

−x1 + x2 + x3 − x4 + x5 > 0,

x1 − x2 + x3 + x4 − x5 > 0,

−x1 + x2 − x3 + x4 + x5 > 0

(a) Find the maximum of (x1 + x3)x2+x4 (b) In how many different ways can we choosex1, ..., x5 to obtain the desired maximum?

http://www.artofproblemsolving.com/This file was downloaded from the AoPS Math Olympiad Resources Page Page 5

Page 37: យុវសិស្ ឈឹ ទីុយ IMO Shortlist 1959-2009 · IMO Shortlist 1959 1 Prove that the fraction 21n+4 14n+3 ... Let α be the acute angel subtending, from A, that

IMO Shortlist 1969

49 (NET4) A boy has a set of trains and pieces of railroad track. Each piece is a quarter ofcircle, and by concatenating these pieces, the boy obtained a closed railway. The railwaydoes not intersect itself. In passing through this railway, the train sometimes goes in theclockwise direction, and sometimes in the opposite direction. Prove that the train passes aneven number of times through the pieces in the clockwise direction and an even number oftimes in the counterclockwise direction. Also, prove that the number of pieces is divisible by4.

50 (NET5) The bisectors of the exterior angles of a pentagon B1B2B3B4B5 form another pen-tagon A1A2A3A4A5. Construct B1B2B3B4B5 from the given pentagon A1A2A3A4A5.

51 (NET6) A curve determined by y =√

x2 − 10x + 52, 0 ≤ x ≤ 100, is constructed in arectangular grid. Determine the number of squares cut by the curve.

52 Prove that a regular polygon with an odd number of edges cannot be partitioned into fourpieces with equal areas by two lines that pass through the center of polygon.

53 (POL2) Given two segments AB and CD not in the same plane, find the locus of points Msuch that MA2 + MB2 = MC2 + MD2.

54 (POL3) Given a polynomial f(x) with integer coefficients whose value is divisible by 3 forthree integers k, k + 1, and k + 2. Prove that f(m) is divisible by 3 for all integers m.

55 For each of k = 1, 2, 3, 4, 5 find necessary and sufficient conditions on a > 0 such that thereexists a tetrahedron with k edges length a and the remainder length 1.

56 Let a and b be two natural numbers that have an equal number n of digits in their decimalexpansions. The first m digits (from left to right) of the numbers a and b are equal. Provethat if m > n

2 , then a1n − b

1n < 1

n

57 Given triangle ABC with points M and N are in the sides AB and AC respectively. IfBM

MA+

CN

NA= 1 , then prove that the centroid of ABC lies on MN .

58 (SWE1) Six points P1, ..., P6 are given in 3−dimensional space such that no four of them liein the same plane. Each of the line segments PjPk is colored black or white. Prove that thereexists one triangle PjPkPl whose edges are of the same color.

59 (SWE2) For each λ(0 < λ < 1 and λ = 1n for all n = 1, 2, 3, · · · ), construct a continuous

function f such that there do not exist x, y with 0 < λ < y = x+λ ≤ 1 for which f(x) = f(y).

60 (SWE3) Find the natural number n with the following properties: (1) Let S = P1, P2, · · · be an arbitrary finite set of points in the plane, and rj the distance from Pj to the origin O.

http://www.artofproblemsolving.com/This file was downloaded from the AoPS Math Olympiad Resources Page Page 6

Page 38: យុវសិស្ ឈឹ ទីុយ IMO Shortlist 1959-2009 · IMO Shortlist 1959 1 Prove that the fraction 21n+4 14n+3 ... Let α be the acute angel subtending, from A, that

IMO Shortlist 1969

We assign to each Pj the closed disk Dj with center Pj and radius rj . Then some n of thesedisks contain all points of S. (2) n is the smallest integer with the above property.

61 (SWE4) Let a0, a1, a2, · · · be determined with a0 = 0, an+1 = 2an + 2n. Prove that if n ispower of 2, then so is an

62 (SWE5) Which natural numbers can be expressed as the difference of squares of two integers?

63 (SWE6) Prove that there are infinitely many positive integers that cannot be expressed asthe sum of squares of three positive integers.

64 (USS1) Prove that for a natural number n > 2, (n!)! > n[(n− 1)!]n!.

65 (USS2) Prove that for a > b2, the identity

√a− b

√a + b

√a− b

√a + · · · =

√a− 3

4b2 − 12b

66 (USS3) (a) Prove that if 0 ≤ a0 ≤ a1 ≤ a2, then (a0 + a1x − a2x2)2 ≤ (a0 + a1 +

a2)2(1 + 1

2x + 13x2 + 1

2x3 + x4)

(b) Formulate and prove the analogous result for polynomialsof third degree.

67 Given real numbers x1, x2, y1, y2, z1, z2 satisfying x1 > 0, x2 > 0, x1y1 > z21 , and x2y2 > z2

2 ,prove that:

8(x1 + x2)(y1 + y2)− (z1 + z2)2

≤ 1x1y1 − z2

1

+1

x2y2 − z22

.

Give necessary and sufficient conditions for equality.

68 (USS5) Given 5 points in the plane, no three of which are collinear, prove that we can choose4 points among them that form a convex quadrilateral.

69 (Y UG1) Suppose that positive real numbers x1, x2, x3 satisfy x1x2x3 > 1, x1 + x2 + x3 <1x1

+ 1x2

+ 1x3

Prove that: (a) None of x1, x2, x3 equals 1. (b) Exactly one of these numbers isless than 1.

70 (Y UG2) A park has the shape of a convex pentagon of area 50000√

3m2. A man standingat an interior point O of the park notices that he stands at a distance of at most 200m fromeach vertex of the pentagon. Prove that he stands at a distance of at least 100m from eachside of the pentagon.

71 (Y UG3) Let four points Ai(i = 1, 2, 3, 4) in the plane determine four triangles. In each ofthese triangles we choose the smallest angle. The sum of these angles is denoted by S. Whatis the exact placement of the points Ai if S = 180?

http://www.artofproblemsolving.com/This file was downloaded from the AoPS Math Olympiad Resources Page Page 7

Page 39: យុវសិស្ ឈឹ ទីុយ IMO Shortlist 1959-2009 · IMO Shortlist 1959 1 Prove that the fraction 21n+4 14n+3 ... Let α be the acute angel subtending, from A, that

IMO Shortlist 1970

1 Consider a regular 2n-gon and the n diagonals of it that pass through its center. Let Pbe a point of the inscribed circle and let a1, a2, . . . , an be the angles in which the diagonalsmentioned are visible from the point P . Prove that

n∑i=1

tan2 ai = 2ncos2 π

2n

sin4 π2n

.

2 We have 0 ≤ xi < b for i = 0, 1, . . . , n and xn > 0, xn−1 > 0. If a > b, and xnxn−1 . . . x0

represents the number A base a and B base b, whilst xn−1xn−2 . . . x0 represents the numberA′ base a and B′ base b, prove that A′B < AB′.

3 In the tetrahedron ABCD,∠BDC = 90o and the foot of the perpendicular from D to ABCis the intersection of the altitudes of ABC. Prove that:

(AB + BC + CA)2 ≤ 6(AD2 + BD2 + CD2).

When do we have equality?

4 Find all positive integers n such that the set n, n + 1, n + 2, n + 3, n + 4, n + 5 can bepartitioned into two subsets so that the product of the numbers in each subset is equal.

5 Let M be an interior point of the tetrahedron ABCD. Prove that

−→MA vol(MBCD)+

−→MB vol(MACD)+

−→MC vol(MABD)+

−→MD vol(MABC) = 0

(vol(PQRS) denotes the volume of the tetrahedron PQRS).

6 In the triangle ABC let B′ and C ′ be the midpoints of the sides AC and AB respectivelyand H the foot of the altitude passing through the vertex A. Prove that the circumcircles ofthe triangles AB′C ′,BC ′H, and B′CH have a common point I and that the line HI passesthrough the midpoint of the segment B′C ′.

7 For which digits a do exist integers n ≥ 4 such that each digit of n(n+1)2 equals a ?

8 M is any point on the side AB of the triangle ABC. r, r1, r2 are the radii of the circlesinscribed in ABC, AMC,BMC. q is the radius of the circle on the opposite side of AB toC, touching the three sides of AB and the extensions of CA and CB. Similarly, q1 and q2.Prove that r1r2q = rq1q2.

9 Let u1, u2, . . . , un, v1, v2, . . . , vn be real numbers. Prove that

1 +n∑

i=1

(ui + vi)2 ≤43

(1 +

n∑i=1

u2i

)(1 +

n∑i=1

v2i

).

http://www.artofproblemsolving.com/This file was downloaded from the AoPS Math Olympiad Resources Page Page 1

Page 40: យុវសិស្ ឈឹ ទីុយ IMO Shortlist 1959-2009 · IMO Shortlist 1959 1 Prove that the fraction 21n+4 14n+3 ... Let α be the acute angel subtending, from A, that

IMO Shortlist 1970

10 The real numbers a0, a1, a2, . . . satisfy 1 = a0 ≤ a1 ≤ a2 ≤ . . . .b1, b2, b3, . . . are defined by

bn =∑n

k=1

1−ak−1

ak√ak

.

a.) Prove that 0 ≤ bn < 2.

b.) Given c satisfying 0 ≤ c < 2, prove that we can find an so that bn > c for all sufficientlylarge n.

11 Let P,Q,R be polynomials and let S(x) = P (x3) + xQ(x3) + x2R(x3) be a polynomial ofdegree n whose roots x1, . . . , xn are distinct. Construct with the aid of the polynomialsP,Q,R a polynomial T of degree n that has the roots x3

1, x32, . . . , x

3n.

12 Given 100 coplanar points, no three collinear, prove that at most 70% of the triangles formedby the points have all angles acute.

http://www.artofproblemsolving.com/This file was downloaded from the AoPS Math Olympiad Resources Page Page 2

Page 41: យុវសិស្ ឈឹ ទីុយ IMO Shortlist 1959-2009 · IMO Shortlist 1959 1 Prove that the fraction 21n+4 14n+3 ... Let α be the acute angel subtending, from A, that

IMO Shortlist 1971

1 Consider a sequence of polynomials P0(x), P1(x), P2(x), . . . , Pn(x), . . ., where P0(x) = 2, P1(x) =x and for every n ≥ 1 the following equality holds:

Pn+1(x) + Pn−1(x) = xPn(x).

Prove that there exist three real numbers a, b, c such that for all n ≥ 1,

(x2 − 4)[P 2n(x)− 4] = [aPn+1(x) + bPn(x) + cPn−1(x)]2.

2 Prove that for every positive integer m we can find a finite set S of points in the plane, suchthat given any point A of S, there are exactly m points in S at unit distance from A.

3 Knowing that the systemx + y + z = 3,

x3 + y3 + z3 = 15,

x4 + y4 + z4 = 35,

has a real solution x, y, z for which x2 + y2 + z2 < 10, find the value of x5 + y5 + z5 for thatsolution.

4 We are given two mutually tangent circles in the plane, with radii r1, r2. A line intersectsthese circles in four points, determining three segments of equal length. Find this length asa function of r1 and r2 and the condition for the solvability of the problem.

5 Let

En = (a1−a2)(a1−a3) . . . (a1−an)+(a2−a1)(a2−a3) . . . (a2−an)+. . .+(an−a1)(an−a2) . . . (an−an−1).

Let Sn be the proposition that En ≥ 0 for all real ai. Prove that Sn is true for n = 3 and 5,but for no other n > 2.

6 Let n ≥ 2 be a natural number. Find a way to assign natural numbers to the vertices of aregular 2n-gon such that the following conditions are satisfied:

(1) only digits 1 and 2 are used;

(2) each number consists of exactly n digits;

(3) different numbers are assigned to different vertices;

(4) the numbers assigned to two neighboring vertices differ at exactly one digit.

http://www.artofproblemsolving.com/This file was downloaded from the AoPS Math Olympiad Resources Page Page 1

Page 42: យុវសិស្ ឈឹ ទីុយ IMO Shortlist 1959-2009 · IMO Shortlist 1959 1 Prove that the fraction 21n+4 14n+3 ... Let α be the acute angel subtending, from A, that

IMO Shortlist 1971

7 All faces of the tetrahedron ABCD are acute-angled. Take a point X in the interior of thesegment AB, and similarly Y in BC, Z in CD and T in AD.

a.) If ∠DAB + ∠BCD 6= ∠CDA + ∠ABC, then prove none of the closed paths XY ZTXhas minimal length;

b.) If ∠DAB + ∠BCD = ∠CDA + ∠ABC, then there are infinitely many shortest pathsXY ZTX, each with length 2AC sin k, where 2k = ∠BAC + ∠CAD + ∠DAB.

8 Determine whether there exist distinct real numbers a, b, c, t for which:

(i) the equation ax2 + btx + c = 0 has two distinct real roots x1, x2,

(ii) the equation bx2 + ctx + a = 0 has two distinct real roots x2, x3,

(iii) the equation cx2 + atx + b = 0 has two distinct real roots x3, x1.

9 Let Tk = k − 1 for k = 1, 2, 3, 4 and

T2k−1 = T2k−2 + 2k−2, T2k = T2k−5 + 2k (k ≥ 3).

Show that for all k,

1 + T2n−1 =[127

2n−1

]and 1 + T2n =

[177

2n−1

],

where [x] denotes the greatest integer not exceeding x.

10 Prove that we can find an infinite set of positive integers of the from 2n − 3 (where n is apositive integer) every pair of which are relatively prime.

11 The matrix

A =

a11 . . . a1n... . . .

...an1 . . . ann

satisfies the inequality

∑nj=1 |aj1x1 + · · · + ajnxn| ≤ M for each choice of numbers xi equal

to ±1. Show that|a11 + a22 + · · ·+ ann| ≥ M.

12 Two congruent equilateral triangles ABC and A′B′C ′ in the plane are given. Show that themidpoints of the segments AA′, BB′, CC ′ either are collinear or form an equilateral triangle.

13 Let A = (aij), where i, j = 1, 2, . . . , n, be a square matrix with all aij non-negative integers.For each i, j such that aij = 0, the sum of the elements in the ith row and the jth column isat least n. Prove that the sum of all the elements in the matrix is at least n2

2 .

http://www.artofproblemsolving.com/This file was downloaded from the AoPS Math Olympiad Resources Page Page 2

Page 43: យុវសិស្ ឈឹ ទីុយ IMO Shortlist 1959-2009 · IMO Shortlist 1959 1 Prove that the fraction 21n+4 14n+3 ... Let α be the acute angel subtending, from A, that

IMO Shortlist 1971

14 A broken line A1A2 . . . An is drawn in a 50×50 square, so that the distance from any point ofthe square to the broken line is less than 1. Prove that its total length is greater than 1248.

15 Natural numbers from 1 to 99 (not necessarily distinct) are written on 99 cards. It is giventhat the sum of the numbers on any subset of cards (including the set of all cards) is notdivisible by 100. Show that all the cards contain the same number.

16 Let P1 be a convex polyhedron with vertices A1, A2, . . . , A9. Let Pi be the polyhedron ob-tained from P1 by a translation that moves A1 to Ai. Prove that at least two of the polyhedraP1, P2, . . . , P9 have an interior point in common.

17 Prove the inequality

a1 + a3

a1 + a2+

a2 + a4

a2 + a3+

a3 + a1

a3 + a4+

a4 + a2

a4 + a1≥ 4,

where ai > 0, i = 1, 2, 3, 4.

http://www.artofproblemsolving.com/This file was downloaded from the AoPS Math Olympiad Resources Page Page 3

Page 44: យុវសិស្ ឈឹ ទីុយ IMO Shortlist 1959-2009 · IMO Shortlist 1959 1 Prove that the fraction 21n+4 14n+3 ... Let α be the acute angel subtending, from A, that

IMO Shortlist 1972

1 f and g are real-valued functions defined on the real line. For all x and y, f(x+y)+f(x−y) =2f(x)g(y). f is not identically zero and |f(x)| ≤ 1 for all x. Prove that |g(x)| ≤ 1 for all x.

2 We are given 3n points A1, A2, . . . , A3n in the plane, no three of them collinear. Prove thatone can construct n disjoint triangles with vertices at the points Ai.

3 The least number is m and the greatest number is M among a1, a2, . . . , an satisfying a1 +a2 + ... + an = 0. Prove that

a21 + · · ·+ a2

n ≤ −nmM

4 Let n1, n2 be positive integers. Consider in a plane E two disjoint sets of points M1 and M2

consisting of 2n1 and 2n2 points, respectively, and such that no three points of the unionM1 ∪ M2 are collinear. Prove that there exists a straightline g with the following property:Each of the two half-planes determined by g on E (g not being included in either) containsexactly half of the points of M1 and exactly half of the points of M2.

5 Prove the following assertion: The four altitudes of a tetrahedron ABCD intersect in a pointif and only if

AB2 + CD2 = BC2 + AD2 = CA2 + BD2.

6 Show that for any n 6≡ 0 (mod 10) there exists a multiple of n not containing the digit 0 inits decimal expansion.

7 Given four distinct parallel planes, prove that there exists a regular tetrahedron with a vertexon each plane.

8 Prove that (2m)!(2n)! is a multiple of m!n!(m + n)! for any non-negative integers m and n.

9 Find all positive real solutions to:

(x21 − x3x5)(x2

2 − x3x5) ≤ 0(x22 − x4x1)(x2

3 − x4x1)≤0(x2

3 − x5x2)(x24 − x5x2) ≤ 0(x2

4 − x1x3)(x25 − x1x3)≤

0(x25 − x2x4)(x2

1 − x2x4) ≤ 0

(0)

Given n > 4, prove that every cyclic quadrilateral can be dissected into n cyclic quadrilaterals.

http://www.artofproblemsolving.com/This file was downloaded from the AoPS Math Olympiad Resources Page Page 1

Page 45: យុវសិស្ ឈឹ ទីុយ IMO Shortlist 1959-2009 · IMO Shortlist 1959 1 Prove that the fraction 21n+4 14n+3 ... Let α be the acute angel subtending, from A, that

IMO Shortlist 1972

Consider a sequence of circles K1,K2,K3,K4, . . . of radii r1, r2, r3, r4, . . . , respectively, situatedinside a triangle ABC. The circle K1 is tangent to AB and AC; K2 is tangent to K1, BA, andBC; K3 is tangent to K2, CA, and CB; K4 is tangent to K3, AB, and AC; etc. (a) Prove therelation

r1 cot12A + 2

√r1r2 + r2 cot

12B = r

(cot

12A + cot

12B

)where r is the radius of the incircle of the triangle ABC. Deduce the existence of a t1 such that

r1 = r cot12B cot

12C sin2 t1

(b) Prove that the sequence of circles K1,K2, . . . is periodic.

Prove that from a set of ten distinct two-digit numbers, it is always possible to find two disjointsubsets whose members have the same sum.

http://www.artofproblemsolving.com/This file was downloaded from the AoPS Math Olympiad Resources Page Page 2

Page 46: យុវសិស្ ឈឹ ទីុយ IMO Shortlist 1959-2009 · IMO Shortlist 1959 1 Prove that the fraction 21n+4 14n+3 ... Let α be the acute angel subtending, from A, that

IMO Shortlist 1973

1 Let a tetrahedron ABCD be inscribed in a sphere S. Find the locus of points P inside thesphere S for which the equality

AP

PA1+

BP

PB1+

CP

PC1+

DP

PD1= 4

holds, where A1, B1, C1, and D1 are the intersection points of S with the lines AP,BP,CP ,and DP , respectively.

2 Given a circle K, find the locus of vertices A of parallelograms ABCD with diagonals AC ≤BD, such that BD is inside K.

3 Prove that the sum of an odd number of vectors of length 1, of common origin O and allsituated in the same semi-plane determined by a straight line which goes through O, is atleast 1.

4 Let P be a set of 7 different prime numbers and C a set of 28 different composite numberseach of which is a product of two (not necessarily different) numbers from P . The set C isdivided into 7 disjoint four-element subsets such that each of the numbers in one set has acommon prime divisor with at least two other numbers in that set. How many such partitionsof C are there ?

5 A circle of radius 1 is located in a right-angled trihedron and touches all its faces. Find thelocus of centers of such circles.

6 Establish if there exists a finite set M of points in space, not all situated in the same plane,so that for any straight line d which contains at least two points from M there exists anotherstraight line d′, parallel with d, but distinct from d, which also contains at least two pointsfrom M .

7 Given a tetrahedron ABCD, let x = AB · CD, y = AC ·BD, and z = AD ·BC. Prove thatthere exists a triangle with edges x, y, z.

8 Prove that there are exactly(

k[k/2]

)arrays a1, a2, . . . , ak+1 of nonnegative integers such that

a1 = 0 and |ai − ai+1| = 1 for i = 1, 2, . . . , k.

9 Let Ox, Oy, Oz be three rays, and G a point inside the trihedron Oxyz. Consider all planespassing through G and cutting Ox, Oy, Oz at points A,B, C, respectively. How is the planeto be placed in order to yield a tetrahedron OABC with minimal perimeter ?

10 Let a1, . . . , an be n positive numbers and 0 < q < 1. Determine n positive numbers b1, . . . , bn

so that:

a.) k < bk for all k = 1, . . . , n, b.) q <bk+1

bk< 1

q for all k = 1, . . . , n−1, c.)n∑

k=1

bk < 1+q1−q ·

n∑k=1

ak.

http://www.artofproblemsolving.com/This file was downloaded from the AoPS Math Olympiad Resources Page Page 1

Page 47: យុវសិស្ ឈឹ ទីុយ IMO Shortlist 1959-2009 · IMO Shortlist 1959 1 Prove that the fraction 21n+4 14n+3 ... Let α be the acute angel subtending, from A, that

IMO Shortlist 1973

11 Determine the minimum value of a2 + b2 when (a, b) traverses all the pairs of real numbersfor which the equation

x4 + ax3 + bx2 + ax + 1 = 0

has at least one real root.

12 Consider the two square matrices

A =

+1 +1 +1 +1 +1+1 +1 +1 −1 −1+1 −1 −1 +1 +1+1 −1 −1 −1 +1+1 +1 −1 +1 −1

and B =

+1 +1 +1 +1 +1+1 +1 +1 −1 −1+1 +1 −1 +1 −1+1 −1 −1 +1 +1+1 −1 +1 −1 +1

with entries +1 and −1. The following operations will be called elementary:

(1) Changing signs of all numbers in one row;

(2) Changing signs of all numbers in one column;

(3) Interchanging two rows (two rows exchange their positions);

(4) Interchanging two columns.

Prove that the matrix B cannot be obtained from the matrix A using these operations.

13 Find the sphere of maximal radius that can be placed inside every tetrahedron that has allaltitudes of length greater than or equal to 1.

14 A soldier needs to check if there are any mines in the interior or on the sides of an equilateraltriangle ABC. His detector can detect a mine at a maximum distance equal to half theheight of the triangle. The soldier leaves from one of the vertices of the triangle. Which isthe minimum distance that he needs to traverse so that at the end of it he is sure that hecompleted successfully his mission?

15 Prove that for all n ∈ N the following is true:

2nn∏

k=1

sinkπ

2n + 1=√

2n + 1

16 Given a, θ ∈ R,m ∈ N, and P (x) = x2m − 2|a|mxm cos θ + a2m, factorize P (x) as a productof m real quadratic polynomials.

http://www.artofproblemsolving.com/This file was downloaded from the AoPS Math Olympiad Resources Page Page 2

Page 48: យុវសិស្ ឈឹ ទីុយ IMO Shortlist 1959-2009 · IMO Shortlist 1959 1 Prove that the fraction 21n+4 14n+3 ... Let α be the acute angel subtending, from A, that

IMO Shortlist 1973

17 G is a set of non-constant functions f . Each f is defined on the real line and has the formf(x) = ax + b for some real a, b. If f and g are in G, then so is fg, where fg is defined byfg(x) = f(g(x)). If f is in G, then so is the inverse f−1. If f(x) = ax+b, then f−1(x) = x−b

a .Every f in G has a fixed point (in other words we can find xf such that f(xf ) = xf . Provethat all the functions in G have a common fixed point.

http://www.artofproblemsolving.com/This file was downloaded from the AoPS Math Olympiad Resources Page Page 3

Page 49: យុវសិស្ ឈឹ ទីុយ IMO Shortlist 1959-2009 · IMO Shortlist 1959 1 Prove that the fraction 21n+4 14n+3 ... Let α be the acute angel subtending, from A, that

IMO Shortlist 1974

1 Three players A,B and C play a game with three cards and on each of these 3 cards it iswritten a positive integer, all 3 numbers are different. A game consists of shuffling the cards,giving each player a card and each player is attributed a number of points equal to the numberwritten on the card and then they give the cards back. After a number (≥ 2) of games wefind out that A has 20 points, B has 10 points and C has 9 points. We also know that in thelast game B had the card with the biggest number. Who had in the first game the card withthe second value (this means the middle card concerning its value).

2 Prove that the squares with sides 11 , 1

2 , 13 , . . . may be put into the square with side 3

2 in sucha way that no two of them have any interior point in common.

3 Let P (x) be a polynomial with integer coefficients. We denote deg(P ) its degree which is≥ 1. Let n(P ) be the number of all the integers k for which we have (P (k))2 = 1. Prove thatn(P )− deg(P ) ≤ 2.

4 The sum of the squares of five real numbers a1, a2, a3, a4, a5 equals 1. Prove that the least ofthe numbers (ai − aj)2, where i, j = 1, 2, 3, 4, 5 and i 6= j, does not exceed 1

10 .

5 Let Ar, Br, Cr be points on the circumference of a given circle S. From the triangle ArBrCr,called ∆r, the triangle ∆r+1 is obtained by constructing the points Ar+1, Br+1, Cr+1on S suchthat Ar+1Ar is parallel to BrCr, Br+1Br is parallel to CrAr, and Cr+1Cr is parallel to ArBr.Each angle of ∆1 is an integer number of degrees and those integers are not multiples of 45.Prove that at least two of the triangles ∆1,∆2, . . . ,∆15 are congruent.

6 Prove that for any n natural, the number

n∑k=0

(2n + 12k + 1

)23k

cannot be divided by 5.

7 Let ai, bi be coprime positive integers for i = 1, 2, . . . , k, and m the least common multipleof b1, . . . , bk. Prove that the greatest common divisor of a1

mb1

, . . . , akmbk

equals the greatestcommon divisor of a1, . . . , ak.

8 The variables a, b, c, d, traverse, independently from each other, the set of positive real values.What are the values which the expression

S =a

a + b + d+

b

a + b + c+

c

b + c + d+

d

a + c + d

takes?

http://www.artofproblemsolving.com/This file was downloaded from the AoPS Math Olympiad Resources Page Page 1

Page 50: យុវសិស្ ឈឹ ទីុយ IMO Shortlist 1959-2009 · IMO Shortlist 1959 1 Prove that the fraction 21n+4 14n+3 ... Let α be the acute angel subtending, from A, that

IMO Shortlist 1974

9 Let x, y, z be real numbers each of whose absolute value is different from 1√3

such that x +y + z = xyz. Prove that

3x− x3

1− 3x2+

3y − y3

1− 3y2+

3z − z3

1− 3z2=

3x− x3

1− 3x2· 3y − y3

1− 3y2· 3z − z3

1− 3z2

10 Let ABC be a triangle. Prove that there exists a point D on the side AB of the triangleABC, such that CD is the geometric mean of AD and DB, iff the triangle ABC satisfies theinequality sinA sinB ≤ sin2 C

2 .

[hide=”Comment”]Alternative formulation, from IMO ShortList 1974, Finland 2: We con-sider a triangle ABC. Prove that: sin(A) sin(B) ≤ sin2

(C2

)is a necessary and sufficient

condition for the existence of a point D on the segment AB so that CD is the geometricalmean of AD and BD.

11 We consider the division of a chess board 8 × 8 in p disjoint rectangles which satisfy theconditions:

a) every rectangle is formed from a number of full squares (not partial) from the 64 and thenumber of white squares is equal to the number of black squares.

b) the numbers a1, . . . , ap of white squares from p rectangles satisfy a1, , . . . , ap. Find thegreatest value of p for which there exists such a division and then for that value of p, all thesequences a1, . . . , ap for which we can have such a division.

12 In a certain language words are formed using an alphabet of three letters. Some words oftwo or more letters are not allowed, and any two such distinct words are of different lengths.Prove that one can form a word of arbitrary length that does not contain any non-allowedword.

http://www.artofproblemsolving.com/This file was downloaded from the AoPS Math Olympiad Resources Page Page 2

Page 51: យុវសិស្ ឈឹ ទីុយ IMO Shortlist 1959-2009 · IMO Shortlist 1959 1 Prove that the fraction 21n+4 14n+3 ... Let α be the acute angel subtending, from A, that

IMO Shortlist 1975

1 There are six ports on a lake. Is it possible to organize a series of routes satisfying thefollowing conditions ? (i) Every route includes exactly three ports; (ii) No two routes containthe same three ports; (iii) The series offers exactly two routes to each tourist who desires tovisit two different arbitrary ports.

2 We consider two sequences of real numbers x1 ≥ x2 ≥ . . . ≥ xn and y1 ≥ y2 ≥ . . . ≥ yn. Let

z1, z2, . . . . , zn be a permutation of the numbers y1, y2, . . . , yn. Prove thatn∑

i=1(xi − yi)2 ≤

n∑i=1

(xi − zi)2.

3 Find the integer represented by[∑109

n=1 n−2/3]. Here [x] denotes the greatest integer less than

or equal to x.

4 Let a1, a2, . . . , an, . . . be a sequence of real numbers such that 0 ≤ an ≤ 1 and an − 2an+1 +an+2 ≥ 0 for n = 1, 2, 3, . . .. Prove that

0 ≤ (n + 1)(an − an+1) ≤ 2 for n = 1, 2, 3, . . .

5 Let M be the set of all positive integers that do not contain the digit 9 (base 10). If x1, . . . , xn

are arbitrary but distinct elements in M , prove that

n∑j=1

1xj

< 80.

6 When 44444444 is written in decimal notation, the sum of its digits is A. Let B be the sum ofthe digits of A. Find the sum of the digits of B. (A and B are written in decimal notation.)

7 Prove that from x + y = 1 (x, y ∈ R) it follows that

xm+1n∑

j=0

(m + j

j

)yj + yn+1

m∑i=0

(n + i

i

)xi = 1 (m,n = 0, 1, 2, . . .).

8 In the plane of a triangle ABC, in its exterior, we draw the triangles ABR,BCP,CAQ sothat ∠PBC = ∠CAQ = 45 , ∠BCP = ∠QCA = 30 , ∠ABR = ∠RAB = 15 .

Prove that

a.) ∠QRP = 90 , and

b.) QR = RP.

http://www.artofproblemsolving.com/This file was downloaded from the AoPS Math Olympiad Resources Page Page 1

Page 52: យុវសិស្ ឈឹ ទីុយ IMO Shortlist 1959-2009 · IMO Shortlist 1959 1 Prove that the fraction 21n+4 14n+3 ... Let α be the acute angel subtending, from A, that

IMO Shortlist 1975

9 Let f(x) be a continuous function defined on the closed interval 0 ≤ x ≤ 1. Let G(f)denote the graph of f(x) : G(f) = (x, y) ∈ R2|0 ≤x ≤ 1, y = f(x). Let Ga(f) denotethe graph of the translated function f(x − a) (translated over a distance a), defined byGa(f) = (x, y) ∈ R2|a ≤ x ≤ a+1, y = f(x−a). Is it possible to find for every a, 0 < a < 1,a continuous function f(x), defined on 0 ≤ x ≤ 1, such that f(0) = f(1) = 0 and G(f) andGa(f) are disjoint point sets ?

10 Determine the polynomials P of two variables so that:

a.) for any real numbers t, x, y we have P (tx, ty) = tnP (x, y) where n is a positive integer,the same for all t, x, y;

b.) for any real numbers a, b, c we have P (a + b, c) + P (b + c, a) + P (c + a, b) = 0;

c.) P (1, 0) = 1.

11 Let a1, . . . , an be an infinite sequence of strictly positive integers, so that ak < ak+1 for anyk. Prove that there exists an infinity of terms m, which can be written like am = x ·ap + y ·aq

with x, y strictly positive integers and p 6= q.

12 Consider on the first quadrant of the trigonometric circle the arcs AM1 = x1, AM2 =x2, AM3 = x3, . . . , AMv = xv , such that x1 < x2 < x3 < · · · < xv. Prove that

v−1∑i=0

sin 2xi −v−1∑i=0

sin(xi − xi+1) <π

2+

v−1∑i=0

sin(xi + xi+1)

13 Let A0, A1, . . . , An be points in a plane such that (i) A0A1 ≤ 12A1A2 ≤ · · · ≤ 1

2n−1 An−1An

and (ii) 0 < ]A0A1A2 < ]A1A2A3 < · · · < ]An−2An−1An < 180, where all these angleshave the same orientation. Prove that the segments AkAk+1, AmAm+1 do not intersect foreach k and n such that 0 ≤ k ≤ m− 2 < n− 2.

14 Let x0 = 5 and xn+1 = xn + 1xn

(n = 0, 1, 2, . . .). Prove that

45 < x1000 < 45.1.

15 Can there be drawn on a circle of radius 1 a number of 1975 distinct points, so that thedistance (measured on the chord) between any two points (from the considered points) is arational number?

http://www.artofproblemsolving.com/This file was downloaded from the AoPS Math Olympiad Resources Page Page 2

Page 53: យុវសិស្ ឈឹ ទីុយ IMO Shortlist 1959-2009 · IMO Shortlist 1959 1 Prove that the fraction 21n+4 14n+3 ... Let α be the acute angel subtending, from A, that

IMO Shortlist 1976

1 Let ABC be a triangle with bisectors AA1, BB1, CC1 (A1 ∈ BC, etc.) and M their com-mon point. Consider the triangles MB1A,MC1A,MC1B,MA1B,MA1C,MB1C, and theirinscribed circles. Prove that if four of these six inscribed circles have equal radii, thenAB = BC = CA.

2 Let a0, a1, . . . , an, an+1 be a sequence of real numbers satisfying the following conditions:

a0 = an+1 = 0,

|ak−1 − 2ak + ak+1| ≤ 1 (k = 1, 2, . . . , n).

Prove that |ak| ≤ k(n+1−k)2 (k = 0, 1, . . . , n + 1).

3 In a convex quadrilateral (in the plane) with the area of 32 cm2 the sum of two opposite sidesand a diagonal is 16 cm. Determine all the possible values that the other diagonal can have.

4 A sequence (un) is defined by

u0 = 2 u1 =52, un+1 = un(u2

n−1 − 2)− u1 for n = 1, . . .

Prove that for any positive integer n we have

[un] = 2(2n−(−1)n)

3

(where [x] denotes the smallest integer ≤ x).

5 We consider the following system with q = 2p:

a11x1 + . . . + a1qxq = 0,a21x1 + . . . + a2qxq = 0,

. . . ,ap1x1 + . . . + apqxq = 0,

in which every coefficient is an element from the set −1, 0, 1. Prove that there exists asolution x1, . . . , xq for the system with the properties:

a.) all xj , j = 1, . . . , q are integers;

b.) there exists at least one j for which xj 6= 0;

c.) |xj | ≤ q for any j = 1, . . . , q.

6 A box whose shape is a parallelepiped can be completely filled with cubes of side 1. If weput in it the maximum possible number of cubes, each ofvolume, 2, with the sides parallel tothose of the box, then exactly 40 percent from the volume of the box is occupied. Determinethe possible dimensions of the box.

http://www.artofproblemsolving.com/This file was downloaded from the AoPS Math Olympiad Resources Page Page 1

Page 54: យុវសិស្ ឈឹ ទីុយ IMO Shortlist 1959-2009 · IMO Shortlist 1959 1 Prove that the fraction 21n+4 14n+3 ... Let α be the acute angel subtending, from A, that

IMO Shortlist 1976

7 Let I = (0, 1] be the unit interval of the real line. For a given number a ∈ (0, 1) we define amap T : I → I by the formula if

T (x, y) =

x + (1− a), if 0 < x ≤ a,

x− a, if a < x ≤ 1.

Show that for every interval J ⊂ I there exists an integer n > 0 such that Tn(J) ∩ J 6= ∅.

8 Let P be a polynomial with real coefficients such that P (x) > 0 if x > 0. Prove that thereexist polynomials Q and R with nonnegative coefficients such that P (x) = Q(x)

R(x) if x > 0.

9 Let P1(x) = x2 − 2 and Pj(x) = P1(Pj−1(x)) for j= 2, . . . Prove that for any positive integern the roots of the equation Pn(x) = x are all real and distinct.

10 Determine the greatest number, who is the product of some positive integers, and the sum ofthese numbers is 1976.

11 Prove that 5n has a block of 1976 consecutive 0′s in its decimal representation.

12 The polynomial 1976(x + x2 + · · ·+ xn) is decomposed into a sum of polynomials of the forma1x+ a2x

2 + · · ·+ anxn, where a1, a2, . . . , an are distinct positive integers not greater than n.Find all values of n for which such a decomposition is possible.

http://www.artofproblemsolving.com/This file was downloaded from the AoPS Math Olympiad Resources Page Page 2

Page 55: យុវសិស្ ឈឹ ទីុយ IMO Shortlist 1959-2009 · IMO Shortlist 1959 1 Prove that the fraction 21n+4 14n+3 ... Let α be the acute angel subtending, from A, that

IMO Shortlist 1977

1 Find all functions f : N → N satisfying following condition:

f(n + 1) > f(f(n)) ∀n ∈ N

2 A lattice point in the plane is a point both of whose coordinates are integers. Each latticepoint has four neighboring points: upper, lower, left, and right. Let k be a circle with radiusr ≥ 2, that does not pass through any lattice point. An interior boundary point is a latticepoint lying inside the circle k that has a neighboring point lying outside k. Similarly, anexterior boundary point is a lattice point lying outside the circle k that has a neighboringpoint lying inside k. Prove that there are four more exterior boundary points than interiorboundary points.

3 Let a, b be two natural numbers. When we divide a2 + b2 by a + b, we the the remainder rand the quotient q. Determine all pairs (a, b) for which q2 + r = 1977.

4 Describe all closed bounded figures Φ in the plane any two points of which are connectableby a semicircle lying in Φ.

5 There are 2n words of length n over the alphabet 0, 1. Prove that the following algorithmgenerates the sequence w0, w1, . . . , w2n−1 of all these words such that any two consecutivewords differ in exactly one digit.

(1) w0 = 00 . . . 0 (n zeros).

(2) Suppose wm−1 = a1a2 . . . an, ai ∈ 0, 1. Let e(m) be the exponent of 2 in the repre-sentation of n as a product of primes, and let j = 1 + e(m). Replace the digit aj in the wordwm−1 by 1− aj . The obtained word is wm.

6 Let n be a positive integer. How many integer solutions (i, j, k, l), 1 ≤ i, j, k, l ≤ n, does thefollowing system of inequalities have:

1 ≤ −j + k + l ≤ n

1 ≤ i− k + l ≤ n

1 ≤ i− j + l ≤ n

1 ≤ i + j − k ≤ n ?

7 Let a, b, A,B be given reals. We consider the function defined by

f(x) = 1− a · cos(x)− b · sin(x)−A · cos(2x)−B · sin(2x).

Prove that if for any real number x we have f(x) ≥ 0 then a2 + b2 ≤ 2 and A2 + B2 ≤ 1.

http://www.artofproblemsolving.com/This file was downloaded from the AoPS Math Olympiad Resources Page Page 1

Page 56: យុវសិស្ ឈឹ ទីុយ IMO Shortlist 1959-2009 · IMO Shortlist 1959 1 Prove that the fraction 21n+4 14n+3 ... Let α be the acute angel subtending, from A, that

IMO Shortlist 1977

8 Let S be a convex quadrilateral ABCD and O a point inside it. The feet of the perpendicularsfrom O to AB,BC, CD,DA are A1, B1, C1, D1 respectively. The feet of the perpendicularsfrom O to the sides of Si, the quadrilateral AiBiCiDi, are Ai+1Bi+1Ci+1Di+1, where i =1, 2, 3. Prove that S4 is similar to S.

9 For which positive integers n do there exist two polynomials f and g with integer coefficientsof n variables x1, x2, . . . , xn such that the following equality is satisfied:

n∑i=1

xif(x1, x2, . . . , xn) = g(x21, x

22, . . . , x

2n) ?

10 Let n be a given number greater than 2. We consider the set Vn of all the integers of the form1 + kn with k = 1, 2, . . . A number m from Vn is called indecomposable in Vn if there are nottwo numbers p and q from Vn so that m = pq. Prove that there exist a number r ∈ Vn thatcan be expressed as the product of elements indecomposable in Vn in more than one way.(Expressions which differ only in order of the elements of Vn will be considered the same.)

11 Let n be an integer greater than 1. Define

x1 = n, y1 = 1, xi+1 =[xi + yi

2

], yi+1 =

[n

xi+1

], for i = 1, 2, . . . ,

where [z] denotes the largest integer less than or equal to z. Prove that

minx1, x2, . . . , xn = [√

n]

12 In the interior of a square ABCD we construct the equilateral triangles ABK,BCL,CDM,DAN.Prove that the midpoints of the four segments KL, LM,MN,NK and the midpoints ofthe eight segments AK, BK,BL, CL,CM,DM,DN,AN are the 12 vertices of a regular do-decagon.

13 Let B be a set of k sequences each having n terms equal to 1 or −1. The product of two suchsequences (a1, a2, . . . , an) and (b1, b2, . . . , bn) is defined as (a1b1, a2b2, . . . , anbn). Prove thatthere exists a sequence (c1, c2, . . . , cn) such that the intersection of B and the set containingall sequences from B multiplied by (c1, c2, . . . , cn) contains at most k2

2n sequences.

14 Let E be a finite set of points such that E is not contained in a plane and no three points ofE are collinear. Show that at least one of the following alternatives holds:

(i) E contains five points that are vertices of a convex pyramid having no other points incommon with E;

(ii) some plane contains exactly three points from E.

http://www.artofproblemsolving.com/This file was downloaded from the AoPS Math Olympiad Resources Page Page 2

Page 57: យុវសិស្ ឈឹ ទីុយ IMO Shortlist 1959-2009 · IMO Shortlist 1959 1 Prove that the fraction 21n+4 14n+3 ... Let α be the acute angel subtending, from A, that

IMO Shortlist 1977

15 In a finite sequence of real numbers the sum of any seven successive terms is negative and thesum of any eleven successive terms is positive. Determine the maximum number of terms inthe sequence.

16 Let E be a set of n points in the plane (n ≥ 3) whose coordinates are integers such that anythree points from E are vertices of a nondegenerate triangle whose centroid doesnt have bothcoordinates integers. Determine the maximal n.

http://www.artofproblemsolving.com/This file was downloaded from the AoPS Math Olympiad Resources Page Page 3

Page 58: យុវសិស្ ឈឹ ទីុយ IMO Shortlist 1959-2009 · IMO Shortlist 1959 1 Prove that the fraction 21n+4 14n+3 ... Let α be the acute angel subtending, from A, that

IMO Shortlist 1978

1 The set M = 1, 2, ..., 2n is partitioned into k nonintersecting subsets M1,M2, . . . ,Mk, wheren ≥ k3 + k. Prove that there exist even numbers 2j1, 2j2, . . . , 2jk+1 in M that are in one andthe same subset Mi (1 ≤ i ≤ k) such that the numbers 2j1− 1, 2j2− 1, . . . , 2jk+1− 1 are alsoin one and the same subset Mj(1 ≤ j ≤ k).

2 Two identically oriented equilateral triangles, ABC with center S and A′B′C, are given inthe plane. We also have A′ 6= S and B′ 6= S. If M is the midpoint of A′B and N the midpointof AB′, prove that the triangles SB′M and SA′N are similar.

3 Let m and n be positive integers such that 1 ≤ m < n. In their decimal representations, thelast three digits of 1978m are equal, respectively, so the last three digits of 1978n. Find mand n such that m + n has its least value.

4 Let T1 be a triangle having a, b, c as lengths of its sides and let T2 be another triangle havingu, v, w as lengths of its sides. If P,Q are the areas of the two triangles, prove that

16PQ ≤ a2(−u2 + v2 + w2) + b2(u2 − v2 + w2) + c2(u2 + v2 − w2).

When does equality hold?

5 For every integer d ≥ 1, let Md be the set of all positive integers that cannot be written as asum of an arithmetic progression with difference d, having at least two terms and consistingof positive integers. Let A = M1, B = M2 \ 2, C = M3. Prove that every c ∈ C may bewritten in a unique way as c = ab with a ∈ A, b ∈ B.

6 Let f be an injective function from 1, 2, 3, . . . in itself. Prove that for any n we have:∑nk=1 f(k)k−2 ≥

∑nk=1 k−1.

7 We consider three distinct half-lines Ox, Oy, Oz in a plane. Prove the existence and unique-ness of three points A ∈ Ox, B ∈ Oy,C ∈ Oz such that the perimeters of the trianglesOAB, OBC,OCA are all equal to a given number 2p > 0.

8 Let S be the set of all the odd positive integers that are not multiples of 5 and that are lessthan 30m, m being an arbitrary positive integer. What is the smallest integer k such that inany subset of k integers from S there must be two different integers, one of which divides theother?

9 Let 0 < f(1) < f(2) < f(3) < . . . a sequence with all its terms positive. The n− th positiveinteger which doesn’t belong to the sequence is f(f(n)) + 1. Find f(240).

10 An international society has its members from six different countries. The list of memberscontain 1978 names, numbered 1, 2, . . . , 1978. Prove that there is at least one member whosenumber is the sum of the numbers of two members from his own country, or twice as large asthe number of one member from his own country.

http://www.artofproblemsolving.com/This file was downloaded from the AoPS Math Olympiad Resources Page Page 1

Page 59: យុវសិស្ ឈឹ ទីុយ IMO Shortlist 1959-2009 · IMO Shortlist 1959 1 Prove that the fraction 21n+4 14n+3 ... Let α be the acute angel subtending, from A, that

IMO Shortlist 1978

11 A function f : I → R, defined on an interval I, is called concave if f(θx + (1 − θ)y) ≥θf(x) + (1 − θ)f(y) for all x, y ∈ I and 0 ≤ θ ≤ 1. Assume that the functions f1, . . . , fn,having all nonnegative values, are concave. Prove that the function (f1f2 · · · fn)1/n is concave.

12 In a triangle ABC we have AB = AC. A circle which is internally tangent with the circum-scribed circle of the triangle is also tangent to the sides AB,AC in the points P, respectivelyQ. Prove that the midpoint of PQ is the center of the inscribed circle of the triangle ABC.

13 We consider a fixed point P in the interior of a fixed sphere. We construct three segmentsPA,PB,PC, perpendicular two by two, with the vertexes A,B, C on the sphere. We considerthe vertex Q which is opposite to P in the parallelepiped (with right angles) with PA,PB,PCas edges. Find the locus of the point Q when A,B, C take all the positions compatible withour problem.

14 Prove that it is possible to place 2n(2n + 1) parallelepipedic (rectangular) pieces of soap ofdimensions 1× 2× (n + 1) in a cubic box with edge 2n + 1 if and only if n is even or n = 1.

Remark. It is assumed that the edges of the pieces of soap are parallel to the edges of thebox.

15 Let p be a prime and A = a1, . . . , ap−1 an arbitrary subset of the set of natural numberssuch that none of its elements is divisible by p. Let us define a mapping f from P(A) (theset of all subsets of A) to the set P = 0, 1, . . . , p− 1 in the following way:

(i) if B = ai1 , . . . , aik ⊂ A and∑k

j=1 aij ≡ n (mod p), then f(B) = n,

(ii) f(∅) = 0, ∅ being the empty set.

Prove that for each n ∈ P there exists B ⊂ A such that f(B) = n.

16 Determine all the triples (a, b, c) of positive real numbers such that the system

ax + by − cz = 0,

a√

1− x2 + b√

1− y2 − c√

1− z2 = 0,

is compatible in the set of real numbers, and then find all its real solutions.

17 Prove that for any positive integers x, y, z with xy−z2 = 1 one can find non-negative integersa, b, c, d such that x = a2 + b2, y = c2 + d2, z = ac + bd. Set z = (2q)! to deduce that for anyprime number p = 4q + 1, p can be represented as the sum of squares of two integers.

http://www.artofproblemsolving.com/This file was downloaded from the AoPS Math Olympiad Resources Page Page 2

Page 60: យុវសិស្ ឈឹ ទីុយ IMO Shortlist 1959-2009 · IMO Shortlist 1959 1 Prove that the fraction 21n+4 14n+3 ... Let α be the acute angel subtending, from A, that

IMO Shortlist 1979

1 Prove that in the Euclidean plane every regular polygon having an even number of sides canbe dissected into lozenges. (A lozenge is a quadrilateral whose four sides are all of equallength).

2 From a bag containing 5 pairs of socks, each pair a different color, a random sample of 4single socks is drawn. Any complete pairs in the sample are discarded and replaced by a newpair draw from the bag. The process continues until the bag is empty or there are 4 socks ofdifferent colors held outside the bag. What is the probability of the latter alternative?

3 Find all polynomials f(x) with real coefficients for which

f(x)f(2x2) = f(2x3 + x).

4 We consider a prism which has the upper and inferior basis the pentagons: A1A2A3A4A5

and B1B2B3B4B5. Each of the sides of the two pentagons and the segments AiBj withi, j = 1, . . . , 5 is colored in red or blue. In every triangle which has all sides colored thereexists one red side and one blue side. Prove that all the 10 sides of the two basis are coloredin the same color.

5 Let n ≥ 2 be an integer. Find the maximal cardinality of a set M of pairs (j, k) of integers,1 ≤ j < k ≤ n, with the following property: If (j, k) ∈ M , then (k, m) 6∈ M for any m.

6 Find the real values of p for which the equation√2p + 1− x2 +

√3x + p + 4 =

√x2 + 9x + 3p + 9

in x has exactly two real distinct roots.(√

t means the positive square root of t).

7 If p and q are natural numbers so that

p

q= 1− 1

2+

13− 1

4+ . . .− 1

1318+

11319

,

prove that p is divisible with 1979.

8 For all rational x satisfying 0 ≤ x < 1, f is defined by

f(x) =

f(2x)

4 , for 0 ≤ x < 12 ,

34 + f(2x−1)

4 , for 12 ≤ x < 1.

Given that x = 0.b1b2b3 · · · is the binary representation of x, find, with proof, f(x).

http://www.artofproblemsolving.com/This file was downloaded from the AoPS Math Olympiad Resources Page Page 1

Page 61: យុវសិស្ ឈឹ ទីុយ IMO Shortlist 1959-2009 · IMO Shortlist 1959 1 Prove that the fraction 21n+4 14n+3 ... Let α be the acute angel subtending, from A, that

IMO Shortlist 1979

9 Let A and E be opposite vertices of an octagon. A frog starts at vertex A. From any vertexexcept E it jumps to one of the two adjacent vertices. When it reaches E it stops. Let an bethe number of distinct paths of exactly n jumps ending at E. Prove that:

a2n−1 = 0, a2n =(2 +

√2)n−1 − (2−

√2)n−1

√2

.

10 Show that for any vectors a, b in Euclidean space,

|a× b|3 ≤ 3√

38

|a|2|b|2|a− b|2

Remark. Here × denotes the vector product.

11 Given real numbers x1, x2, . . . , xn (n ≥ 2), with xi ≥ 1n (i = 1, 2, . . . , n) and with x2

1 + x22 +

· · · + x2n = 1 , find whether the product P = x1x2x3 · · ·xn has a greatest and/or least value

and if so, give these values.

12 Let R be a set of exactly 6 elements. A set F of subsets of R is called an S-family over R ifand only if it satisfies the following three conditions: (i) For no two sets X, Y in F is X ⊆ Y;

(ii) For any three sets X, Y, Z in F , X ∪ Y ∪ Z 6= R,

(iii)⋃

X∈F X = R

13 Show that 2060 < sin 20 < 21

60 .

14 Find all bases of logarithms in which a real positive number can be equal to its logarithm orprove that none exist.

15 Determine all real numbers a for which there exists positive reals x1, . . . , x5 which satisfy therelations

∑5k=1 kxk = a,

∑5k=1 k3xk = a2,

∑5k=1 k5xk = a3.

16 Let K denote the set a, b, c, d, e. F is a collection of 16 different subsets of K, and it isknown that any three members of F have at least one element in common. Show that all 16members of F have exactly one element in common.

17 Inside an equilateral triangle ABC one constructs points P,Q and R such that

∠QAB = ∠PBA = 15,∠RBC = ∠QCB = 20,∠PCA = ∠RAC = 25.

Determine the angles of triangle PQR.

http://www.artofproblemsolving.com/This file was downloaded from the AoPS Math Olympiad Resources Page Page 2

Page 62: យុវសិស្ ឈឹ ទីុយ IMO Shortlist 1959-2009 · IMO Shortlist 1959 1 Prove that the fraction 21n+4 14n+3 ... Let α be the acute angel subtending, from A, that

IMO Shortlist 1979

18 Let m positive integers a1, . . . , am be given. Prove that there exist fewer than 2m positiveintegers b1, . . . , bn such that all sums of distinct bks are distinct and all ai (i ≤ m) occuramong them.

19 Consider the sequences (an), (bn) defined by

a1 = 3, b1 = 100, an+1 = 3an , bn+1 = 100bn

Find the smallest integer m for which bm > a100.

20 Given the integer n > 1 and the real number a > 0 determine the maximum of∑n−1

i=1 xixi+1

taken over all nonnegative numbers xi with sum a.

21 Let N be the number of integral solutions of the equation

x2 − y2 = z3 − t3

satisfying the condition 0 ≤ x, y, z, t ≤ 106, and let M be the number of integral solutions ofthe equation

x2 − y2 = z3 − t3 + 1

satisfying the condition 0 ≤ x, y, z, t ≤ 106. Prove that N > M.

22 Two circles in a plane intersect. A is one of the points of intersection. Starting simultaneouslyfrom A two points move with constant speed, each travelling along its own circle in the samesense. The two points return to A simultaneously after one revolution. Prove that there is afixed point P in the plane such that the two points are always equidistant from P.

23 Find all natural numbers n for which 28 + 211 + 2n is a perfect square.

24 A circle C with center O on base BC of an isosceles triangle ABC is tangent to the equal sidesAB,AC. If point P on AB and point Q on AC are selected such that PB × CQ = (BC

2 )2,prove that line segment PQ is tangent to circle C, and prove the converse.

25 We consider a point P in a plane p and a point Q 6∈ p. Determine all the points R from p forwhich

QP + PR

QR

is maximum.

26 Prove that the functional equations

f(x + y) = f(x) + f(y),

and f(x + y + xy) = f(x) + f(y) + f(xy) (x, y ∈ R)

are equivalent.

http://www.artofproblemsolving.com/This file was downloaded from the AoPS Math Olympiad Resources Page Page 3

Page 63: យុវសិស្ ឈឹ ទីុយ IMO Shortlist 1959-2009 · IMO Shortlist 1959 1 Prove that the fraction 21n+4 14n+3 ... Let α be the acute angel subtending, from A, that

IMO Shortlist 1980

1 Let α, β and γ denote the angles of the triangle ABC. The perpendicular bisector of ABintersects BC at the point X, the perpendicular bisector of AC intersects it at Y . Prove thattan(β) · tan(γ) = 3 implies BC = XY (or in other words: Prove that a sufficient conditionfor BC = XY is tan(β) · tan(γ) = 3). Show that this condition is not necessary, and give anecessary and sufficient condition for BC = XY .

2 Define the numbers a0, a1, . . . , an in the following way:

a0 =12, ak+1 = ak +

a2k

n(n > 1, k = 0, 1, . . . , n− 1).

Prove that1− 1

n< an < 1.

3 Prove that the equationxn + 1 = yn+1,

where n is a positive integer not smaller then 2, has no positive integer solutions in x and yfor which x and n + 1 are relatively prime.

4 Determine all positive integers n such that the following statement holds: If a convex polygonwith with 2n sides A1A2 . . . A2n is inscribed in a circle and n − 1 of its n pairs of oppositesides are parallel, which means if the pairs of opposite sides

(A1A2, An+1An+2), (A2A3, An+2An+2), . . . , (An−1An, A2n−1A2n)

are parallel, then the sidesAnAn+1, A2nA1

are parallel as well.

5 In a rectangular coordinate system we call a horizontal line parallel to the x -axis triangularif it intersects the curve with equation

y = x4 + px3 + qx2 + rx + s

in the points A,B, C and D (from left to right) such that the segments AB,AC and AD arethe sides of a triangle. Prove that the lines parallel to the x - axis intersecting the curve infour distinct points are all triangular or none of them is triangular.

6 Find the digits left and right of the decimal point in the decimal form of the number

(√

2 +√

3)1980.

http://www.artofproblemsolving.com/This file was downloaded from the AoPS Math Olympiad Resources Page Page 1

Page 64: យុវសិស្ ឈឹ ទីុយ IMO Shortlist 1959-2009 · IMO Shortlist 1959 1 Prove that the fraction 21n+4 14n+3 ... Let α be the acute angel subtending, from A, that

IMO Shortlist 1980

7 The function f is defined on the set Q of all rational numbers and has values in Q. It satisfiesthe conditions f(1) = 2 and f(xy) = f(x)f(y)− f(x + y) + 1 for all x, y ∈ Q. Determine f .

8 Three points A,B, C are such that B ∈]AC[. On the side of AC we draw the three semicircleswith diameters [AB], [BC] and [AC]. The common interior tangent at B to the first two semi-circles meets the third circle in E. Let U and V be the points of contact of the common exteriortangent to the first two semi-circles. Denote the are of the triangle ABC as area(ABC). Thenplease evaluate the ratio R = area(EUV )/area(EAC) as a function of r1 = AB

2 and r2 = BC2 .

9 Let p be a prime number. Prove that there is no number divisible by p in the n− th row ofPascal’s triangle if and only if n can be represented in the form n = psq − 1, where s and qare integers with s ≥ 0, 0 < q < p.

10 Two circles C1 and C2 are (externally or internally) tangent at a point P . The straight lineD is tangent at A to one of the circles and cuts the other circle at the points B and C. Provethat the straight line PA is an interior or exterior bisector of the angle ∠BPC.

11 Ten gamblers started playing with the same amount of money. Each turn they cast (threw)five dice. At each stage the gambler who had thrown paid to each of his 9 opponents 1

n timesthe amount which that opponent owned at that moment. They threw and paid one afterthe other. At the 10th round (i.e. when each gambler has cast the five dice once), the diceshowed a total of 12, and after payment it turned out that every player had exactly the samesum as he had at the beginning. Is it possible to determine the total shown by the dice atthe nine former rounds ?

12 Find all pairs of solutions (x, y):

x3 + x2y + xy2 + y3 = 8(x2 + xy + y2 + 1).

13 Given three infinite arithmetic progressions of natural numbers such that each of the numbers1,2,3,4,5,6,7 and 8 belongs to at least one of them, prove that the number 1980 also belongsto at least one of them.

14 Let xn be a sequence of natural numbers such that

(a)1 = x1 < x2 < x3 < . . . ; (b)x2n+1 ≤ 2n ∀n.

Prove that, for every natural number k, there exist terms xr and xs such that xr − xs = k.

15 Prove that the sum of the six angles subtended at an interior point of a tetrahedron by itssix edges is greater than 540.

16 Prove that∑ 1

i1i2...ik = n is taken over all non-empty subsets i1, i2, . . . , ik.

http://www.artofproblemsolving.com/This file was downloaded from the AoPS Math Olympiad Resources Page Page 2

Page 65: យុវសិស្ ឈឹ ទីុយ IMO Shortlist 1959-2009 · IMO Shortlist 1959 1 Prove that the fraction 21n+4 14n+3 ... Let α be the acute angel subtending, from A, that

IMO Shortlist 1980

17 Let A1A2A3 be a triangle and, for 1 ≤ i ≤ 3, let Bi be an interior point of edge opposite Ai.Prove that the perpendicular bisectors of AiBi for 1 ≤ i ≤ 3 are not concurrent.

18 Given a sequence an of real numbers such that |ak+m−ak−am| ≤ 1 for all positive integersk and m, prove that, for all positive integers p and q,

|ap

p− aq

q| < 1

p+

1q.

19 Find the greatest natural number n such there exist natural numbers x1, x2, . . . , xn and nat-ural a1 < a2 < . . . < an−1 satisfying the following equations for i = 1, 2, . . . , n− 1:

x1x2 . . . xn = 1980 and xi +1980xi

= ai.

20 Let S be a set of 1980 points in the plane such that the distance between every pair of themis at least 1. Prove that S has a subset of 220 points such that the distance between everypair of them is at least

√3.

21 Let AB be a diameter of a circle; let t1 and t2 be the tangents at A and B, respectively; letC be any point other than A on t1; and let D1D2.E1E2 be arcs on the circle determined bytwo lines through C. Prove that the lines AD1 and AD2 determine a segment on t2 equal inlength to that of the segment on t2 determined by AE1 and AE2.

http://www.artofproblemsolving.com/This file was downloaded from the AoPS Math Olympiad Resources Page Page 3

Page 66: យុវសិស្ ឈឹ ទីុយ IMO Shortlist 1959-2009 · IMO Shortlist 1959 1 Prove that the fraction 21n+4 14n+3 ... Let α be the acute angel subtending, from A, that

IMO Shortlist 1981

1 a.) For which n > 2 is there a set of n consecutive positive integers such that the largestnumber in the set is a divisor of the least common multiple of the remaining n− 1 numbers?

b.) For which n > 2 is there exactly one set having this property?

2 A sphere S is tangent to the edges AB,BC, CD,DA of a tetrahedron ABCD at the pointsE,F, G,H respectively. The points E,F, G,H are the vertices of a square. Prove that if thesphere is tangent to the edge AC, then it is also tangent to the edge BD.

3 Find the minimum value of

max(a + b + c, b + c + d, c + d + e, d + e + f, e + f + g)

subject to the constraints

(i) a, b, c, d, e, f, g ≥ 0,

(ii)a + b + c + d + e + f + g = 1.

4 Let fn be the Fibonacci sequence 1, 1, 2, 3, 5, . . . ..(a) Find all pairs (a, b) of real numbers such that for each n, afn + bfn+1 is a member of thesequence.

(b) Find all pairs (u, v) of positive real numbers such that for each n, uf2n +vf2

n+1 is a memberof the sequence.

5 A cube is assembled with 27 white cubes. The larger cube is then painted black on the outsideand disassembled. A blind man reassembles it. What is the probability that the cube is nowcompletely black on the outside? Give an approximation of the size of your answer.

6 Let P (z) and Q(z) be complex-variable polynomials, with degree not less than 1. Let

Pk = z ∈ C|P (z) = k, Qk = z ∈ C|Q(z) = k.

Let also P0 = Q0 and P1 = Q1. Prove that P (z) ≡ Q(z).

7 The function f(x, y) satisfies: f(0, y) = y+1, f(x+1, 0) = f(x, 1), f(x+1, y+1) = f(x, f(x+1, y)) for all non-negative integers x, y. Find f(4, 1981).

8 Take r such that 1 ≤ r ≤ n, and consider all subsets of r elements of the set 1, 2, . . . , n.Each subset has a smallest element. Let F (n, r) be the arithmetic mean of these smallestelements. Prove that:

F (n, r) =n + 1r + 1

.

http://www.artofproblemsolving.com/This file was downloaded from the AoPS Math Olympiad Resources Page Page 1

Page 67: យុវសិស្ ឈឹ ទីុយ IMO Shortlist 1959-2009 · IMO Shortlist 1959 1 Prove that the fraction 21n+4 14n+3 ... Let α be the acute angel subtending, from A, that

IMO Shortlist 1981

9 A sequence (an) is defined by means of the recursion

a1 = 1, an+1 =1 + 4an +

√1 + 24an

16.

Find an explicit formula for an.

10 Determine the smallest natural number n having the following property: For every integerp, p ≥ n, it is possible to subdivide (partition) a given square into p squares (not necessarilyequal).

11 On a semicircle with unit radius four consecutive chords AB,BC,CD,DE with lengthsa, b, c, d, respectively, are given. Prove that

a2 + b2 + c2 + d2 + abc + bcd < 4.

12 Determine the maximum value of m2+n2, where m and n are integers in the range 1, 2, . . . , 1981satisfying (n2 −mn−m2)2 = 1.

13 Let P be a polynomial of degree n satisfying

P (k) =(

n + 1k

)−1

for k = 0, 1, ..., n.

Determine P (n + 1).

14 Prove that a convex pentagon (a five-sided polygon) ABCDE with equal sides and for whichthe interior angles satisfy the condition ∠A ≥ ∠B ≥ ∠C ≥ ∠D ≥ ∠E is a regular pentagon.

15 Consider a variable point P inside a given triangle ABC. Let D, E, F be the feet of theperpendiculars from the point P to the lines BC, CA, AB, respectively. Find all points Pwhich minimize the sum

BC

PD+

CA

PE+

AB

PF.

16 A sequence of real numbers u1, u2, u3, . . . is determined by u1 and the following recurrencerelation for n ≥ 1:

4un+1 = 3√

64un + 15.

Describe, with proof, the behavior of un as n →∞.

17 Three circles of equal radius have a common point O and lie inside a given triangle. Eachcircle touches a pair of sides of the triangle. Prove that the incenter and the circumcenter ofthe triangle are collinear with the point O.

http://www.artofproblemsolving.com/This file was downloaded from the AoPS Math Olympiad Resources Page Page 2

Page 68: យុវសិស្ ឈឹ ទីុយ IMO Shortlist 1959-2009 · IMO Shortlist 1959 1 Prove that the fraction 21n+4 14n+3 ... Let α be the acute angel subtending, from A, that

IMO Shortlist 1981

18 Several equal spherical planets are given in outer space. On the surface of each planet thereis a set of points that is invisible from any of the remaining planets. Prove that the sum ofthe areas of all these sets is equal to the area of the surface of one planet.

19 A finite set of unit circles is given in a plane such that the area of their union U is S. Provethat there exists a subset of mutually disjoint circles such that the area of their union isgreater that 2S

9 .

http://www.artofproblemsolving.com/This file was downloaded from the AoPS Math Olympiad Resources Page Page 3

Page 69: យុវសិស្ ឈឹ ទីុយ IMO Shortlist 1959-2009 · IMO Shortlist 1959 1 Prove that the fraction 21n+4 14n+3 ... Let α be the acute angel subtending, from A, that

IMO Shortlist 1982

1 The function f(n) is defined on the positive integers and takes non-negative integer values.f(2) = 0, f(3) > 0, f(9999) = 3333 and for all m,n :

f(m + n)− f(m)− f(n) = 0 or 1.

Determine f(1982).

2 Let K be a convex polygon in the plane and suppose that K is positioned in the coordinatesystem in such a way that

area (K ∩Qi) =14area K(i = 1, 2, 3, 4, ),

where the Qi denote the quadrants of the plane. Prove that if K contains no nonzero latticepoint, then the area of K is less than 4.

3 Consider infinite sequences xn of positive reals such that x0 = 1 and x0 ≥ x1 ≥ x2 ≥ . . ..

a.) Prove that for every such sequence there is an n ≥ 1 such that:

x20

x1+

x21

x2+ . . . +

x2n−1

xn≥ 3.999.

b.) Find such a sequence such that for all n:

x20

x1+

x21

x2+ . . . +

x2n−1

xn< 4.

4 Determine all real values of the parameter a for which the equation

16x4 − ax3 + (2a + 17)x2 − ax + 16 = 0

has exactly four distinct real roots that form a geometric progression.

5 The diagonals AC and CE of the regular hexagon ABCDEF are divided by inner points Mand N respectively, so that

AM

AC=

CN

CE= r.

Determine r if B,M and N are collinear.

6 Let S be a square with sides length 100. Let L be a path within S which does not meetitself and which is composed of line segments A0A1, A1A2, A2A3, . . . , An−1An with A0 = An.Suppose that for every point P on the boundary of S there is a point of L at a distance fromP no greater than 1

2 . Prove that there are two points X and Y of L such that the distancebetween X and Y is not greater than 1 and the length of the part of L which lies between Xand Y is not smaller than 198.

http://www.artofproblemsolving.com/This file was downloaded from the AoPS Math Olympiad Resources Page Page 1

Page 70: យុវសិស្ ឈឹ ទីុយ IMO Shortlist 1959-2009 · IMO Shortlist 1959 1 Prove that the fraction 21n+4 14n+3 ... Let α be the acute angel subtending, from A, that

IMO Shortlist 1982

7 Let p(x) be a cubic polynomial with integer coefficients with leading coefficient 1 and withone of its roots equal to the product of the other two. Show that 2p(−1) is a multiple ofp(1) + p(−1)− 2(1 + p(0)).

8 A convex, closed figure lies inside a given circle. The figure is seen from every point of thecircumference at a right angle (that is, the two rays drawn from the point and supporting theconvex figure are perpendicular). Prove that the center of the circle is a center of symmetryof the figure.

9 Let ABC be a triangle, and let P be a point inside it such that ∠PAC = ∠PBC. Theperpendiculars from P to BC and CA meet these lines at L and M , respectively, and D isthe midpoint of AB. Prove that DL = DM.

10 A box contains p white balls and q black balls. Beside the box there is a pile of black balls.Two balls are taken out of the box. If they have the same color, a black ball from the pile isput into the box. If they have different colors, the white ball is put back into the box. Thisprocedure is repeated until the last two balls are removed from the box and one last ball isput in. What is the probability that this last ball is white?

11 (a) Find the rearrangement a1, . . . , an of 1, 2, . . . , n that maximizes

a1a2 + a2a3 + · · ·+ ana1 = Q.

(b) Find the rearrangement that minimizes Q.

12 Four distinct circles C,C1, C2, C3 and a line L are given in the plane such that C and L aredisjoint and each of the circles C1, C2, C3 touches the other two, as well as C and L. Assumingthe radius of C to be 1, determine the distance between its center and L.

13 A non-isosceles triangle A1A2A3 has sides a1, a2, a3 with the side ai lying opposite to thevertex Ai. Let Mi be the midpoint of the side ai, and let Ti be the point where the inscribedcircle of triangle A1A2A3 touches the side ai. Denote by Si the reflection of the point Ti inthe interior angle bisector of the angle Ai. Prove that the lines M1S1, M2S2 and M3S3 areconcurrent.

14 Let ABCD be a convex plane quadrilateral and let A1 denote the circumcenter of 4BCD.Define B1, C1, D1 in a corresponding way.

(a) Prove that either all of A1, B1, C1, D1 coincide in one point, or they are all distinct.Assuming the latter case, show that A1, C1 are on opposite sides of the line B1D1, andsimilarly,B1, D1 are on opposite sides of the line A1C1. (This establishes the convexity of thequadrilateral A1B1C1D1.)

(b) Denote by A2 the circumcenter of B1C1D1, and define B2, C2, D2 in an analogous way.Show that the quadrilateral A2B2C2D2 is similar to the quadrilateral ABCD.

http://www.artofproblemsolving.com/This file was downloaded from the AoPS Math Olympiad Resources Page Page 2

Page 71: យុវសិស្ ឈឹ ទីុយ IMO Shortlist 1959-2009 · IMO Shortlist 1959 1 Prove that the fraction 21n+4 14n+3 ... Let α be the acute angel subtending, from A, that

IMO Shortlist 1982

15 15. C3 (CAN 5) Show that1− sa

1− s≤ (1 + s)a−1

holds for every 1 6= s > 0real and 0 < a ≤ 1 rational.

16 Prove that if n is a positive integer such that the equation

x3 − 3xy2 + y3 = n

has a solution in integers x, y, then it has at least three such solutions. Show that the equationhas no solutions in integers for n = 2891.

17 The right triangles ABC and AB1C1 are similar and have opposite orientation. The rightangles are at C and C1, and we also have ∠CAB = ∠C1AB1. Let M be the point ofintersection of the lines BC1 and B1C. Prove that if the lines AM and CC1 exist, they areperpendicular.

18 Let O be a point of three-dimensional space and let l1, l2, l3 be mutually perpendicular straightlines passing through O. Let S denote the sphere with center O and radius R, and for everypoint M of S, let SM denote the sphere with center M and radius R. We denote by P1, P2, P3

the intersection of SM with the straight lines l1, l2, l3, respectively, where we put Pi 6= O if limeets SM at two distinct points and Pi = O otherwise (i = 1, 2, 3). What is the set of centersof gravity of the (possibly degenerate) triangles P1P2P3 as M runs through the points of S ?

19 Let M be the set of real numbers of the form m+n√m2+n2

, where m and n are positive integers.Prove that for every pair x ∈ M,y ∈ M with x < y, there exists an element z ∈ M such thatx < z < y.

20 Let ABCD be a convex quadrilateral and draw regular triangles ABM, CDP, BCN,ADQ,the first two outward and the other two inward. Prove that MN = AC. What can be saidabout the quadrilateral MNPQ ?

http://www.artofproblemsolving.com/This file was downloaded from the AoPS Math Olympiad Resources Page Page 3

Page 72: យុវសិស្ ឈឹ ទីុយ IMO Shortlist 1959-2009 · IMO Shortlist 1959 1 Prove that the fraction 21n+4 14n+3 ... Let α be the acute angel subtending, from A, that

IMO Shortlist 1983

1 The localities P1, P2, . . . , P1983 are served by ten international airlines A1, A2, . . . , A10. It isnoticed that there is direct service (without stops) between any two of these localities andthat all airline schedules offer round-trip flights. Prove that at least one of the airlines canoffer a round trip with an odd number of landings.

2 Let n be a positive integer. Let σ(n) be the sum of the natural divisors d of n (including 1 andn). We say that an integer m ≥ 1 is superabundant (P.Erdos, 1944) if ∀k ∈ 1, 2, . . . ,m− 1,σ(m)

m > σ(k)k . Prove that there exists an infinity of superabundant numbers.

3 Let ABC be an equilateral triangle and E the set of all points contained in the three segmentsAB, BC, and CA (including A, B, and C). Determine whether, for every partition of E intotwo disjoint subsets, at least one of the two subsets contains the vertices of a right-angledtriangle.

4 On the sides of the triangle ABC, three similar isosceles triangles ABP (AP = PB),AQC (AQ = QC), and BRC (BR = RC) are constructed. The first two are constructedexternally to the triangle ABC, but the third is placed in the same half-plane determined bythe line BC as the triangle ABC. Prove that APRQ is a parallelogram.

5 Consider the set of all strictly decreasing sequences of n natural numbers having the propertythat in each sequence no term divides any other term of the sequence. Let A = (aj) andB = (bj) be any two such sequences. We say that A precedes B if for some k, ak < bk andai = bi for i < k. Find the terms of the first sequence of the set under this ordering.

6 Suppose that x1, x2, . . . , xn are positive integers for which x1 +x2 + · · ·+xn = 2(n+1). Showthat there exists an integer r with 0 ≤ r ≤ n − 1 for which the following n − 1 inequalitieshold:

xr+1 + · · ·+ xr+i ≤ 2i + 1, ∀i, 1 ≤ i ≤ n− r;

xr+1 + · · ·+ xn + x1 + · · ·+ xi ≤ 2(n− r + i) + 1, ∀i, 1 ≤ i ≤ r − 1.

Prove that if all the inequalities are strict, then r is unique and that otherwise there areexactly two such r.

7 Let a be a positive integer and let an be defined by a0 = 0 and

an+1 = (an + 1)a + (a + 1)an + 2√

a(a + 1)an(an + 1) (n = 1, 2, . . . ).

Show that for each positive integer n, an is a positive integer.

8 In a test, 3n students participate, who are located in three rows of n students in each. Thestudents leave the test room one by one. If N1(t), N2(t), N3(t) denote the numbers of studentsin the first, second, and third row respectively at time t, find the probability that for each tduring the test,

|Ni(t)−Nj(t)| < 2, i 6= j, i, j = 1, 2, . . . .

http://www.artofproblemsolving.com/This file was downloaded from the AoPS Math Olympiad Resources Page Page 1

Page 73: យុវសិស្ ឈឹ ទីុយ IMO Shortlist 1959-2009 · IMO Shortlist 1959 1 Prove that the fraction 21n+4 14n+3 ... Let α be the acute angel subtending, from A, that

IMO Shortlist 1983

9 Let a, b and c be the lengths of the sides of a triangle. Prove that

a2b(a− b) + b2c(b− c) + c2a(c− a) ≥ 0.

Determine when equality occurs.

10 Let p and q be integers. Show that there exists an interval I of length 1/q and a polynomialP with integral coefficients such that ∣∣∣∣P (x)− p

q

∣∣∣∣ <1q2

for all x ∈ I.

11 Let f : [0, 1] → R be continuous and satisfy:

bf(2x) = f(x), 0 ≤ x ≤ 1/2;f(x) = b + (1− b)f(2x− 1), 1/2 ≤ x ≤ 1,

where b = 1+c2+c , c > 0. Show that 0 < f(x)− x < c for every x, 0 < x < 1.

12 Find all functions f defined on the set of positive reals which take positive real values andsatisfy: f(xf(y)) = yf(x) for all x, y; and f(x) → 0 as x →∞.

13 Let E be the set of 19833 points of the space R3 all three of whose coordinates are integersbetween 0 and 1982 (including 0 and 1982). A coloring of E is a map from E to the set red,blue. How many colorings of E are there satisfying the following property: The number ofred vertices among the 8 vertices of any right-angled parallelepiped is a multiple of 4 ?

14 Is it possible to choose 1983 distinct positive integers, all less than or equal to 105, no threeof which are consecutive terms of an arithmetic progression?

15 Decide whether there exists a set M of positive integers satisfying the following conditions:

(i) For any natural numberm > 1 there are a, b ∈ M such that a + b = m.

(ii) If a, b, c, d ∈ M,a, b, c, d > 10 and a + b = c + d, then a = c or a = d.

16 Let F (n) be the set of polynomials P (x) = a0 + a1x + · · ·+ anxn, with a0, a1, ..., an ∈ R and0 ≤ a0 = an ≤ a1 = an−1 ≤ · · · ≤ a[n/2] = a[(n+1)/2]. Prove that if f ∈ F (m) and g ∈ F (n),then fg ∈ F (m + n).

17 Let P1, P2, . . . , Pn be distinct points of the plane, n ≥ 2. Prove that

max1≤i<j≤n

PiPj >

√3

2(n− 1) min

1≤i<j≤nPiPj

http://www.artofproblemsolving.com/This file was downloaded from the AoPS Math Olympiad Resources Page Page 2

Page 74: យុវសិស្ ឈឹ ទីុយ IMO Shortlist 1959-2009 · IMO Shortlist 1959 1 Prove that the fraction 21n+4 14n+3 ... Let α be the acute angel subtending, from A, that

IMO Shortlist 1983

18 Let a, b and c be positive integers, no two of which have a common divisor greater than 1.Show that 2abc − ab − bc − ca is the largest integer which cannot be expressed in the formxbc + yca + zab, where x, y, z are non-negative integers.

19 Let (Fn)n≥1 be the Fibonacci sequence F1 = F2 = 1, Fn+2 = Fn+1 +Fn(n ≥ 1), and P (x) thepolynomial of degree 990 satisfying

P (k) = Fk, for k = 992, ..., 1982.

Prove that P (1983) = F1983 − 1.

20 Find all solutions of the following system of n equations in n variables:

x1|x1| − (x1 − a)|x1 − a| = x2|x2|,x2|x2| − (x2 − a)|x2 − a| = x3|x3|,

...xn|xn| − (xn − a)|xn − a| = x1|x1|

where a is a given number.

21 Find the greatest integer less than or equal to∑21983

k=1 k1

1983−1.

22 Let n be a positive integer having at least two different prime factors. Show that there existsa permutation a1, a2, . . . , an of the integers 1, 2, . . . , n such that

n∑k=1

k · cos2πak

n= 0.

23 Let A be one of the two distinct points of intersection of two unequal coplanar circles C1 andC2 with centers O1 and O2 respectively. One of the common tangents to the circles touchesC1 at P1 and C2 at P2, while the other touches C1 at Q1 and C2 at Q2. Let M1 be themidpoint of P1Q1 and M2 the midpoint of P2Q2. Prove that ∠O1AO2 = ∠M1AM2.

24 Let dn be the last nonzero digit of the decimal representation of n!. Prove that dn is aperiodic;that is, there do not exist T and n0 such that for all n ≥ n0, dn+T = dn.

25 Prove that every partition of 3-dimensional space into three disjoint subsets has the followingproperty: One of these subsets contains all possible distances; i.e., for every a ∈ R+, thereare points M and N inside that subset such that distance between M and N is exactly a.

http://www.artofproblemsolving.com/This file was downloaded from the AoPS Math Olympiad Resources Page Page 3

Page 75: យុវសិស្ ឈឹ ទីុយ IMO Shortlist 1959-2009 · IMO Shortlist 1959 1 Prove that the fraction 21n+4 14n+3 ... Let α be the acute angel subtending, from A, that

IMO Shortlist 1984

1 Find all solutions of the following system of n equations in n variables:

x1|x1| − (x1 − a)|x1 − a| = x2|x2|,x2|x2| − (x2 − a)|x2 − a| = x3|x3|,

...xn|xn| − (xn − a)|xn − a| = x1|x1|

where a is a given number.

2 Prove:

(a) There are infinitely many triples of positive integers m,n, p such that 4mn−m−n = p2−1.

(b) There are no positive integers m,n, p such that 4mn−m− n = p2.

3 Find all positive integers n such that

n = d26 + d2

7 − 1,

where 1 = d1 < d2 < · · · < dk = n are all positive divisors of the number n.

4 Let d be the sum of the lengths of all the diagonals of a plane convex polygon with n vertices(where n > 3). Let p be its perimeter. Prove that:

n− 3 <2d

p<

[n

2

]·[n + 1

2

]− 2,

where [x] denotes the greatest integer not exceeding x.

5 Prove that 0 ≤ yz + zx + xy − 2xyz ≤ 727 , where x, y and z are non-negative real numbers

satisfying x + y + z = 1.

6 Let c be a positive integer. The sequence fn is defined as follows:

f1 = 1, f2 = c, fn+1 = 2fn − fn−1 + 2 (n ≥ 2).

Show that for each k ∈ N there exists r ∈ N such that fkfk+1 = fr.

7 (a) Decide whether the fields of the 8 × 8 chessboard can be numbered by the numbers1, 2, . . . , 64 in such a way that the sum of the four numbers in each of its parts of one of theforms

[img]http://www.artofproblemsolving.com/Forum/download/file.php?id=28446[/img]

is divisible by four.

(b) Solve the analogous problem for

[img]http://www.artofproblemsolving.com/Forum/download/file.php?id=28447[/img]

http://www.artofproblemsolving.com/This file was downloaded from the AoPS Math Olympiad Resources Page Page 1

Page 76: យុវសិស្ ឈឹ ទីុយ IMO Shortlist 1959-2009 · IMO Shortlist 1959 1 Prove that the fraction 21n+4 14n+3 ... Let α be the acute angel subtending, from A, that

IMO Shortlist 1984

8 Given points O and A in the plane. Every point in the plane is colored with one of a finitenumber of colors. Given a point X in the plane, the circle C(X) has center O and radiusOX + ∠AOX

OX , where ∠AOX is measured in radians in the range [0, 2π). Prove that we canfind a point X, not on OA, such that its color appears on the circumference of the circleC(X).

9 Let a, b, c be positive numbers with√

a +√

b +√

c =√

32 . Prove that the system of equations

√y − a +

√z − a = 1,

√z − b +

√x− b = 1,

√x− c +

√y − c = 1

has exactly one solution (x, y, z) in real numbers.

10 Prove that the product of five consecutive positive integers cannot be the square of an integer.

11 Let n be a positive integer and a1, a2, . . . , a2n mutually distinct integers. Find all integers xsatisfying

(x− a1) · (x− a2) · · · (x− a2n) = (−1)n(n!)2.

12 Find one pair of positive integers a, b such that ab(a+b) is not divisible by 7, but (a+b)7−a7−b7

is divisible by 77.

13 Prove that the volume of a tetrahedron inscribed in a right circular cylinder of volume 1 doesnot exceed 2

3π .

14 Let ABCD be a convex quadrilateral with the line CD being tangent to the circle on diameterAB. Prove that the line AB is tangent to the circle on diameter CD if and only if the linesBC and AD are parallel.

15 Angles of a given triangle ABC are all smaller than 120. Equilateral triangles AFB,BDCand CEA are constructed in the exterior of ABC.

(a) Prove that the lines AD,BE, and CF pass through one point S.

(b) Prove that SD + SE + SF = 2(SA + SB + SC).

16 Let a, b, c, d be odd integers such that 0 < a < b < c < d and ad = bc. Prove that if a+d = 2k

and b + c = 2m for some integers k and m, then a = 1.

17 In a permutation (x1, x2, . . . , xn) of the set 1, 2, . . . , n we call a pair (xi, xj) discordant if i < jand xi > xj . Let d(n, k) be the number of such permutations with exactly k discordant pairs.Find d(n, 2) and d(n, 3).

http://www.artofproblemsolving.com/This file was downloaded from the AoPS Math Olympiad Resources Page Page 2

Page 77: យុវសិស្ ឈឹ ទីុយ IMO Shortlist 1959-2009 · IMO Shortlist 1959 1 Prove that the fraction 21n+4 14n+3 ... Let α be the acute angel subtending, from A, that

IMO Shortlist 1984

18 Inside triangle ABC there are three circles k1, k2, k3 each of which is tangent to two sides ofthe triangle and to its incircle k. The radii of k1, k2, k3 are 1, 4, and 9. Determine the radiusof k.

19 The harmonic table is a triangular array:

112

12

13

16

13

14

112

112

14

Where an,1 = 1n and an,k+1 = an−1,k − an,k for 1 ≤ k ≤ n− 1. Find the harmonic mean of the

1985th row.

20 Determine all pairs (a, b) of positive real numbers with a 6= 1 such that

loga b < loga+1(b + 1).

http://www.artofproblemsolving.com/This file was downloaded from the AoPS Math Olympiad Resources Page Page 3

Page 78: យុវសិស្ ឈឹ ទីុយ IMO Shortlist 1959-2009 · IMO Shortlist 1959 1 Prove that the fraction 21n+4 14n+3 ... Let α be the acute angel subtending, from A, that

IMO Shortlist 1985

1 Given a set M of 1985 positive integers, none of which has a prime divisor larger than 26,prove that the set has four distinct elements whose geometric mean is an integer.

2 A polyhedron has 12 faces and is such that:

(i) all faces are isosceles triangles, (ii) all edges have length either x or y, (iii) at each vertexeither 3 or 6 edges meet, and (iv) all dihedral angles are equal.

Find the ratio x/y.

3 For any polynomial P (x) = a0 + a1x + . . . + akxk with integer coefficients, the number of

odd coefficients is denoted by o(P ). For i − 0, 1, 2, . . . let Qi(x) = (1 + x)i. Prove that ifi1, i2, . . . , in are integers satisfying 0 ≤ i1 < i2 < . . . < in, then:

o(Qi1 + Qi2 + . . . + Qin) ≥ o(Qi1).

4 Each of the numbers in the set N = 1, 2, 3, · · · , n− 1, where n ≥ 3, is colored with one oftwo colors, say red or black, so that:

(i) i and n− i always receive the same color, and

(ii) for some j ∈ N , relatively prime to n, i and |j−i| receive the same color for all i ∈ N, i 6= j.

Prove that all numbers in N must receive the same color.

5 Let D be the interior of the circle C and let A ∈ C. Show that the function f : D → R, f(M) =|MA||MM ′| where M ′ = AM ∩C, is strictly convex; i.e., f(P ) < f(M1)+f(M2)

2 ,∀M1,M2 ∈ D,M1 6=M2 where P is the midpoint of the segment M1M2.

6 Let xn = 2

√2 + 3

√3 + · · ·+ n

√n. Prove that

xn+1 − xn <1n!

n = 2, 3, · · ·

7 The positive integers x1, · · · , xn, n ≥ 3, satisfy x1 < x2 < · · · < xn < 2x1. Set P =x1x2 · · ·xn. Prove that if p is a prime number, k a positive integer, and P is divisible by pk,then P

pk ≥ n!.

8 Let A be a set of n points in space. From the family of all segments with endpoints in A,q segments have been selected and colored yellow. Suppose that all yellow segments are ofdifferent length. Prove that there exists a polygonal line composed of m yellow segments,where m ≥ 2q

n , arranged in order of increasing length.

9 Determine the radius of a sphere S that passes through the centroids of each face of a giventetrahedron T inscribed in a unit sphere with center O. Also, determine the distance from Oto the center of S as a function of the edges of T.

http://www.artofproblemsolving.com/This file was downloaded from the AoPS Math Olympiad Resources Page Page 1

Page 79: យុវសិស្ ឈឹ ទីុយ IMO Shortlist 1959-2009 · IMO Shortlist 1959 1 Prove that the fraction 21n+4 14n+3 ... Let α be the acute angel subtending, from A, that

IMO Shortlist 1985

10 Prove that for every point M on the surface of a regular tetrahedron there exists a point M ′

such that there are at least three different curves on the surface joining M to M ′ with thesmallest possible length among all curves on the surface joining M to M ′.

11 Find a method by which one can compute the coefficients of P (x) = x6 +a1x5 + · · ·+a6 from

the roots of P (x) = 0 by performing not more than 15 additions and 15 multiplications.

12 A sequence of polynomials Pm(x, y, z),m = 0, 1, 2, · · · , in x, y, and z is defined by P0(x, y, z) =1 and by

Pm(x, y, z) = (x + z)(y + z)Pm−1(x, y, z + 1)− z2Pm−1(x, y, z)

for m > 0. Prove that each Pm(x, y, z) is symmetric, in other words, is unaltered by anypermutation of x, y, z.

13 Let m boxes be given, with some balls in each box. Let n < m be a given integer. Thefollowing operation is performed: choose n of the boxes and put 1 ball in each of them.Prove:

(a) If m and n are relatively prime, then it is possible, by performing the operation a finitenumber of times, to arrive at the situation that all the boxes contain an equal number ofballs.

(b) If m and n are not relatively prime, there exist initial distributions of balls in the boxessuch that an equal distribution is not possible to achieve.

14 A set of 1985 points is distributed around the circumference of a circle and each of the pointsis marked with 1 or −1. A point is called good if the partial sums that can be formed bystarting at that point and proceeding around the circle for any distance in either directionare all strictly positive. Show that if the number of points marked with −1 is less than 662,there must be at least one good point.

15 Let K and K ′ be two squares in the same plane, their sides of equal length. Is it possible todecompose K into a finite number of triangles T1, T2, · · · , Tp with mutually disjoint interiorsand find translations t1, t2, · · · , tp such that

K ′ =p⋃

i=1

ti(Ti) ?

16 If possible, construct an equilateral triangle whose three vertices are on three given circles.

17 The sequence f1,f2, · · · , fn, · · · of functions is defined for x > 0 recursively by

f1(x) = x, fn+1(x) = fn(x)(

fn(x) +1n

)

http://www.artofproblemsolving.com/This file was downloaded from the AoPS Math Olympiad Resources Page Page 2

Page 80: យុវសិស្ ឈឹ ទីុយ IMO Shortlist 1959-2009 · IMO Shortlist 1959 1 Prove that the fraction 21n+4 14n+3 ... Let α be the acute angel subtending, from A, that

IMO Shortlist 1985

Prove that there exists one and only one positive number a such that 0 < fn(a) < fn+1(a) < 1for all integers n ≥ 1.

18 Let x1, x2, · · · , xn be positive numbers. Prove that

x21

x21 + x2x3

+x2

2

x22 + x3x4

+ · · ·+x2

n−1

x2n−1 + xnx1

+x2

n

x2n + x1x2

≤ n− 1

19 For which integers n ≥ 3 does there exist a regular n-gon in the plane such that all its verticeshave integer coordinates in a rectangular coordinate system?

20 A circle whose center is on the side ED of the cyclic quadrilateral BCDE touches the otherthree sides. Prove that EB + CD = ED.

21 The tangents at B and C to the circumcircle of the acute-angled triangle ABC meet at X.Let M be the midpoint of BC. Prove that

(a) ∠BAM = ∠CAX, and

(b) AMAX = cos ∠BAC.

22 A circle with center O passes through the vertices A and C of the triangle ABC and intersectsthe segments AB and BC again at distinct points K and N respectively. Let M be the pointof intersection of the circumcircles of triangles ABC and KBN (apart from B). Prove that∠OMB = 90.

http://www.artofproblemsolving.com/This file was downloaded from the AoPS Math Olympiad Resources Page Page 3

Page 81: យុវសិស្ ឈឹ ទីុយ IMO Shortlist 1959-2009 · IMO Shortlist 1959 1 Prove that the fraction 21n+4 14n+3 ... Let α be the acute angel subtending, from A, that

IMO Shortlist 1986

1 Let A,B be adjacent vertices of a regular n-gon (n ≥ 5) with center O. A triangle XY Z,which is congruent to and initially coincides with OAB, moves in the plane in such a waythat Y and Z each trace out the whole boundary of the polygon, with X remaining insidethe polygon. Find the locus of X.

2 Let f(x) = xn where n is a fixed positive integer and x = 1, 2, · · · . Is the decimal expansiona = 0.f(1)f(2)f(3)... rational for any value of n ?

The decimal expansion of a is defined as follows: If f(x) = d1(x)d2(x) · · · dr(x)(x) is thedecimal expansion of f(x), then a = 0.1d1(2)d2(2) · · · dr(2)(2)d1(3)...dr(3)(3)d1(4) · · · .

3 Let A,B, and C be three points on the edge of a circular chord such that B is due west of Cand ABC is an equilateral triangle whose side is 86 meters long. A boy swam from A directlytoward B. After covering a distance of x meters, he turned and swam westward, reaching theshore after covering a distance of y meters. If x and y are both positive integers, determiney.

4 Provided the equation xyz = pn(x + y + z) where p ≥ 3 is a prime and n ∈ N. Prove thatthe equation has at least 3n + 3 different solutions (x, y, z) with natural numbers x, y, z andx < y < z. Prove the same for p > 3 being an odd integer.

5 Let d be any positive integer not equal to 2, 5 or 13. Show that one can find distinct a, b inthe set 2, 5, 13, d such that ab− 1 is not a perfect square.

6 Find four positive integers each not exceeding 70000 and each having more than 100 divisors.

7 Let real numbers x1, x2, · · · , xn satisfy 0 < x1 < x2 < · · · < xn < 1 and set x0 = 0, xn+1 = 1.Suppose that these numbers satisfy the following system of equations:

n+1∑j=0,j 6=i

1xi − xj

= 0 where i = 1, 2, ..., n.

Prove that xn+1−i = 1− xi for i = 1, 2, ..., n.

8 From a collection of n persons q distinct two-member teams are selected and ranked 1, · · · , q(no ties). Let m be the least integer larger than or equal to 2q/n. Show that there are mdistinct teams that may be listed so that : (i) each pair of consecutive teams on the list haveone member in common and (ii) the chain of teams on the list are in rank order.

Alternative formulation. Given a graph with n vertices and q edges numbered 1, · · · , q, showthat there exists a chain of m edges, m ≥ 2q

n , each two consecutive edges having a commonvertex, arranged monotonically with respect to the numbering.

http://www.artofproblemsolving.com/This file was downloaded from the AoPS Math Olympiad Resources Page Page 1

Page 82: យុវសិស្ ឈឹ ទីុយ IMO Shortlist 1959-2009 · IMO Shortlist 1959 1 Prove that the fraction 21n+4 14n+3 ... Let α be the acute angel subtending, from A, that

IMO Shortlist 1986

9 Given a finite set of points in the plane, each with integer coordinates, is it always possible tocolor the points red or white so that for any straight line L parallel to one of the coordinateaxes the difference (in absolute value) between the numbers of white and red points on L isnot greater than 1?

10 Three persons A,B, C, are playing the following game:

A k-element subset of the set 1, ..., 1986 is randomly chosen, with an equal probability ofeach choice, where k is a fixed positive integer less than or equal to 1986. The winner is A,Bor C, respectively, if the sum of the chosen numbers leaves a remainder of 0, 1, or 2 whendivided by 3.

For what values of k is this game a fair one? (A game is fair if the three outcomes are equallyprobable.)

11 Let f(n) be the least number of distinct points in the plane such that for each k = 1, 2, · · · , nthere exists a straight line containing exactly k of these points. Find an explicit expressionfor f(n).

Simplified version.

Show that f(n) =[

n+12

] [n+2

2

]. Where [x] denoting the greatest integer not exceeding x.

12 To each vertex of a regular pentagon an integer is assigned, so that the sum of all fivenumbers is positive. If three consecutive vertices are assigned the numbers x, y, z respectively,and y < 0, then the following operation is allowed: x, y, z are replaced by x + y,−y, z + yrespectively. Such an operation is performed repeatedly as long as at least one of the fivenumbers is negative. Determine whether this procedure necessarily comes to an end after afinite number of steps.

13 A particle moves from (0, 0) to (n, n) directed by a fair coin. For each head it moves one stepeast and for each tail it moves one step north. At (n, y), y < n, it stays there if a head comesup and at (x, n), x < n, it stays there if a tail comes up. Letk be a fixed positive integer.Find the probability that the particle needs exactly 2n + k tosses to reach (n, n).

14 The circle inscribed in a triangle ABC touches the sides BC, CA, AB in D,E, F , respectively,and X, Y, Z are the midpoints of EF,FD,DE, respectively. Prove that the centers of theinscribed circle and of the circles around XY Z and ABC are collinear.

15 Let ABCD be a convex quadrilateral whose vertices do not lie on a circle. Let A′B′C ′D′

be a quadrangle such that A′, B′, C ′, D′ are the centers of the circumcircles of trianglesBCD,ACD,ABD, and ABC. We write T (ABCD) = A′B′C ′D′. Let us define A′′B′′C ′′D′′ =T (A′B′C ′D′) = T (T (ABCD)).

(a) Prove that ABCD and A′′B′′C ′′D′′ are similar.

(b) The ratio of similitude depends on the size of the angles of ABCD. Determine this ratio.

http://www.artofproblemsolving.com/This file was downloaded from the AoPS Math Olympiad Resources Page Page 2

Page 83: យុវសិស្ ឈឹ ទីុយ IMO Shortlist 1959-2009 · IMO Shortlist 1959 1 Prove that the fraction 21n+4 14n+3 ... Let α be the acute angel subtending, from A, that

IMO Shortlist 1986

16 Let A,B be adjacent vertices of a regular n-gon (n ≥ 5) with center O. A triangle XY Z,which is congruent to and initially coincides with OAB, moves in the plane in such a waythat Y and Z each trace out the whole boundary of the polygon, with X remaining insidethe polygon. Find the locus of X.

17 Given a point P0 in the plane of the triangle A1A2A3. Define As = As−3 for all s ≥ 4.Construct a set of points P1, P2, P3, . . . such that Pk+1 is the image of Pk under a rotationcenter Ak+1 through an angle 120o clockwise for k = 0, 1, 2, . . .. Prove that if P1986 = P0,then the triangle A1A2A3 is equilateral.

18 Let AX, BY,CZ be three cevians concurrent at an interior point D of a triangle ABC. Provethat if two of the quadrangles DY AZ, DZBX, DXCY are circumscribable, so is the third.

19 A tetrahedron ABCD is given such that AD = BC = a;AC = BD = b;AB · CD = c2. Letf(P ) = AP + BP + CP + DP , where P is an arbitrary point in space. Compute the leastvalue of f(P ).

20 Prove that the sum of the face angles at each vertex of a tetrahedron is a straight angle ifand only if the faces are congruent triangles.

21 Let ABCD be a tetrahedron having each sum of opposite sides equal to 1. Prove that

rA + rB + rC + rD ≤√

33

where rA, rB, rC , rD are the inradii of the faces, equality holding only if ABCD is regular.

http://www.artofproblemsolving.com/This file was downloaded from the AoPS Math Olympiad Resources Page Page 3

Page 84: យុវសិស្ ឈឹ ទីុយ IMO Shortlist 1959-2009 · IMO Shortlist 1959 1 Prove that the fraction 21n+4 14n+3 ... Let α be the acute angel subtending, from A, that

IMO Shortlist 1987

1 Let f be a function that satisfies the following conditions:

(i) If x > y and f(y) − y ≥ v ≥ f(x) − x, then f(z) = v + z, for some number z betweenx and y. (ii) The equation f(x) = 0 has at least one solution, and among the solutions ofthis equation, there is one that is not smaller than all the other solutions; (iii) f(0) = 1. (iv)f(1987) ≤ 1988. (v) f(x)f(y) = f(xf(y) + yf(x)− xy).

Find f(1987).

Proposed by Australia.

2 At a party attended by n married couples, each person talks to everyone else at the partyexcept his or her spouse. The conversations involve sets of persons or cliques C1, C2, · · · , Ck

with the following property: no couple are members of the same clique, but for every otherpair of persons there is exactly one clique to which both members belong. Prove that if n ≥ 4,then k ≥ 2n.

Proposed by USA.

3 Does there exist a second-degree polynomial p(x, y) in two variables such that every non-negative integer n equals p(k, m) for one and only one ordered pair (k, m) of non-negativeintegers?

Proposed by Finland.

4 Let ABCDEFGH be a parallelepiped with AE ‖ BF ‖ CG ‖ DH. Prove the inequality

AF + AH + AC ≤ AB + AD + AE + AG.

In what cases does equality hold?

Proposed by France.

5 Find, with proof, the point P in the interior of an acute-angled triangle ABC for whichBL2 + CM2 + AN2 is a minimum, where L,M,N are the feet of the perpendiculars from Pto BC, CA, AB respectively.

Proposed by United Kingdom.

6 Show that if a, b, c are the lengths of the sides of a triangle and if 2S = a + b + c, then

an

b + c+

bn

c + a+

cn

a + b≥

(23

)n−2

Sn−1 ∀n ∈ N

Proposed by Greece.

http://www.artofproblemsolving.com/This file was downloaded from the AoPS Math Olympiad Resources Page Page 1

Page 85: យុវសិស្ ឈឹ ទីុយ IMO Shortlist 1959-2009 · IMO Shortlist 1959 1 Prove that the fraction 21n+4 14n+3 ... Let α be the acute angel subtending, from A, that

IMO Shortlist 1987

7 Given five real numbers u0, u1, u2, u3, u4, prove that it is always possible to find five realnumbers v0, v1, v2, v3, v4 that satisfy the following conditions:

(i) ui − vi ∈ N, 0 ≤ i ≤ 4

(ii)∑

0≤i<j≤4(vi − vj)2 < 4.

Proposed by Netherlands.

8 (a) Let gcd(m, k) = 1. Prove that there exist integers a1, a2, ..., am and b1, b2, ..., bk such thateach product aibj (i = 1, 2, · · · ,m; j = 1, 2, · · · , k) gives a different residue when divided bymk.

(b) Let gcd(m, k) > 1. Prove that for any integers a1, a2, ..., am and b1, b2, ..., bk there mustbe two products aibj and asbt ((i, j) 6= (s, t)) that give the same residue when divided by mk.

Proposed by Hungary.

9 Does there exist a set M in usual Euclidean space such that for every plane λ the intersectionM ∩ λ is finite and nonempty ?

Proposed by Hungary.

[hide=”Remark”]I’m not sure I’m posting this in a right Forum.

10 Let S1 and S2 be two spheres with distinct radii that touch externally. The spheres lie insidea cone C, and each sphere touches the cone in a full circle. Inside the cone there are nadditional solid spheres arranged in a ring in such a way that each solid sphere touches thecone C, both of the spheres S1 and S2 externally, as well as the two neighboring solid spheres.What are the possible values of n?

Proposed by Iceland.

11 Find the number of partitions of the set 1, 2, · · · , n into three subsets A1, A2, A3, some ofwhich may be empty, such that the following conditions are satisfied:

(i) After the elements of every subset have been put in ascending order, every two consecutiveelements of any subset have different parity.

(ii) If A1, A2, A3 are all nonempty, then in exactly one of them the minimal number is even .

Proposed by Poland.

12 Given a nonequilateral triangle ABC, the vertices listed counterclockwise, find the locus ofthe centroids of the equilateral triangles A′B′C ′ (the vertices listed counterclockwise) forwhich the triples of points A,B′, C ′;A′, B, C ′; and A′, B′, C are collinear.

Proposed by Poland.

http://www.artofproblemsolving.com/This file was downloaded from the AoPS Math Olympiad Resources Page Page 2

Page 86: យុវសិស្ ឈឹ ទីុយ IMO Shortlist 1959-2009 · IMO Shortlist 1959 1 Prove that the fraction 21n+4 14n+3 ... Let α be the acute angel subtending, from A, that

IMO Shortlist 1987

13 Is it possible to put 1987 points in the Euclidean plane such that the distance between eachpair of points is irrational and each three points determine a non-degenerate triangle withrational area? (IMO Problem 5)

Proposed by Germany, DR

14 How many words with n digits can be formed from the alphabet 0, 1, 2, 3, 4, if neighboringdigits must differ by exactly one?

Proposed by Germany, FR.

15 Let x1, x2, . . . , xn be real numbers satisfying x21 + x2

2 + . . . + x2n = 1. Prove that for every

integer k ≥ 2 there are integers a1, a2, . . . , an, not all zero, such that |ai| ≤ k− 1 for all i, and|a1x1 + a2x2 + . . . + anxn| ≤ (k−1)

√n

kn−1 . (IMO Problem 3)

Proposed by Germany, FR

16 Let pn(k) be the number of permutations of the set 1, 2, 3, . . . , n which have exactly k fixedpoints. Prove that

∑nk=0 kpn(k) = n!.(IMO Problem 1)

Original formulation

Let S be a set of n elements. We denote the number of all permutations of S that haveexactly k fixed points by pn(k). Prove:

(a)∑n

k=0 kpn(k) = n! ;

(b)∑n

k=0(k − 1)2pn(k) = n!

Proposed by Germany, FR

17 Prove that there exists a four-coloring of the set M = 1, 2, · · · , 1987 such that any arithmeticprogression with 10 terms in the set M is not monochromatic.

Alternative formulation

Let M = 1, 2, · · · , 1987. Prove that there is a function f : M → 1, 2, 3, 4 that is notconstant on every set of 10 terms from M that form an arithmetic progression.

Proposed by Romania

18 For any integer r ≥ 1, determine the smallest integer h(r) ≥ 1 such that for any partitionof the set 1, 2, · · · , h(r) into r classes, there are integers a ≥ 0 ; 1 ≤ x ≤ y, such thata + x, a + y, a + x + y belong to the same class.

Proposed by Romania

19 Let α, β, γ be positive real numbers such that α+β +γ < π, α+β > γ,β +γ > α, γ +α > β.Prove that with the segments of lengths sinα, sinβ, sin γ we can construct a triangle and that

http://www.artofproblemsolving.com/This file was downloaded from the AoPS Math Olympiad Resources Page Page 3

Page 87: យុវសិស្ ឈឹ ទីុយ IMO Shortlist 1959-2009 · IMO Shortlist 1959 1 Prove that the fraction 21n+4 14n+3 ... Let α be the acute angel subtending, from A, that

IMO Shortlist 1987

its area is not greater than

A =18

(sin 2α + sin 2β + sin 2γ) .

Proposed by Soviet Union

20 Let n ≥ 2 be an integer. Prove that if k2 + k + n is prime for all integers k such that0 ≤ k ≤

√n3 , then k2 +k+n is prime for all integers k such that 0 ≤ k ≤ n−2.(IMO Problem

6)

Original Formulation

Let f(x) = x2 + x + p, p ∈ N. Prove that if the numbers f(0), f(1), · · · , f(√p3) are primes, then all the numbers f(0), f(1), · · · , f(p− 2) are primes.

Proposed by Soviet Union.

21 In an acute-angled triangle ABC the interior bisector of angle A meets BC at L and meets thecircumcircle of ABC again at N . From L perpendiculars are drawn to AB and AC, with feetK and M respectively. Prove that the quadrilateral AKNM and the triangle ABC have equalareas.(IMO Problem 2)

Proposed by Soviet Union.

22 Does there exist a function f : N → N, such that f(f(n)) = n + 1987 for every natural number n?(IMO Problem 4)

Proposed by Vietnam.

23 Prove that for every natural number k (k ≥ 2) there exists an irrational number r such that forevery natural number m,

[rm] ≡ −1 (mod k).

Remark. An easier variant: Find r as a root of a polynomial of second degree with integer coeffi-cients.

Proposed by Yugoslavia.

http://www.artofproblemsolving.com/This file was downloaded from the AoPS Math Olympiad Resources Page Page 4

Page 88: យុវសិស្ ឈឹ ទីុយ IMO Shortlist 1959-2009 · IMO Shortlist 1959 1 Prove that the fraction 21n+4 14n+3 ... Let α be the acute angel subtending, from A, that

IMO Shortlist 1988

1 An integer sequence is defined by

an = 2 · an−1 + an−2, (n > 1), a0 = 0, a1 = 1.

Prove that 2k divides an if and only if 2k divides n.

2 Let n be a positive integer. Find the number of odd coefficients of the polynomial

un(x) = (x2 + x + 1)n.

3 The triangle ABC is inscribed in a circle. The interior bisectors of the angles A,B and Cmeet the circle again at A′, B′ and C ′ respectively. Prove that the area of triangle A′B′C ′ isgreater than or equal to the area of triangle ABC.

4 An n × n, n ≥ 2 chessboard is numbered by the numbers 1, 2, . . . , n2 (and every numberoccurs). Prove that there exist two neighbouring (with common edge) squares such that theirnumbers differ by at least n.

5 Let n be an even positive integer. Let A1, A2, . . . , An+1 be sets having n elements each suchthat any two of them have exactly one element in common while every element of their unionbelongs to at least two of the given sets. For which n can one assign to every element of theunion one of the numbers 0 and 1 in such a manner that each of the sets has exactly n

2 zeros?

6 In a given tedrahedron ABCD let K and L be the centres of edges AB and CD respectively.Prove that every plane that contains the line KL divides the tedrahedron into two parts ofequal volume.

7 Let a be the greatest positive root of the equation x3 − 3 · x2 + 1 = 0. Show that[a1788

]and[

a1988]

are both divisible by 17. Here [x] denotes the integer part of x.

8 Let u1, u2, . . . , um be m vectors in the plane, each of length ≤ 1, with zero sum. Showthat one can arrange u1, u2, . . . , um as a sequence v1, v2, . . . , vm such that each partial sumv1, v1 + v2, v1 + v2 + v3, . . . , v1, v2, . . . , vm has length less than or equal to

√5.

9 Let a and b be two positive integers such that a · b + 1 divides a2 + b2. Show that a2+b2

a·b+1 is aperfect square.

10 Let N = 1, 2 . . . , n, n ≥ 2. A collection F = A1, . . . , At of subsets Ai ⊆ N, i = 1, . . . , t, issaid to be separating, if for every pair x, y ⊆ N, there is a set Ai ∈ F so that Ai ∩ x, ycontains just one element. F is said to be covering, if every element of N is contained in atleast one set Ai ∈ F. What is the smallest value f(n) of t, so there is a set F = A1, . . . , Atwhich is simultaneously separating and covering?

http://www.artofproblemsolving.com/This file was downloaded from the AoPS Math Olympiad Resources Page Page 1

Page 89: យុវសិស្ ឈឹ ទីុយ IMO Shortlist 1959-2009 · IMO Shortlist 1959 1 Prove that the fraction 21n+4 14n+3 ... Let α be the acute angel subtending, from A, that

IMO Shortlist 1988

11 The lock of a safe consists of 3 wheels, each of which may be set in 8 different ways positions.Due to a defect in the safe mechanism the door will open if any two of the three wheels arein the correct position. What is the smallest number of combinations which must be triedif one is to guarantee being able to open the safe (assuming the ”right combination” is notknown)?

12 In a triangle ABC, choose any points K ∈ BC, L ∈ AC,M ∈ AB,N ∈ LM, R ∈ MK and F ∈KL. If E1, E2, E3, E4, E5, E6 and E denote the areas of the triangles AMR, CKR,BKF,ALF, BNM,CLNand ABC respectively, show that

E ≥ 8 · 6√

E1E2E3E4E5E6.

13 In a right-angled triangle ABC let AD be the altitude drawn to the hypotenuse and let thestraight line joining the incentres of the triangles ABD, ACD intersect the sides AB,ACat the points K, L respectively. If E and E1 dnote the areas of triangles ABC and AKLrespectively, show that

E

E1≥ 2.

14 For what values of n does there exist an n × n array of entries -1, 0 or 1 such that the 2 · nsums obtained by summing the elements of the rows and the columns are all different?

15 Let ABC be an acute-angled triangle. The lines LA, LB and LC are constructed through thevertices A, B and C respectively according the following prescription: Let H be the foot ofthe altitude drawn from the vertex A to the side BC; let SA be the circle with diameter AH;let SA meet the sides AB and AC at M and N respectively, where M and N are distinctfrom A; then let LA be the line through A perpendicular to MN . The lines LB and LC areconstructed similarly. Prove that the lines LA, LB and LC are concurrent.

16 Show that the solution set of the inequality

70∑k=1

k

x− k≥ 5

4

is a union of disjoint intervals, the sum of whose length is 1988.

17 In the convex pentagon ABCDE, the sides BC, CD,DE are equal. Moreover each diagonalof the pentagon is parallel to a side (AC is parallel to DE, BD is parallel to AE etc.). Provethat ABCDE is a regular pentagon.

18 Consider 2 concentric circle radii R and r (R > r) with centre O. Fix P on the small circleand consider the variable chord PA of the small circle. Points B and C lie on the large circle;B,P,C are collinear and BC is perpendicular to AP.

http://www.artofproblemsolving.com/This file was downloaded from the AoPS Math Olympiad Resources Page Page 2

Page 90: យុវសិស្ ឈឹ ទីុយ IMO Shortlist 1959-2009 · IMO Shortlist 1959 1 Prove that the fraction 21n+4 14n+3 ... Let α be the acute angel subtending, from A, that

IMO Shortlist 1988

i.) For which values of ∠OPA is the sum BC2 + CA2 + AB2 extremal?

ii.) What are the possible positions of the midpoints U of BA and V of AC as ∠OPA varies?

19 Let f(n) be a function defined on the set of all positive integers and having its values inthe same set. Suppose that f(f(n) + f(m)) = m + n for all positive integers n, m. Find thepossible value for f(1988).

20 Find the least natural number n such that, if the set 1, 2, . . . , n is arbitrarily divided intotwo non-intersecting subsets, then one of the subsets contains 3 distinct numbers such thatthe product of two of them equals the third.

21 Forty-nine students solve a set of 3 problems. The score for each problem is a whole number ofpoints from 0 to 7. Prove that there exist two students A and B such that, for each problem,A will score at least as many points as B.

22 Let p be the product of two consecutive integers greater than 2. Show that there are nointegers x1, x2, . . . , xp satisfying the equation

p∑i=1

x2i −

44 · p + 1

(p∑

i=1

xi

)2

= 1

OR

Show that there are only two values of p for which there are integers x1, x2, . . . , xp satisfying

p∑i=1

x2i −

44 · p + 1

(p∑

i=1

xi

)2

= 1

23 Let Q be the centre of the inscribed circle of a triangle ABC. Prove that for any point P,

a(PA)2 + b(PB)2 + c(PC)2 = a(QA)2 + b(QB)2 + c(QC)2 + (a + b + c)(QP )2,

where a = BC, b = CA and c = AB.

24 Let ak∞1 be a sequence of non-negative real numbers such that:

ak − 2 · ak+1 + ak+2 ≥ 0

and∑k

j=1 aj ≤ 1 for all k = 1, 2, . . .. Prove that:

0 ≤ (ak − ak+1) <2k2

for all k = 1, 2, . . ..

http://www.artofproblemsolving.com/This file was downloaded from the AoPS Math Olympiad Resources Page Page 3

Page 91: យុវសិស្ ឈឹ ទីុយ IMO Shortlist 1959-2009 · IMO Shortlist 1959 1 Prove that the fraction 21n+4 14n+3 ... Let α be the acute angel subtending, from A, that

IMO Shortlist 1988

25 A positive integer is called a double number if its decimal representation consists of a blockof digits, not commencing with 0, followed immediately by an identical block. So, for instance,360360 is a double number, but 36036 is not. Show that there are infinitely many doublenumbers which are perfect squares.

26 A function f defined on the positive integers (and taking positive integers values) is given by:

f(1) = 1, f(3) = 3f(2 · n) = f(n)

f(4 · n + 1) = 2 · f(2 · n + 1)− f(n)f(4 · n + 3) = 3 · f(2 · n + 1)− 2 · f(n),for all positive integers n. Determine with proof the number of positive integers ≤ 1988 forwhich f(n) = n.

27 Let ABC be an acute-angled triangle. Let L be any line in the plane of the triangle ABC.Denote by u, v, w the lengths of the perpendiculars to L from A, B, C respectively. Provethe inequality u2 · tanA + v2 · tanB + w2 · tanC ≥ 2 · S, where S is the area of the triangleABC. Determine the lines L for which equality holds.

28 The sequence an of integers is defined by

a1 = 2, a2 = 7

and

−12

< an+1 −a2

n

an−1≤ , n ≥ 2.

Prove that an is odd for all n > 1.

29 A number of signal lights are equally spaced along a one-way railroad track, labeled in oder1, 2, . . . , N, N ≥ 2. As a safety rule, a train is not allowed to pass a signal if any other trainis in motion on the length of track between it and the following signal. However, there isno limit to the number of trains that can be parked motionless at a signal, one behind theother. (Assume the trains have zero length.) A series of K freight trains must be driven fromSignal 1 to Signal N. Each train travels at a distinct but constant spped at all times when itis not blocked by the safety rule. Show that, regardless of the order in which the trains arearranged, the same time will elapse between the first train’s departure from Signal 1 and thelast train’s arrival at Signal N.

30 A point M is chosen on the side AC of the triangle ABC in such a way that the radii of thecircles inscribed in the triangles ABM and BMC are equal. Prove that

BM2 = X cot(

B

2

)where X is the area of triangle ABC.

http://www.artofproblemsolving.com/This file was downloaded from the AoPS Math Olympiad Resources Page Page 4

Page 92: យុវសិស្ ឈឹ ទីុយ IMO Shortlist 1959-2009 · IMO Shortlist 1959 1 Prove that the fraction 21n+4 14n+3 ... Let α be the acute angel subtending, from A, that

IMO Shortlist 1988

31 Around a circular table an even number of persons have a discussion. After a break they sitagain around the circular table in a different order. Prove that there are at least two peoplesuch that the number of participants sitting between them before and after a break is thesame.

http://www.artofproblemsolving.com/This file was downloaded from the AoPS Math Olympiad Resources Page Page 5

Page 93: យុវសិស្ ឈឹ ទីុយ IMO Shortlist 1959-2009 · IMO Shortlist 1959 1 Prove that the fraction 21n+4 14n+3 ... Let α be the acute angel subtending, from A, that

IMO Shortlist 1989

1 ABC is a triangle, the bisector of angle A meets the circumcircle of triangle ABC in A1, pointsB1 and C1 are defined similarly. Let AA1 meet the lines that bisect the two external anglesat B and C in A0. Define B0 and C0 similarly. Prove that the area of triangle A0B0C0 = 2·area of hexagon AC1BA1CB1 ≥ 4· area of triangle ABC.

2 Ali Barber, the carpet merchant, has a rectangular piece of carpet whose dimensions are un-known. Unfortunately, his tape measure is broken and he has no other measuring instruments.However, he finds that if he lays it flat on the floor of either of his storerooms, then eachcorner of the carpet touches a different wall of that room. If the two rooms have dimensionsof 38 feet by 55 feet and 50 feet by 55 feet, what are the carpet dimensions?

3 Ali Barber, the carpet merchant, has a rectangular piece of carpet whose dimensions areunknown. Unfortunately, his tape measure is broken and he has no other measuring instru-ments. However, he finds that if he lays it flat on the floor of either of his storerooms, theneach corner of the carpet touches a different wall of that room. He knows that the sides ofthe carpet are integral numbers of feet and that his two storerooms have the same (unknown)length, but widths of 38 feet and 50 feet respectively. What are the carpet dimensions?

4 Prove that ∀n > 1, n ∈ N the equation

n∑k=1

xk

k!+ 1 = 0

has no rational roots.

5 Find the roots ri ∈ R of the polynomial

p(x) = xn + n · xn−1 + a2 · xn−2 + . . . + an

satisfying16∑

k=1

r16k = n.

6 For a triangle ABC, let k be its circumcircle with radius r. The bisectors of the inner anglesA,B, and C of the triangle intersect respectively the circle k again at points A′, B′, and C ′.Prove the inequality

16Q3 ≥ 27r4P,

where Q and P are the areas of the triangles A′B′C ′ and ABC respectively.

http://www.artofproblemsolving.com/This file was downloaded from the AoPS Math Olympiad Resources Page Page 1

Page 94: យុវសិស្ ឈឹ ទីុយ IMO Shortlist 1959-2009 · IMO Shortlist 1959 1 Prove that the fraction 21n+4 14n+3 ... Let α be the acute angel subtending, from A, that

IMO Shortlist 1989

7 Show that any two points lying inside a regular n−gon E can be joined by two circular arcslying inside E and meeting at an angle of at least

(1− 2

n

)· π.

8 Let R be a rectangle that is the union of a finite number of rectangles Ri, 1 ≤ i ≤ n, satisfyingthe following conditions:

(i) The sides of every rectangle Ri are parallel to the sides of R. (ii) The interiors of any twodifferent rectangles Ri are disjoint. (iii) Each rectangle Ri has at least one side of integrallength.

Prove that R has at least one side of integral length.

Variant: Same problem but with rectangular parallelepipeds having at least one integral side.

9 ∀n > 0, n ∈ Z, there exists uniquely determined integers an, bn, cn ∈ Z such(1 + 4 · 3

√2− 4 · 3

√4)n

= an + bn · 3√

2 + cn · 3√

4.

Prove that cn = 0 implies n = 0.

10 Let g : C → C, ω ∈ C, a ∈ C, ω3 = 1, and ω 6= 1. Show that there is one and only onefunction f : C → C such that

f(z) + f(ωz + a) = g(z), z ∈ C

11 Define sequence (an) by∑

d|n ad = 2n. Show that n|an.

12 There are n cars waiting at distinct points of a circular race track. At the starting signal eachcar starts. Each car may choose arbitrarily which of the two possible directions to go. Eachcar has the same constant speed. Whenever two cars meet they both change direction (butnot speed). Show that at some time each car is back at its starting point.

13 Let ABCD be a convex quadrilateral such that the sides AB,AD,BC satisfy AB = AD+BC.There exists a point P inside the quadrilateral at a distance h from the line CD such thatAP = h + AD and BP = h + BC. Show that:

1√h≥ 1√

AD+

1√BC

14 A bicentric quadrilateral is one that is both inscribable in and circumscribable about a circle,i.e. both the incircle and circumcircle exists. Show that for such a quadrilateral, the centersof the two associated circles are collinear with the point of intersection of the diagonals.

http://www.artofproblemsolving.com/This file was downloaded from the AoPS Math Olympiad Resources Page Page 2

Page 95: យុវសិស្ ឈឹ ទីុយ IMO Shortlist 1959-2009 · IMO Shortlist 1959 1 Prove that the fraction 21n+4 14n+3 ... Let α be the acute angel subtending, from A, that

IMO Shortlist 1989

15 Let a, b, c, d, m, n ∈ Z+ such that

a2 + b2 + c2 + d2 = 1989,

a + b + c + d = m2,

and the largest of a, b, c, d is n2. Determine, with proof, the values of m and n.

16 The set a0, a1, . . . , an of real numbers satisfies the following conditions:

(i) a0 = an = 0, (ii) for 1 ≤ k ≤ n− 1,

ak = c +n−1∑i=k

ai−k · (ai + ai+1)

Prove that c ≤ 14n .

17 Given seven points in the plane, some of them are connected by segments such that:

(i) among any three of the given points, two are connected by a segment; (ii) the number ofsegments is minimal.

How many segments does a figure satisfying (i) and (ii) have? Give an example of such afigure.

18 Given a convex polygon A1A2 . . . An with area S and a point M in the same plane, determinethe area of polygon M1M2 . . .Mn, where Mi is the image of M under rotation Rα

Aiaround

Ai by αi, i = 1, 2, . . . , n.

19 A natural number is written in each square of an m× n chess board. The allowed move is toadd an integer k to each of two adjacent numbers in such a way that non-negative numbersare obtained. (Two squares are adjacent if they have a common side.) Find a necessary andsufficient condition for it to be possible for all the numbers to be zero after finitely manyoperations.

20 Let n and k be positive integers and let S be a set of n points in the plane such that

i.) no three points of S are collinear, and

ii.) for every point P of S there are at least k points of S equidistant from P.

Prove that:k <

12

+√

2 · n

21 Prove that the intersection of a plane and a regular tetrahedron can be an obtuse-angledtriangle and that the obtuse angle in any such triangle is always smaller than 120.

http://www.artofproblemsolving.com/This file was downloaded from the AoPS Math Olympiad Resources Page Page 3

Page 96: យុវសិស្ ឈឹ ទីុយ IMO Shortlist 1959-2009 · IMO Shortlist 1959 1 Prove that the fraction 21n+4 14n+3 ... Let α be the acute angel subtending, from A, that

IMO Shortlist 1989

22 Prove that in the set 1, 2, . . . , 1989 can be expressed as the disjoint union of subsets Ai, i =1, 2, . . . , 117 such that

i.) each Ai contains 17 elements

ii.) the sum of all the elements in each Ai is the same.

23 A permutation x1, . . . , x2n of the set 1, 2, . . . , 2n where n is a positive integer, is said tohave property T if |xi − xi+1| = n for at least one i in 1, 2, . . . , 2n− 1. Show that, for eachn, there are more permutations with property T than without.

24 For points A1, . . . , A5 on the sphere of radius 1, what is the maximum value that min1≤i,j≤5AiAj

can take? Determine all configurations for which this maximum is attained. (Or: determinethe diameter of any set A1, . . . , A5 for which this maximum is attained.)

25 Let a, b ∈ Z which are not perfect squares. Prove that if

x2 − ay2 − bz2 + abw2 = 0

has a nontrivial solution in integers, then so does

x2 − ay2 − bz2 = 0.

26 Let n ∈ Z+ and let a, b ∈ R. Determine the range of x0 for which

n∑i=0

xi = a andn∑

i=0

x2i = b,

where x0, x1, . . . , xn are real variables.

27 Let m be a positive odd integer, m > 2. Find the smallest positive integer n such that 21989

divides mn − 1.

28 Consider in a plane P the points O,A1, A2, A3, A4 such that

σ(OAiAj) ≥ 1 ∀i, j = 1, 2, 3, 4, i 6= j.

where σ(OAiAj) is the area of triangle OAiAj . Prove that there exists at least one pairi0, j0 ∈ 1, 2, 3, 4 such that

σ(OAiAj) ≥√

2.

29 155 birds P1, . . . , P155 are sitting down on the boundary of a circle C. Two birds Pi, Pj aremutually visible if the angle at centre m(·) of their positions m(PiPj) ≤ 10. Find the smallestnumber of mutually visible pairs of birds, i.e. minimal set of pairs x, y of mutually visiblepairs of birds with x, y ∈ P1, . . . , P155. One assumes that a position (point) on C can beoccupied simultaneously by several birds, e.g. all possible birds.

http://www.artofproblemsolving.com/This file was downloaded from the AoPS Math Olympiad Resources Page Page 4

Page 97: យុវសិស្ ឈឹ ទីុយ IMO Shortlist 1959-2009 · IMO Shortlist 1959 1 Prove that the fraction 21n+4 14n+3 ... Let α be the acute angel subtending, from A, that

IMO Shortlist 1989

30 Prove that for each positive integer n there exist n consecutive positive integers none of whichis an integral power of a prime number.

31 Let a1 ≥ a2 ≥ a3 ∈ Z+ be given and let N(a1, a2, a3) be the number of solutions (x1, x2, x3)of the equation

3∑k=1

ak

xk= 1.

where x1, x2, and x3 are positive integers. Prove that

N(a1, a2, a3) ≤ 6a1a2(3 + ln(2a1)).

32 The vertex A of the acute triangle ABC is equidistant from the circumcenter O and theorthocenter H. Determine all possible values for the measure of angle A.

http://www.artofproblemsolving.com/This file was downloaded from the AoPS Math Olympiad Resources Page Page 5

Page 98: យុវសិស្ ឈឹ ទីុយ IMO Shortlist 1959-2009 · IMO Shortlist 1959 1 Prove that the fraction 21n+4 14n+3 ... Let α be the acute angel subtending, from A, that

IMO Shortlist 1990

1 The integer 9 can be written as a sum of two consecutive integers: 9 = 4 + 5. Moreover, itcan be written as a sum of (more than one) consecutive positive integers in exactly two ways:9 = 4 + 5 = 2 + 3 + 4. Is there an integer that can be written as a sum of 1990 consecutiveintegers and that can be written as a sum of (more than one) consecutive positive integers inexactly 1990 ways?

2 Given n countries with three representatives each, m committees A(1), A(2), . . . , A(m) arecalled a cycle if

(i) each committee has n members, one from each country; (ii) no two committees have thesame membership; (iii) for i = 1, 2, . . . ,m, committee A(i) and committee A(i + 1) have nomember in common, where A(m+1) denotes A(1); (iv) if 1 < |i−j| < m−1, then committeesA(i) and A(j) have at least one member in common.

Is it possible to have a cycle of 1990 committees with 11 countries?

3 Let n ≥ 3 and consider a set E of 2n− 1 distinct points on a circle. Suppose that exactly kof these points are to be colored black. Such a coloring is good if there is at least one pair ofblack points such that the interior of one of the arcs between them contains exactly n pointsfrom E. Find the smallest value of k so that every such coloring of k points of E is good.

4 Assume that the set of all positive integers is decomposed into r (disjoint) subsets A1∪A2∪. . .∪Ar = N. Prove that one of them, say Ai, has the following property: There exists a positivem such that for any k one can find numbers a1, a2, . . . , ak in Ai with 0 < aj+1 − aj ≤ m,(1 ≤ j ≤ k − 1).

5 Given a triangle ABC. Let G, I, H be the centroid, the incenter and the orthocenter oftriangle ABC, respectively. Prove that ∠GIH > 90.

6 Given an initial integer n0 > 1, two players, A and B, choose integers n1, n2, n3, . . . alternatelyaccording to the following rules :

I.) Knowing n2k, A chooses any integer n2k+1 such that

n2k ≤ n2k+1 ≤ n22k.

II.) Knowing n2k+1, B chooses any integer n2k+2 such thatn2k+1

n2k+2

is a prime raised to a positive integer power.

Player A wins the game by choosing the number 1990; player B wins by choosing the number1. For which n0 does :

a.) A have a winning strategy? b.) B have a winning strategy? c.) Neither player have awinning strategy?

http://www.artofproblemsolving.com/This file was downloaded from the AoPS Math Olympiad Resources Page Page 1

Page 99: យុវសិស្ ឈឹ ទីុយ IMO Shortlist 1959-2009 · IMO Shortlist 1959 1 Prove that the fraction 21n+4 14n+3 ... Let α be the acute angel subtending, from A, that

IMO Shortlist 1990

7 Let f(0) = f(1) = 0 and

f(n + 2) = 4n+2 · f(n + 1)− 16n+1 · f(n) + n · 2n2, n = 0, 1, 2, . . .

Show that the numbers f(1989), f(1990), f(1991) are divisible by 13.

8 For a given positive integer k denote the square of the sum of its digits by f1(k) and letfn+1(k) = f1(fn(k)). Determine the value of f1991(21990).

9 The incenter of the triangle ABC is K. The midpoint of AB is C1 and that of AC is B1. Thelines C1K and AC meet at B2, the lines B1K and AB at C2. If the areas of the trianglesAB2C2 and ABC are equal, what is the measure of angle ∠CAB?

10 A plane cuts a right circular cone of volume V into two parts. The plane is tangent to thecircumference of the base of the cone and passes through the midpoint of the altitude. Findthe volume of the smaller part.

Original formulation:

A plane cuts a right circular cone into two parts. The plane is tangent to the circumferenceof the base of the cone and passes through the midpoint of the altitude. Find the ratio of thevolume of the smaller part to the volume of the whole cone.

11 Chords AB and CD of a circle intersect at a point E inside the circle. Let M be an interiorpoint of the segment EB. The tangent line at E to the circle through D, E, and M intersectsthe lines BC and AC at F and G, respectively. If

AM

AB= t,

find EGEF in terms of t.

12 Let ABC be a triangle, and let the angle bisectors of its angles CAB and ABC meet thesides BC and CA at the points D and F , respectively. The lines AD and BF meet theline through the point C parallel to AB at the points E and G respectively, and we haveFG = DE. Prove that CA = CB.

Original formulation:

Let ABC be a triangle and L the line through C parallel to the side AB. Let the internalbisector of the angle at A meet the side BC at D and the line L at E and let the internalbisector of the angle at B meet the side AC at F and the line L at G. If GF = DE, provethat AC = BC.

http://www.artofproblemsolving.com/This file was downloaded from the AoPS Math Olympiad Resources Page Page 2

Page 100: យុវសិស្ ឈឹ ទីុយ IMO Shortlist 1959-2009 · IMO Shortlist 1959 1 Prove that the fraction 21n+4 14n+3 ... Let α be the acute angel subtending, from A, that

IMO Shortlist 1990

13 An eccentric mathematician has a ladder with n rungs that he always ascends and descendsin the following way: When he ascends, each step he takes covers a rungs of the ladder, andwhen he descends, each step he takes covers b rungs of the ladder, where a and b are fixedpositive integers. By a sequence of ascending and descending steps he can climb from groundlevel to the top rung of the ladder and come back down to ground level again. Find, withproof, the minimum value of n, expressed in terms of a and b.

14 In the coordinate plane a rectangle with vertices (0, 0), (m, 0), (0, n), (m,n) is given whereboth m and n are odd integers. The rectangle is partitioned into triangles in such a way that

(i) each triangle in the partition has at least one side (to be called a good side) that lies on aline of the form x = j or y = k, where j and k are integers, and the altitude on this side haslength 1;

(ii) each bad side (i.e., a side of any triangle in the partition that is not a good one) is acommon side of two triangles in the partition.

Prove that there exist at least two triangles in the partition each of which has two good sides.

15 Determine for which positive integers k the set

X = 1990, 1990 + 1, 1990 + 2, . . . , 1990 + k

can be partitioned into two disjoint subsets A and B such that the sum of the elements of Ais equal to the sum of the elements of B.

16 Prove that there exists a convex 1990-gon with the following two properties :

a.) All angles are equal. b.) The lengths of the 1990 sides are the numbers 12, 22, 32, · · · ,19902 in some order.

17 Unit cubes are made into beads by drilling a hole through them along a diagonal. The beadsare put on a string in such a way that they can move freely in space under the restrictionthat the vertices of two neighboring cubes are touching. Let A be the beginning vertex andB be the end vertex. Let there be p× q × r cubes on the string (p, q, r ≥ 1).

(a) Determine for which values of p, q, and r it is possible to build a block with dimensionsp, q, and r. Give reasons for your answers. (b) The same question as (a) with the extracondition that A = B.

18 Let a, b ∈ N with 1 ≤ a ≤ b, and M =[

a+b2

]. Define a function f : Z 7→ Z by

f(n) =

n + a, if n ≤ M,

n− b, if n ≥ M.

Let f1(n) = f(n), fi+1(n) = f(f i(n)), i = 1, 2, . . . Find the smallest natural number k suchthat fk(0) = 0.

http://www.artofproblemsolving.com/This file was downloaded from the AoPS Math Olympiad Resources Page Page 3

Page 101: យុវសិស្ ឈឹ ទីុយ IMO Shortlist 1959-2009 · IMO Shortlist 1959 1 Prove that the fraction 21n+4 14n+3 ... Let α be the acute angel subtending, from A, that

IMO Shortlist 1990

19 Let P be a point inside a regular tetrahedron T of unit volume. The four planes passingthrough P and parallel to the faces of T partition T into 14 pieces. Let f(P ) be the jointvolume of those pieces that are neither a tetrahedron nor a parallelepiped (i.e., pieces adjacentto an edge but not to a vertex). Find the exact bounds for f(P ) as P varies over T.

20 Prove that every integer k greater than 1 has a multiple that is less than k4 and can bewritten in the decimal system with at most four different digits.

21 Let n be a composite natural number and p a proper divisor of n. Find the binary represen-tation of the smallest natural number N such that

(1 + 2p + 2n−p)N − 12n

is an integer.

22 Ten localities are served by two international airlines such that there exists a direct service(without stops) between any two of these localities and all airline schedules offer round-tripservice between the cities they serve. Prove that at least one of the airlines can offer twodisjoint round trips each containing an odd number of landings.

23 Determine all integers n > 1 such that

2n + 1n2

is an integer.

24 Let w, x, y, z are non-negative reals such that wx + xy + yz + zw = 1. Show that w3

x+y+z +x3

w+y+z + y3

w+x+z + z3

w+x+y ≥13 .

25 Let Q+ be the set of positive rational numbers. Construct a function f : Q+ → Q+ such that

f(xf(y)) =f(x)

y

for all x, y in Q+.

26 Let p(x) be a cubic polynomial with rational coefficients. q1, q2, q3, ... is a sequence ofrationals such that qn = p(qn+1) for all positive n. Show that for some k, we have qn+k = qn

for all positive n.

27 Find all natural numbers n for which every natural number whose decimal representation hasn− 1 digits 1 and one digit 7 is prime.

http://www.artofproblemsolving.com/This file was downloaded from the AoPS Math Olympiad Resources Page Page 4

Page 102: យុវសិស្ ឈឹ ទីុយ IMO Shortlist 1959-2009 · IMO Shortlist 1959 1 Prove that the fraction 21n+4 14n+3 ... Let α be the acute angel subtending, from A, that

IMO Shortlist 1990

28 Prove that on the coordinate plane it is impossible to draw a closed broken line such that

(i) the coordinates of each vertex are rational; (ii) the length each of its edges is 1; (iii) theline has an odd number of vertices.

http://www.artofproblemsolving.com/This file was downloaded from the AoPS Math Olympiad Resources Page Page 5

Page 103: យុវសិស្ ឈឹ ទីុយ IMO Shortlist 1959-2009 · IMO Shortlist 1959 1 Prove that the fraction 21n+4 14n+3 ... Let α be the acute angel subtending, from A, that

IMO Shortlist 1991

1 Given a point P inside a triangle 4ABC. Let D, E, F be the orthogonal projections of thepoint P on the sides BC, CA, AB, respectively. Let the orthogonal projections of the pointA on the lines BP and CP be M and N , respectively. Prove that the lines ME, NF , BCare concurrent.

Original formulation:

Let ABC be any triangle and P any point in its interior. Let P1, P2 be the feet of theperpendiculars from P to the two sides AC and BC. Draw AP and BP, and from C dropperpendiculars to AP and BP. Let Q1 and Q2 be the feet of these perpendiculars. Prove thatthe lines Q1P2, Q2P1, and AB are concurrent.

2 ABC is an acute-angled triangle. M is the midpoint of BC and P is the point on AM suchthat MB = MP . H is the foot of the perpendicular from P to BC. The lines through Hperpendicular to PB, PC meet AB,AC respectively at Q,R. Show that BC is tangent tothe circle through Q,H,R at H.

Original Formulation:

For an acute triangle ABC, M is the midpoint of the segment BC, P is a point on the segmentAM such that PM = BM, H is the foot of the perpendicular line from P to BC, Q is thepoint of intersection of segment AB and the line passing through H that is perpendicular toPB, and finally, R is the point of intersection of the segment AC and the line passing throughH that is perpendicular to PC. Show that the circumcircle of QHR is tangent to the sideBC at point H.

3 Let S be any point on the circumscribed circle of PQR. Then the feet of the perpendicularsfrom S to the three sides of the triangle lie on the same straight line. Denote this line byl(S, PQR). Suppose that the hexagon ABCDEF is inscribed in a circle. Show that the fourlines l(A,BDF ), l(B,ACE), l(D,ABF ), and l(E,ABC) intersect at one point if and only ifCDEF is a rectangle.

4 Let ABC be a triangle and P an interior point of ABC . Show that at least one of theangles ∠PAB, ∠PBC, ∠PCA is less than or equal to 30.

5 In the triangle ABC, with ∠A = 60, a parallel IF to AC is drawn through the incenterI of the triangle, where F lies on the side AB. The point P on the side BC is such that3BP = BC. Show that ∠BFP = ∠B

2 .

7 ABCD is a terahedron: AD + BD = AC + BC, BD + CD = BA + CA, CD + AD =CB + AB, M, N,P are the mid points of BC, CA, AB. OA = OB = OC = OD. Prove that∠MOP = ∠NOP = ∠NOM.

http://www.artofproblemsolving.com/This file was downloaded from the AoPS Math Olympiad Resources Page Page 1

Page 104: យុវសិស្ ឈឹ ទីុយ IMO Shortlist 1959-2009 · IMO Shortlist 1959 1 Prove that the fraction 21n+4 14n+3 ... Let α be the acute angel subtending, from A, that

IMO Shortlist 1991

8 S be a set of n points in the plane. No three points of S are collinear. Prove that there existsa set P containing 2n − 5 points satisfying the following condition: In the interior of everytriangle whose three vertices are elements of S lies a point that is an element of P.

9 In the plane we are given a set E of 1991 points, and certain pairs of these points are joinedwith a path. We suppose that for every point of E, there exist at least 1593 other points ofE to which it is joined by a path. Show that there exist six points of E every pair of whichare joined by a path.

Alternative version: Is it possible to find a set E of 1991 points in the plane and paths joiningcertain pairs of the points in E such that every point of E is joined with a path to at least1592 other points of E, and in every subset of six points of E there exist at least two pointsthat are not joined?

10 Suppose G is a connected graph with k edges. Prove that it is possible to label the edges1, 2, . . . , k in such a way that at each vertex which belongs to two or more edges, the greatestcommon divisor of the integers labeling those edges is equal to 1.

[hide=”Graph-Definition”]A graph consists of a set of points, called vertices, together witha set of edges joining certain pairs of distinct vertices. Each pair of vertices u, v belongs toat most one edge. The graph G is connected if for each pair of distinct vertices x, y there issome sequence of vertices x = v0, v1, v2, · · · , vm = y such that each pair vi, vi+1 (0 ≤ i < m)is joined by an edge of G.

11 Prove that∑995

k=0(−1)k

1991−k

(1991−k

k

)= 1

1991

12 Let S = 1, 2, 3, · · · , 280. Find the smallest integer n such that each n-element subset of Scontains five numbers which are pairwise relatively prime.

13 Given any integer n ≥ 2, assume that the integers a1, a2, . . . , an are not divisible by n and,moreover, that n does not divide

∑ni=1 ai. Prove that there exist at least n different sequences

(e1, e2, . . . , en) consisting of zeros or ones such∑n

i=1 ei · ai is divisible by n.

14 Let a, b, c be integers and p an odd prime number. Prove that if f(x) = ax2 + bx + c is aperfect square for 2p− 1 consecutive integer values of x, then p divides b2 − 4ac.

15 Let an be the last nonzero digit in the decimal representation of the number n!. Does thesequence a1, a2, . . . , an, . . . become periodic after a finite number of terms?

16 Let n > 6 be an integer and a1, a2, · · · , ak be all the natural numbers less than n andrelatively prime to n. If

a2 − a1 = a3 − a2 = · · · = ak − ak−1 > 0,

prove that n must be either a prime number or a power of 2.

http://www.artofproblemsolving.com/This file was downloaded from the AoPS Math Olympiad Resources Page Page 2

Page 105: យុវសិស្ ឈឹ ទីុយ IMO Shortlist 1959-2009 · IMO Shortlist 1959 1 Prove that the fraction 21n+4 14n+3 ... Let α be the acute angel subtending, from A, that

IMO Shortlist 1991

17 Find all positive integer solutions x, y, z of the equation 3x + 4y = 5z.

18 Find the highest degree k of 1991 for which 1991k divides the number

199019911992+ 199219911990

.

19 Let α be a rational number with 0 < α < 1 and cos(3πα)+2 cos(2πα) = 0. Prove that α = 23 .

20 Let α be the positive root of the equation x2 = 1991x + 1. For natural numbers m and ndefine

m ∗ n = mn + bαmcbαnc.

Prove that for all natural numbers p, q, and r,

(p ∗ q) ∗ r = p ∗ (q ∗ r).

21 Let f(x) be a monic polynomial of degree 1991 with integer coefficients. Define g(x) =f2(x)−9. Show that the number of distinct integer solutions of g(x) = 0 cannot exceed 1995.

22 Real constants a, b, c are such that there is exactly one square all of whose vertices lie on thecubic curve y = x3 + ax2 + bx + c. Prove that the square has sides of length 4

√72.

23 Let f and g be two integer-valued functions defined on the set of all integers such that

(a) f(m+f(f(n))) = −f(f(m+1)−n for all integers m and n; (b) g is a polynomial functionwith integer coefficients and g(n) = g(f(n)) ∀n ∈ Z.

24 An odd integer n ≥ 3 is said to be nice if and only if there is at least one permutation a1, · · · , an

of 1, · · · , n such that the n sums a1 − a2 + a3 − · · · − an−1 + an, a2 − a3 + a3 − · · · − an + a1,a3 − a4 + a5 − · · · − a1 + a2, · · · , an − a1 + a2 − · · · − an−2 + an−1 are all positive. Determinethe set of all ‘nice’ integers.

25 Suppose that n ≥ 2 and x1, x2, . . . , xn are real numbers between 0 and 1 (inclusive). Provethat for some index i between 1 and n− 1 the inequality

xi(1− xi+1) ≥14x1(1− xn)

26 Let n ≥ 2, n ∈ N and let p, a1, a2, . . . , an, b1, b2, . . . , bn ∈ R satisfying 12 ≤ p ≤ 1, 0 ≤ ai,

0 ≤ bi ≤ p, i = 1, . . . , n, andn∑

i=1

ai =n∑

i=1

bi.

http://www.artofproblemsolving.com/This file was downloaded from the AoPS Math Olympiad Resources Page Page 3

Page 106: យុវសិស្ ឈឹ ទីុយ IMO Shortlist 1959-2009 · IMO Shortlist 1959 1 Prove that the fraction 21n+4 14n+3 ... Let α be the acute angel subtending, from A, that

IMO Shortlist 1991

Prove the inequality:n∑

i=1

bi

n∏j=1,j 6=i

aj ≤p

(n− 1)n−1.

27 Determine the maximum value of the sum∑i<j

xixj(xi + xj)

over all n−tuples (x1, . . . , xn), satisfying xi ≥ 0 and∑n

i=1 xi = 1.

28 An infinite sequence x0, x1, x2, . . . of real numbers is said to be bounded if there is a constantC such that |xi| ≤ C for every i ≥ 0. Given any real number a > 1, construct a boundedinfinite sequence x0, x1, x2, . . . such that

|xi − xj ||i− j|a ≥ 1

for every pair of distinct nonnegative integers i, j.

29 We call a set S on the real line R superinvariant if for any stretching A of the set by thetransformation taking x to A(x) = x0 + a(x− x0), a > 0 there exists a translation B, B(x) =x + b, such that the images of S under A and B agree; i.e., for any x ∈ S there is a y ∈ Ssuch that A(x) = B(y) and for any t ∈ S there is a u ∈ S such that B(t) = A(u). Determineall superinvariant sets.

30 Two students A and B are playing the following game: Each of them writes down on a sheetof paper a positive integer and gives the sheet to the referee. The referee writes down on ablackboard two integers, one of which is the sum of the integers written by the players. Afterthat, the referee asks student A : Can you tell the integer written by the other student? If Aanswers no, the referee puts the same question to student B. If B answers no, the referee putsthe question back to A, and so on. Assume that both students are intelligent and truthful.Prove that after a finite number of questions, one of the students will answer yes.

http://www.artofproblemsolving.com/This file was downloaded from the AoPS Math Olympiad Resources Page Page 4

Page 107: យុវសិស្ ឈឹ ទីុយ IMO Shortlist 1959-2009 · IMO Shortlist 1959 1 Prove that the fraction 21n+4 14n+3 ... Let α be the acute angel subtending, from A, that

IMO Shortlist 1992

1 Prove that for any positive integer m there exist an infinite number of pairs of integers (x, y)such that

(i) x and y are relatively prime; (ii) y divides x2+m; (iii) x divides y2+m. (iv) x+y ≤ m+1−(optional condition)

2 Let R+ be the set of all non-negative real numbers. Given two positive real numbers a andb, suppose that a mapping f : R+ 7→ R+ satisfies the functional equation:

f(f(x)) + af(x) = b(a + b)x.

Prove that there exists a unique solution of this equation.

3 The diagonals of a quadrilateral ABCD are perpendicular: AC ⊥ BD. Four squares, ABEF, BCGH,CDIJ,DAKL,are erected externally on its sides. The intersection points of the pairs of straight linesCL,DF,AH, BJ are denoted by P1, Q1, R1, S1, respectively (left figure), and the intersectionpoints of the pairs of straight lines AI,BK, CEDG are denoted by P2, Q2, R2, S2, respectively(right figure). Prove that P1Q1R1S1

∼= P2Q2R2S2 where P1, Q1, R1, S1 and P2, Q2, R2, S2 arethe two quadrilaterals.

Alternative formulation: Outside a convex quadrilateral ABCD with perpendicular diag-onals, four squares AEFB, BGHC,CIJD, DKLA, are constructed (vertices are given incounterclockwise order). Prove that the quadrilaterals Q1 and Q2 formed by the linesAG, BI, CK, DE and AJ,BL, CF,DH, respectively, are congruent.

4 Consider 9 points in space, no four of which are coplanar. Each pair of points is joined byan edge (that is, a line segment) and each edge is either colored blue or red or left uncolored.Find the smallest value of n such that whenever exactly n edges are colored, the set ofcolored edges necessarily contains a triangle all of whose edges have the same color.

5 A convex quadrilateral has equal diagonals. An equilateral triangle is constructed on theoutside of each side of the quadrilateral. The centers of the triangles on opposite sides arejoined. Show that the two joining lines are perpendicular.

Alternative formulation. Given a convex quadrilateral ABCD with congruent diagonals AC =BD. Four regular triangles are errected externally on its sides. Prove that the segments joiningthe centroids of the triangles on the opposite sides are perpendicular to each other.

Original formulation: Let ABCD be a convex quadrilateral such that AC = BD. Equilateraltriangles are constructed on the sides of the quadrilateral. Let O1, O2, O3, O4 be the centers ofthe triangles constructed on AB,BC, CD,DA respectively. Show that O1O3 is perpendicularto O2O4.

http://www.artofproblemsolving.com/This file was downloaded from the AoPS Math Olympiad Resources Page Page 1

Page 108: យុវសិស្ ឈឹ ទីុយ IMO Shortlist 1959-2009 · IMO Shortlist 1959 1 Prove that the fraction 21n+4 14n+3 ... Let α be the acute angel subtending, from A, that

IMO Shortlist 1992

6 Let R denote the set of all real numbers. Find all functions f : R → R such that

f(x2 + f(y)

)= y + (f(x))2 for all x, y ∈ R.

7 Two circles Ω1 and Ω2 are externally tangent to each other at a point I, and both of thesecircles are tangent to a third circle Ω which encloses the two circles Ω1 and Ω2. The commontangent to the two circles Ω1 and Ω2 at the point I meets the circle Ω at a point A. Onecommon tangent to the circles Ω1 and Ω2 which doesn’t pass through I meets the circle Ω atthe points B and C such that the points A and I lie on the same side of the line BC. Provethat the point I is the incenter of triangle ABC.

Alternative formulation. Two circles touch externally at a point I. The two circles lie insidea large circle and both touch it. The chord BC of the large circle touches both smaller circles(not at I). The common tangent to the two smaller circles at the point I meets the largecircle at a point A, where the points A and I are on the same side of the chord BC. Showthat the point I is the incenter of triangle ABC.

8 Show that in the plane there exists a convex polygon of 1992 sides satisfying the followingconditions:

(i) its side lengths are 1, 2, 3, . . . , 1992 in some order; (ii) the polygon is circumscribable abouta circle.

Alternative formulation: Does there exist a 1992-gon with side lengths 1, 2, 3, . . . , 1992 cir-cumscribed about a circle? Answer the same question for a 1990-gon.

9 Let f(x) be a polynomial with rational coefficients and α be a real number such that

α3 − α = [f(α)]3 − f(α) = 331992.

Prove that for each n ≥ 1,[fn(α)]3 − fn(α) = 331992,

where fn(x) = f(f(· · · f(x))), and n is a positive integer.

10 Let S be a finite set of points in three-dimensional space. Let Sx, Sy, Sz be the sets consist-ing of the orthogonal projections of the points of S onto the yz-plane, zx-plane, xy-plane,respectively. Prove that

|S|2 ≤ |Sx| · |Sy| · |Sz|,

where |A| denotes the number of elements in the finite set A.

[hide=”Note”] Note: The orthogonal projection of a point onto a plane is the foot of theperpendicular from that point to the plane.

http://www.artofproblemsolving.com/This file was downloaded from the AoPS Math Olympiad Resources Page Page 2

Page 109: យុវសិស្ ឈឹ ទីុយ IMO Shortlist 1959-2009 · IMO Shortlist 1959 1 Prove that the fraction 21n+4 14n+3 ... Let α be the acute angel subtending, from A, that

IMO Shortlist 1992

11 In a triangle ABC, let D and E be the intersections of the bisectors of ∠ABC and ∠ACBwith the sides AC,AB, respectively. Determine the angles ∠A,∠B,∠C if ∠BDE = 24,∠CED = 18.

12 Let f, g and a be polynomials with real coefficients, f and g in one variable and a in twovariables. Suppose

f(x)− f(y) = a(x, y)(g(x)− g(y))∀x, y ∈ R

Prove that there exists a polynomial h with f(x) = h(g(x)) ∀x ∈ R.

13 Find all integers a, b, c with 1 < a < b < c such that

(a− 1)(b− 1)(c− 1)

is a divisor of abc− 1.

14 For any positive integer x define g(x) as greatest odd divisor of x, and

f(x) =

x2 + x

g(x) if x is even,

2x+12 if x is odd.

Construct the sequence x1 = 1, xn+1 = f(xn). Show that the number 1992 appears in thissequence, determine the least n such that xn = 1992, and determine whether n is unique.

15 Does there exist a set M with the following properties?

(i) The set M consists of 1992 natural numbers. (ii) Every element in M and the sum of anynumber of elements have the form mk (m, k ∈ N, k ≥ 2).

16 Prove that 5125−1525−1

is a composite number.

17 Let α(n) be the number of digits equal to one in the binary representation of a positive integern. Prove that:

(a) the inequality α(n)(n2

)≤ 1

2α(n)(α(n) + 1) holds; (b) the above inequality is an equality

for infinitely many positive integers, and (c) there exists a sequence (ni)∞1 such that

α(n2i )

α(ni

goes to zero as i goes to ∞.

Alternative problem: Prove that there exists a sequence a sequence (ni)∞1 such that

α(n2i )

α(ni)

(d) ∞; (e) an arbitrary real number γ ∈ (0, 1); (f) an arbitrary real number γ ≥ 0;

as i goes to ∞.

http://www.artofproblemsolving.com/This file was downloaded from the AoPS Math Olympiad Resources Page Page 3

Page 110: យុវសិស្ ឈឹ ទីុយ IMO Shortlist 1959-2009 · IMO Shortlist 1959 1 Prove that the fraction 21n+4 14n+3 ... Let α be the acute angel subtending, from A, that

IMO Shortlist 1992

18 Let bxc denote the greatest integer less than or equal to x. Pick any x1 in [0, 1) and define

the sequence x1, x2, x3, . . . by xn+1 = 0 if xn = 0 and xn+1 = 1xn−

⌊1

xn

⌋otherwise. Prove

that

x1 + x2 + . . . + xn <F1

F2+

F2

F3+ . . . +

Fn

Fn+1,

where F1 = F2 = 1 and Fn+2 = Fn+1 + Fn for n ≥ 1.

19 Let f(x) = x8 + 4x6 + 2x4 + 28x2 + 1. Let p > 3 be a prime and suppose there exists aninteger z such that p divides f(z). Prove that there exist integers z1, z2, . . . , z8 such that if

g(x) = (x− z1)(x− z2) · . . . · (x− z8),

then all coefficients of f(x)− g(x) are divisible by p.

20 In the plane let C be a circle, L a line tangent to the circle C, and M a point on L. Findthe locus of all points P with the following property: there exists two points Q,R on Lsuch that M is the midpoint of QR and C is the inscribed circle of triangle PQR.

21 For each positive integer n, S(n) is defined to be the greatest integer such that, for everypositive integer k ≤ S(n), n2 can be written as the sum of k positive squares.

a.) Prove that S(n) ≤ n2 − 14 for each n ≥ 4. b.) Find an integer n such that S(n) =n2 − 14. c.) Prove that there are infintely many integers n such that S(n) = n2 − 14.

http://www.artofproblemsolving.com/This file was downloaded from the AoPS Math Olympiad Resources Page Page 4

Page 111: យុវសិស្ ឈឹ ទីុយ IMO Shortlist 1959-2009 · IMO Shortlist 1959 1 Prove that the fraction 21n+4 14n+3 ... Let α be the acute angel subtending, from A, that

IMO Shortlist 1993

Algebra

1 Define a sequence < f(n) >∞n=1 of positive integers by

f(1) = 1

and

f(n) =

f(n− 1)− n if f(n− 1) > n;f(n− 1) + n if f(n− 1) ≤ n,

for n ≥ 2. Let S = n ∈ N|f(n) = 1993.(i) Prove that S is an infinite set. (ii) Find the least positive integer in S. (iii) If all theelements of S are written in ascending order as

n1 < n2 < n3 < . . . ,

show thatlimi→∞

ni+1

ni= 3.

2 Show that there exists a finite set A ⊂ R2 such that for every X ∈ A there are pointsY1, Y2, . . . , Y1993 in A such that the distance between X and Yi is equal to 1, for every i.

3 Prove thata

b + 2c + 3d+

b

c + 2d + 3a+

c

d + 2a + 3b+

d

a + 2b + 3c≥ 2

3for all positive real numbers a, b, c, d.

4 Solve the following system of equations, in which a is a given number satisfying |a| > 1:

x21 = ax2 + 1

x22 = ax3 + 1

. . .x2

999 = ax1000 + 1x2

1000 = ax1 + 1

5 a > 0 and b, c are integers such that ac b2 is a square-free positive integer P. [hide=”Forexample”] P could be 3*5, but not 32 ∗ 5.

Let f(n) be the number of pairs of integers d, e such that ad2 + 2bde + ce2 = n. Show thatf(n) isfinite and that f(n) = f(P kn) for every positive integer k.

Original Statement:

http://www.artofproblemsolving.com/This file was downloaded from the AoPS Math Olympiad Resources Page Page 1

Page 112: យុវសិស្ ឈឹ ទីុយ IMO Shortlist 1959-2009 · IMO Shortlist 1959 1 Prove that the fraction 21n+4 14n+3 ... Let α be the acute angel subtending, from A, that

IMO Shortlist 1993

Let a, b, c be given integers a > 0, ac − b2 = P = P1 · · ·Pn where P1 · · ·Pn are (distinct) primenumbers. Let M(n) denote the number of pairs of integers (x, y) for which

ax2 + 2bxy + cy2 = n.

Prove that M(n) is finite and M(n) = M(Pk · n) for every integer k ≥ 0. Note that the ”n” in PN

and the ”n” in M(n) do not have to be the same.

6 Let N = 1, 2, 3, . . .. Determine if there exists a strictly increasing function f : N 7→ N with thefollowing properties:

(i) f(1) = 2;

(ii) f(f(n)) = f(n) + n, (n ∈ N).

7 Let n > 1 be an integer and let f(x) = xn + 5 · xn−1 + 3. Prove that there do not exist polynomialsg(x), h(x), each having integer coefficients and degree at least one, such that f(x) = g(x) · h(x).

8 Let c1, . . . , cn ∈ R with n ≥ 2 such that

0 ≤n∑

i=1

ci ≤ n.

Show that we can find integers k1, . . . , kn such that

n∑i=1

ki = 0

and1− n ≤ ci + n · ki ≤ n

for every i = 1, . . . , n.

[hide=”Another formulation:”] Let x1, . . . , xn, with n ≥ 2 be real numbers such that

|x1 + . . . + xn| ≤ n.

Show that there exist integers k1, . . . , kn such that

|k1 + . . . + kn| = 0.

and|xi + 2 · n · ki| ≤ 2 · n− 1

for every i = 1, . . . , n. In order to prove this, denote ci = 1+xi2 for i = 1, . . . , n, etc.

http://www.artofproblemsolving.com/This file was downloaded from the AoPS Math Olympiad Resources Page Page 2

Page 113: យុវសិស្ ឈឹ ទីុយ IMO Shortlist 1959-2009 · IMO Shortlist 1959 1 Prove that the fraction 21n+4 14n+3 ... Let α be the acute angel subtending, from A, that

IMO Shortlist 1993

9 Let a, b, c, d be four non-negative numbers satisfying

a + b + c + d = 1.

Prove the inequality

a · b · c + b · c · d + c · d · a + d · a · b ≤ 127

+17627

· a · b · c · d.

http://www.artofproblemsolving.com/This file was downloaded from the AoPS Math Olympiad Resources Page Page 3

Page 114: យុវសិស្ ឈឹ ទីុយ IMO Shortlist 1959-2009 · IMO Shortlist 1959 1 Prove that the fraction 21n+4 14n+3 ... Let α be the acute angel subtending, from A, that

IMO Shortlist 1993

Combinatorics

1 a) Show that the set Q+ of all positive rationals can be partitioned into three disjoint subsets.A,B, C satisfying the following conditions:

BA = B;B2 = C;BC = A;

where HK stands for the set hk : h ∈ H, k ∈ K for any two subsets H,K of Q+ and H2

stands for HH.

b) Show that all positive rational cubes are in A for such a partition of Q+.

c) Find such a partition Q+ = A∪B∪C with the property that for no positive integer n ≤ 34,both n and n + 1 are in A, that is,

minn ∈ N : n ∈ A,n + 1 ∈ A > 34.

2 Let n, k ∈ Z+ with k ≤ n and let S be a set containing n distinct real numbers. Let T be aset of all real numbers of the form x1 +x2 + . . .+xk where x1, x2, . . . , xk are distinct elementsof S. Prove that T contains at least k(n− k) + 1 distinct elements.

3 Let n > 1 be an integer. In a circular arrangement of n lamps L0, . . . , Ln−1, each of of whichcan either ON or OFF, we start with the situation where all lamps are ON, and then carryout a sequence of steps, Step0, Step1, . . . . If Lj−1 (j is taken mod n) is ON then Stepj changesthe state of Lj (it goes from ON to OFF or from OFF to ON) but does not change the stateof any of the other lamps. If Lj−1 is OFF then Stepj does not change anything at all. Showthat:

(i) There is a positive integer M(n) such that after M(n) steps all lamps are ON again,

(ii) If n has the form 2k then all the lamps are ON after n2 − 1 steps,

(iii) If n has the form 2k + 1 then all lamps are ON after n2 − n + 1 steps.

4 Let n ≥ 2, n ∈ N and A0 = (a01, a02, . . . , a0n) be any n−tuple of natural numbers, such that0 ≤ a0i ≤ i−1, for i = 1, . . . , n. n−tuples A1 = (a11, a12, . . . , a1n), A2 = (a21, a22, . . . , a2n), . . .are defined by: ai+1,j = Cardai,l|1 ≤ l ≤ j − 1, ai,l ≥ ai,j, for i ∈ N and j = 1, . . . , n. Provethat there exists k ∈ N, such that Ak+2 = Ak.

5 Let Sn be the number of sequences (a1, a2, . . . , an), where ai ∈ 0, 1, in which no six consec-utive blocks are equal. Prove that Sn →∞ when n →∞.

http://www.artofproblemsolving.com/This file was downloaded from the AoPS Math Olympiad Resources Page Page 4

Page 115: យុវសិស្ ឈឹ ទីុយ IMO Shortlist 1959-2009 · IMO Shortlist 1959 1 Prove that the fraction 21n+4 14n+3 ... Let α be the acute angel subtending, from A, that

IMO Shortlist 1993

Geometry

1 Let ABC be a triangle, and I its incenter. Consider a circle which lies inside the circumcircleof triangle ABC and touches it, and which also touches the sides CA and BC of triangleABC at the points D and E, respectively. Show that the point I is the midpoint of thesegment DE.

2 A circle S bisects a circle S′ if it cuts S′ at opposite ends of a diameter. SA, SB,SC are circleswith distinct centers A,B, C (respectively). Show that A,B, C are collinear iff there is nounique circle S which bisects each of SA, SB,SC . Show that if there is more than one circleS which bisects each of SA, SB,SC , then all such circles pass through two fixed points. Findthese points.

Original Statement:

A circle S is said to cut a circle Σ diametrically if and only if their common chord isa diameter of Σ. Let SA, SB, SC be three circles with distinct centres A,B, C respectively.Prove that A,B, C are collinear if and only if there is no unique circle S which cuts each ofSA, SB, SC diametrically. Prove further that if there exists more than one circle S which cutseach SA, SB, SC diametrically, then all such circles S pass through two fixed points. Locatethese points in relation to the circles SA, SB, SC .

3 Let triangle ABC be such that its circumradius is R = 1. Let r be the inradius of ABC andlet p be the inradius of the orthic triangle A′B′C ′ of triangle ABC. Prove that

p ≤ 1− 13 · (1 + r)2

.

[hide=”Similar Problem posted by Pascual2005”]

Let ABC be a triangle with circumradius R and inradius r. If p is the inradius of the orthic

triangle of triangle ABC, show that pR ≤ 1− (1+ r

R)2

3 .

Note. The orthic triangle of triangle ABC is defined as the triangle whose vertices are thefeet of the altitudes of triangle ABC.

SOLUTION 1 by mecrazywong:

p = 2R cos A cos B cos C, 1 + rR = 1 + 4 sinA/2 sinB/2 sinC/2 = cos A + cos B + cos C. Thus,

the ineqaulity is equivalent to 6 cos A cos B cos C + (cos A + cos B + cos C)2 ≤ 3. But this iseasy since cos A + cos B + cos C ≤ 3/2, cos A cos B cos C ≤ 1/8.

SOLUTION 2 by Virgil Nicula:

http://www.artofproblemsolving.com/This file was downloaded from the AoPS Math Olympiad Resources Page Page 5

Page 116: យុវសិស្ ឈឹ ទីុយ IMO Shortlist 1959-2009 · IMO Shortlist 1959 1 Prove that the fraction 21n+4 14n+3 ... Let α be the acute angel subtending, from A, that

IMO Shortlist 1993

I note the inradius r′ of a orthic triangle.

Must prove the inequality r′

R ≤ 1− 13

(1 + r

R

)2.

From the wellknown relations r′ = 2R cos A cos B cos C

and cos A cos B cos C ≤ 18 results r′

R ≤ 14 .

But 14 ≤ 1− 1

3

(1 + r

R

)2 ⇐⇒ 13

(1 + r

R

)2 ≤ 34 ⇐⇒(

1 + rR

)2 ≤(

32

)2 ⇐⇒ 1 + rR ≤ 3

2 ⇐⇒rR ≤ 1

2 ⇐⇒ 2r ≤ R (true).

Therefore, r′

R ≤ 14 ≤ 1− 1

3

(1 + r

R

)2 =⇒ r′

R ≤ 1− 13

(1 + r

R

)2.

SOLUTION 3 by darij grinberg:

I know this is not quite an ML reference, but the problem was discussed in Hyacinthosmessages 6951, 6978, 6981, 6982, 6985, 6986 (particularly the last message).

4 Given a triangle ABC, let D and E be points on the side BC such that ∠BAD = ∠CAE. IfM and N are, respectively, the points of tangency of the incircles of the triangles ABD andACE with the line BC, then show that

1MB

+1

MD=

1NC

+1

NE.

5 On an infinite chessboard, a solitaire game is played as follows: at the start, we have n2 piecesoccupying a square of side n. The only allowed move is to jump over an occupied square toan unoccupied one, and the piece which has been jumped over is removed. For which n canthe game end with only one piece remaining on the board?

6 For three points A,B, C in the plane, we define m(ABC) to be the smallest length of the threeheights of the triangle ABC, where in the case A, B, C are collinear, we set m(ABC) = 0.Let A, B, C be given points in the plane. Prove that for any point X in the plane,

m(ABC) ≤ m(ABX) + m(AXC) + m(XBC).

7 Let A, B, C, D be four points in the plane, with C and D on the same side of the line AB,such that AC ·BD = AD ·BC and ∠ADB = 90 + ∠ACB. Find the ratio

AB · CD

AC ·BD,

and prove that the circumcircles of the triangles ACD and BCD are orthogonal. (Intersectingcircles are said to be orthogonal if at either common point their tangents are perpendicuar.Thus, proving that the circumcircles of the triangles ACD and BCD are orthogonal is equiv-alent to proving that the tangents to the circumcircles of the triangles ACD and BCD atthe point C are perpendicular.)

http://www.artofproblemsolving.com/This file was downloaded from the AoPS Math Olympiad Resources Page Page 6

Page 117: យុវសិស្ ឈឹ ទីុយ IMO Shortlist 1959-2009 · IMO Shortlist 1959 1 Prove that the fraction 21n+4 14n+3 ... Let α be the acute angel subtending, from A, that

IMO Shortlist 1993

8 The vertices D,E, F of an equilateral triangle lie on the sides BC, CA, AB respectively of atriangle ABC. If a, b, c are the respective lengths of these sides, and S the area of ABC, provethat

DE ≥ 2 ·√

2 · S√a2 + b2 + c2 + 4 ·

√3 · S

.

http://www.artofproblemsolving.com/This file was downloaded from the AoPS Math Olympiad Resources Page Page 7

Page 118: យុវសិស្ ឈឹ ទីុយ IMO Shortlist 1959-2009 · IMO Shortlist 1959 1 Prove that the fraction 21n+4 14n+3 ... Let α be the acute angel subtending, from A, that

IMO Shortlist 1993

Number Theory

2 A natural number n is said to have the property P, if, for all a, n2 divides an − 1 whenever ndivides an − 1.

a.) Show that every prime number n has property P.

b.) Show that there are infinitely many composite numbers n that possess property P.

3 Let a, b, n be positive integers, b > 1 and bn−1|a. Show that the representation of the numbera in the base b contains at least n digits different from zero.

4 Show that for any finite set S of distinct positive integers, we can find a set T S such thatevery member of T divides the sum of all the members of T .

Original Statement:

A finite set of (distinct) positive integers is called a DS-set if each of the integers divides thesum of them all. Prove that every finite set of positive integers is a subset of some DS-set.

5 Let S be the set of all pairs (m,n) of relatively prime positive integers m,n with n even andm < n. For s = (m,n) ∈ S write n = 2k · no where k, n0 are positive integers with n0 oddand define

f(s) = (n0,m + n− n0).

Prove that f is a function from S to S and that for each s = (m,n) ∈ S, there exists a positiveinteger t ≤ m+n+1

4 such thatf t(s) = s,

wheref t(s) = (f f · · · f)︸ ︷︷ ︸

t times

(s).

If m + n is a prime number which does not divide 2k − 1 for k = 1, 2, . . . ,m + n − 2, provethat the smallest value t which satisfies the above conditions is

[m+n+1

4

]where [x] denotes

the greatest integer ≤ x.

http://www.artofproblemsolving.com/This file was downloaded from the AoPS Math Olympiad Resources Page Page 8

Page 119: យុវសិស្ ឈឹ ទីុយ IMO Shortlist 1959-2009 · IMO Shortlist 1959 1 Prove that the fraction 21n+4 14n+3 ... Let α be the acute angel subtending, from A, that

IMO Shortlist 1994

Algebra

1 Let a0 = 1994 and an+1 = a2n

an+1 for each nonnegative integer n. Prove that 1994 − n is thegreatest integer less than or equal to an, 0 ≤ n ≤ 998

2 Let m and n be two positive integers. Let a1, a2, . . ., am be m different numbers from theset 1, 2, . . . , n such that for any two indices i and j with 1 ≤ i ≤ j ≤ m and ai + aj ≤ n,there exists an index k such that ai + aj = ak. Show that

a1 + a2 + ... + am

m≥ n + 1

2.

3 Let S be the set of all real numbers strictly greater than 1. Find all functions f : S → Ssatisfying the two conditions:

(a) f(x + f(y) + xf(y)) = y + f(x) + yf(x) for all x, y in S;

(b) f(x)x is strictly increasing on each of the two intervals −1 < x < 0 and 0 < x.

4 Let R denote the set of all real numbers and R+ the subset of all positive ones. Let α and βbe given elements in R, not necessarily distinct. Find all functions f : R+ 7→ R such that

f(x)f(y) = yαf(x

2

)+ xβf

(y

2

)∀x, y ∈ R+.

5 Let f(x) = x2+12x for x 6= 0. Define f (0)(x) = x and f (n)(x) = f(f (n−1)(x)) for all positive

integers n and x 6= 0. Prove that for all non-negative integers n and x 6= −1, 0, 1

f (n)(x)f (n+1)(x)

= 1 +1

f

((x+1x−1

)2n) .

http://www.artofproblemsolving.com/This file was downloaded from the AoPS Math Olympiad Resources Page Page 1

Page 120: យុវសិស្ ឈឹ ទីុយ IMO Shortlist 1959-2009 · IMO Shortlist 1959 1 Prove that the fraction 21n+4 14n+3 ... Let α be the acute angel subtending, from A, that

IMO Shortlist 1994

Combinatorics

1 Two players play alternately on a 5 × 5 board. The first player always enters a 1 into anempty square and the second player always enters a 0 into an empty square. When the boardis full, the sum of the numbers in each of the nine 3 × 3 squares is calculated and the firstplayer’s score is the largest such sum. What is the largest score the first player can make,regardless of the responses of the second player?

2 In a certain city, age is reckoned in terms of real numbers rather than integers. Every twocitizens x and x′ either know each other or do not know each other. Moreover, if they do not,then there exists a chain of citizens x = x0, x1, . . . , xn = x′ for some integer n ≥ 2 such thatxi−1 and xi know each other. In a census, all male citizens declare their ages, and there isat least one male citizen. Each female citizen provides only the information that her age isthe average of the ages of all the citizens she knows. Prove that this is enough to determineuniquely the ages of all the female citizens.

3 Peter has three accounts in a bank, each with an integral number of dollars. He is onlyallowed to transfer money from one account to another so that the amount of money in thelatter is doubled. Prove that Peter can always transfer all his money into two accounts. CanPeter always transfer all his money into one account?

4 There are n + 1 cells in a row labeled from 0 to n and n + 1 cards labeled from 0 to n. Thecards are arbitrarily placed in the cells, one per cell. The objective is to get card i into celli for each i. The allowed move is to find the smallest h such that cell h has a card with alabel k > h, pick up that card, slide the cards in cells h + 1, h + 2, ... , k one cell to the leftand to place card k in cell k. Show that at most 2n − 1 moves are required to get every cardinto the correct cell and that there is a unique starting position which requires 2n− 1 moves.[For example, if n = 2 and the initial position is 210, then we get 102, then 012, a total of 2moves.]

5 1994 girls are seated at a round table. Initially one girl holds n tokens. Each turn a girl whois holding more than one token passes one token to each of her neighbours.

a.) Show that if n < 1994, the game must terminate. b.) Show that if n = 1994 it cannotterminate.

6 Two players play alternatively on an infinite square grid. The first player puts a X in anempty cell and the second player puts a O in an empty cell. The first player wins if he gets11 adjacent Xs in a line, horizontally, vertically or diagonally. Show that the second playercan always prevent the first player from winning.

http://www.artofproblemsolving.com/This file was downloaded from the AoPS Math Olympiad Resources Page Page 2

Page 121: យុវសិស្ ឈឹ ទីុយ IMO Shortlist 1959-2009 · IMO Shortlist 1959 1 Prove that the fraction 21n+4 14n+3 ... Let α be the acute angel subtending, from A, that

IMO Shortlist 1994

7 Let n > 2. Show that there is a set of 2n− 1 points in the plane, no three collinear such thatno 2n form a convex 2n-gon.

http://www.artofproblemsolving.com/This file was downloaded from the AoPS Math Olympiad Resources Page Page 3

Page 122: យុវសិស្ ឈឹ ទីុយ IMO Shortlist 1959-2009 · IMO Shortlist 1959 1 Prove that the fraction 21n+4 14n+3 ... Let α be the acute angel subtending, from A, that

IMO Shortlist 1994

Geometry

1 C and D are points on a semicircle. The tangent at C meets the extended diameter of thesemicircle at B, and the tangent at D meets it at A, so that A and B are on opposite sidesof the center. The lines AC and BD meet at E. F is the foot of the perpendicular from Eto AB. Show that EF bisects angle CFD

2 ABCD is a quadrilateral with BC parallel to AD. M is the midpoint of CD, P is themidpoint of MA and Q is the midpoint of MB. The lines DP and CQ meet at N . Provethat N is inside the quadrilateral ABCD.

3 A circle C has two parallel tangents L′ andL”. A circle C ′ touches L′ at A and C at X. Acircle C” touches L” at B, C at Y and C ′ at Z. The lines AY and BX meet at Q. Showthat Q is the circumcenter of XY Z

4 Let ABC be an isosceles triangle with AB = AC. M is the midpoint of BC and O is thepoint on the line AM such that OB is perpendicular to AB. Q is an arbitrary point on BCdifferent from B and C. E lies on the line AB and F lies on the line AC such that E,Q,Fare distinct and collinear. Prove that OQ is perpendicular to EF if and only if QE = QF .

5 A circle C with center O. and a line L which does not touch circle C. OQ is perpendicular toL, Q is on L. P is on L, draw two tangents L1, L2 to circle C. QA,QB are perpendicular toL1, L2 respectively. (A on L1, B on L2). Prove that, line AB intersect QO at a fixed point.

Original formulation:

A line l does not meet a circle ω with center O. E is the point on l such that OE is perpen-dicular to l. M is any point on l other than E. The tangents from M to ω touch it at A andB. C is the point on MA such that EC is perpendicular to MA. D is the point on MB suchthat ED is perpendicular to MB. The line CD cuts OE at F. Prove that the location of Fis independent of that of M.

http://www.artofproblemsolving.com/This file was downloaded from the AoPS Math Olympiad Resources Page Page 4

Page 123: យុវសិស្ ឈឹ ទីុយ IMO Shortlist 1959-2009 · IMO Shortlist 1959 1 Prove that the fraction 21n+4 14n+3 ... Let α be the acute angel subtending, from A, that

IMO Shortlist 1994

Number Theory

1 M is a subset of 1, 2, 3, . . . , 15 such that the product of any three distinct elements of M isnot a square. Determine the maximum number of elements in M.

2 Find all ordered pairs (m,n) where m and n are positive integers such that n3+1mn−1 is an integer.

3 Show that there exists a set A of positive integers with the following property: for any infiniteset S of primes, there exist two positive integers m in A and n not in A, each of which is aproduct of k distinct elements of S for some k ≥ 2.

4 Define the sequences an, bn, cn as follows. a0 = k, b0 = 4, c0 = 1. If an is even then an+1 = an2 ,

bn+1 = 2bn, cn+1 = cn. If an is odd, then an+1 = an − bn2 − cn, bn+1 = bn, cn+1 = bn + cn.

Find the number of positive integers k < 1995 such that some an = 0.

5 For any positive integer k, let fk be the number of elements in the set k + 1, k + 2, . . . , 2kwhose base 2 representation contains exactly three 1s.

(a) Prove that for any positive integer m, there exists at least one positive integer k such thatf(k) = m.

(b) Determine all positive integers m for which there exists exactly one k with f(k) = m.

6 Define the sequence a1, a2, a3, ... as follows. a1 and a2 are coprime positive integers andan+2 = an+1an + 1. Show that for every m > 1 there is an n > m such that am

m divides ann.

Is it true that a1 must divide ann for some n > 1?

7 A wobbly number is a positive integer whose digits are alternately zero and non-zero withthe last digit non-zero (for example, 201). Find all positive integers which do not divide anywobbly number.

http://www.artofproblemsolving.com/This file was downloaded from the AoPS Math Olympiad Resources Page Page 5

Page 124: យុវសិស្ ឈឹ ទីុយ IMO Shortlist 1959-2009 · IMO Shortlist 1959 1 Prove that the fraction 21n+4 14n+3 ... Let α be the acute angel subtending, from A, that

IMO Shortlist 1995

Algebra

1 Let a, b, c be positive real numbers such that abc = 1. Prove that

1a3 (b + c)

+1

b3 (c + a)+

1c3 (a + b)

≥ 32.

2 Let a and b be non-negative integers such that ab ≥ c2, where c is an integer. Prove thatthere is a number n and integers x1, x2, . . . , xn, y1, y2, . . . , yn such that

n∑i=1

x2i = a,

n∑i=1

y2i = b, and

n∑i=1

xiyi = c.

3 Let n be an integer, n ≥ 3. Let a1, a2, . . . , an be real numbers such that 2 ≤ ai ≤ 3 fori = 1, 2, . . . , n. If s = a1 + a2 + . . . + an, prove that

a21 + a2

2 − a23

a1 + a2 − a3+

a22 + a2

3 − a24

a2 + a3 − a4+ . . . +

a2n + a2

1 − 2a2

an + a1 − a2≤ 2s− 2n.

4 Find all of the positive real numbers like x, y, z, such that :

1.) x + y + z = a + b + c

2.) 4xyz = a2x + b2y + c2z + abc

Proposed to Gazeta Matematica in the 80s by VASILE CRTOAJE and then by Titu An-dreescu to IMO 1995.

5 Let R be the set of real numbers. Does there exist a function f : R 7→ R which simultaneouslysatisfies the following three conditions?

(a) There is a positive number M such that ∀x : −M ≤ f(x) ≤ M. (b) The value of f(1) is1. (c) If x 6= 0, then

f

(x +

1x2

)= f(x) +

[f

(1x

)]2

6 Let n be an integer,n ≥ 3. Let x1, x2, . . . , xn be real numbers such that xi < xi+1 for 1 ≤ i ≤n− 1. Prove that

n(n− 1)2

∑i<j

xixj >

(n−1∑i=1

(n− i) · xi

n∑j=2

(j − 1) · xj

http://www.artofproblemsolving.com/This file was downloaded from the AoPS Math Olympiad Resources Page Page 1

Page 125: យុវសិស្ ឈឹ ទីុយ IMO Shortlist 1959-2009 · IMO Shortlist 1959 1 Prove that the fraction 21n+4 14n+3 ... Let α be the acute angel subtending, from A, that

IMO Shortlist 1995

Geometry

1 Let A,B, C, D be four distinct points on a line, in that order. The circles with diametersAC and BD intersect at X and Y . The line XY meets BC at Z. Let P be a point on theline XY other than Z. The line CP intersects the circle with diameter AC at C and M ,and the line BP intersects the circle with diameter BD at B and N . Prove that the linesAM,DN,XY are concurrent.

2 Let A,B and C be non-collinear points. Prove that there is a unique point X in the plane ofABC such that

XA2 + XB2 + AB2 = XB2 + XC2 + BC2 = XC2 + XA2 + CA2.

3 The incircle of triangle 4ABC touches the sides BC, CA, AB at D,E, F respectively. X isa point inside triangle of 4ABC such that the incircle of triangle 4XBC touches BC at D,and touches CX and XB at Y and Z respectively. Show that E,F, Z, Y are concyclic.

4 An acute triangle ABC is given. Points A1 and A2 are taken on the side BC (with A2 betweenA1 and C), B1 and B2 on the side AC (with B2 between B1 and A), and C1 and C2 on theside AB (with C2 between C1 and B) so that

∠AA1A2 = ∠AA2A1 = ∠BB1B2 = ∠BB2B1 = ∠CC1C2 = ∠CC2C1.

The lines AA1, BB1, and CC1 bound a triangle, and the lines AA2, BB2, and CC2 bound asecond triangle. Prove that all six vertices of these two triangles lie on a single circle.

5 Let ABCDEF be a convex hexagon with AB = BC = CD and DE = EF = FA, such that∠BCD = ∠EFA = π

3 . Suppose G and H are points in the interior of the hexagon such that∠AGB = ∠DHE = 2π

3 . Prove that AG + GB + GH + DH + HE ≥ CF .

6 Let A1A2A3A4 be a tetrahedron, G its centroid, and A′1, A

′2, A

′3, and A′

4 the points where thecircumsphere of A1A2A3A4 intersects GA1, GA2, GA3, and GA4, respectively. Prove that

GA1 ·GA2 ·GA3 ·GA·4 ≤ GA′1 ·GA′

2 ·GA′3 ·GA′

4

and

1GA′

1

+1

GA′2

+1

GA′3

+1

GA′4

≤ 1GA1

+1

GA2+

1GA3

+1

GA4.

http://www.artofproblemsolving.com/This file was downloaded from the AoPS Math Olympiad Resources Page Page 2

Page 126: យុវសិស្ ឈឹ ទីុយ IMO Shortlist 1959-2009 · IMO Shortlist 1959 1 Prove that the fraction 21n+4 14n+3 ... Let α be the acute angel subtending, from A, that

IMO Shortlist 1995

7 Let ABCD be a convex quadrilateral and O a point inside it. Let the parallels to the linesBC, AB, DA, CD through the point O meet the sides AB, BC, CD, DA of the quadrilateralABCD at the points E, F, G, H, respectively. Then, prove that

√|AHOE| +

√|CFOG| ≤√

|ABCD|, where |P1P2...Pn| is an abbreviation for the non-directed area of an arbitrarypolygon P1P2...Pn.

8 Suppose that ABCD is a cyclic quadrilateral. Let E = AC∩BD and F = AB∩CD. Denoteby H1 and H2 the orthocenters of triangles EAD and EBC, respectively. Prove that thepoints F , H1, H2 are collinear.

Original formulation:

Let ABC be a triangle. A circle passing through B and C intersects the sides AB and ACagain at C ′ and B′, respectively. Prove that B′, CC ′ and HH ′ are concurrent, where H andH ′ are the orthocentres of triangles ABC and AB′C ′ respectively.

http://www.artofproblemsolving.com/This file was downloaded from the AoPS Math Olympiad Resources Page Page 3

Page 127: យុវសិស្ ឈឹ ទីុយ IMO Shortlist 1959-2009 · IMO Shortlist 1959 1 Prove that the fraction 21n+4 14n+3 ... Let α be the acute angel subtending, from A, that

IMO Shortlist 1995

NT, Combs

1 Let k be a positive integer. Show that there are infinitely many perfect squares of the formn · 2k − 7 where n is a positive integer.

2 Let Z denote the set of all integers. Prove that for any integers A and B, one can find aninteger C for which M1 = x2 + Ax + B : x ∈ Z and M2 = 2x2 + 2x + C : x ∈ Z do notintersect.

3 Determine all integers n > 3 for which there exist n points A1, · · · , An in the plane, no threecollinear, and real numbers r1, · · · , rn such that for 1 ≤ i < j < k ≤ n, the area of 4AiAjAk

is ri + rj + rk.

4 Find all x, y and z in positive integer: z + y2 + x3 = xyz and x = gcd(y, z).

5 At a meeting of 12k people, each person exchanges greetings with exactly 3k + 6 others. Forany two people, the number who exchange greetings with both is the same. How many peopleare at the meeting?

6 Let p be an odd prime number. How many p-element subsets A of 1, 2, · · · 2p are there,the sum of whose elements is divisible by p?

7 Does there exist an integer n > 1 which satisfies the following condition? The set of positiveintegers can be partitioned into n nonempty subsets, such that an arbitrary sum of n − 1integers, one taken from each of any n− 1 of the subsets, lies in the remaining subset.

8 Let p be an odd prime. Determine positive integers x and y for which x ≤ y and√

2p−√

x−√yis non-negative and as small as possible.

http://www.artofproblemsolving.com/This file was downloaded from the AoPS Math Olympiad Resources Page Page 4

Page 128: យុវសិស្ ឈឹ ទីុយ IMO Shortlist 1959-2009 · IMO Shortlist 1959 1 Prove that the fraction 21n+4 14n+3 ... Let α be the acute angel subtending, from A, that

IMO Shortlist 1995

Sequences

1 Does there exist a sequence F (1), F (2), F (3), . . . of non-negative integers that simultaneouslysatisfies the following three conditions?

(a) Each of the integers 0, 1, 2, . . . occurs in the sequence. (b) Each positive integer occursin the sequence infinitely often. (c) For any n ≥ 2,

F (F (n163)) = F (F (n)) + F (F (361)).

2 Find the maximum value of x0 for which there exists a sequence x0, x1 · · · , x1995 of positivereals with x0 = x1995, such that

xi−1 +2

xi−1= 2xi +

1xi

,

for all i = 1, · · · , 1995.

3 For an integer x ≥ 1, let p(x) be the least prime that does not divide x, and define q(x) tobe the product of all primes less than p(x). In particular, p(1) = 2. For x having p(x) = 2,define q(x) = 1. Consider the sequence x0, x1, x2, . . . defined by x0 = 1 and

xn+1 =xnp(xn)q(xn)

for n ≥ 0. Find all n such that xn = 1995.

4 Suppose that x1, x2, x3, . . . are positive real numbers for which

xnn =

n−1∑j=0

xjn

for n = 1, 2, 3, . . . Prove that ∀n,

2− 12n−1

≤ xn < 2− 12n

.

5 For positive integers n, the numbers f(n) are defined inductively as follows: f(1) = 1, andfor every positive integer n, f(n+1) is the greatest integer m such that there is an arithmeticprogression of positive integers a1 < a2 < . . . < am = n for which

f(a1) = f(a2) = . . . = f(am).

Prove that there are positive integers a and b such that f(an + b) = n + 2 for every positiveinteger n.

http://www.artofproblemsolving.com/This file was downloaded from the AoPS Math Olympiad Resources Page Page 5

Page 129: យុវសិស្ ឈឹ ទីុយ IMO Shortlist 1959-2009 · IMO Shortlist 1959 1 Prove that the fraction 21n+4 14n+3 ... Let α be the acute angel subtending, from A, that

IMO Shortlist 1995

6 Let N denote the set of all positive integers. Prove that there exists a unique functionf : N 7→ N satisfying

f(m + f(n)) = n + f(m + 95)

for all m and n in N. What is the value of∑19

k=1 f(k)?

http://www.artofproblemsolving.com/This file was downloaded from the AoPS Math Olympiad Resources Page Page 6

Page 130: យុវសិស្ ឈឹ ទីុយ IMO Shortlist 1959-2009 · IMO Shortlist 1959 1 Prove that the fraction 21n+4 14n+3 ... Let α be the acute angel subtending, from A, that

IMO Shortlist 1996

Algebra

1 Suppose that a, b, c > 0 such that abc = 1. Prove that

ab

ab + a5 + b5+

bc

bc + b5 + c5+

ca

ca + c5 + a5≤ 1.

2 Let a1 ≥ a2 ≥ . . . ≥ an be real numbers such that for all integers k > 0,

ak1 + ak

2 + . . . + akn ≥ 0.

Let p = max|a1|, . . . , |an|. Prove that p = a1 and that

(x− a1) · (x− a2) · · · (x− an) ≤ xn − an1

for all x > a1.

3 Let a > 2 be given, and starting a0 = 1, a1 = a define recursively:

an+1 =(

a2n

a2n−1

− 2)· an.

Show that for all integers k > 0, we have:∑k

i=01ai

< 12 · (2 + a−

√a2 − 4).

4 Let a1, a2...an be non-negative reals, not all zero. Show that that (a) The polynomial p(x) =xn − a1x

n−1 + ...− an−1x− an has preceisely 1 positive real root R. (b) let A =∑n

i=1 ai andB =

∑ni=1 iai. Show that AA ≤ RB.

5 Let P (x) be the real polynomial function, P (x) = ax3 + bx2 + cx+d. Prove that if |P (x)| ≤ 1for all x such that |x| ≤ 1, then

|a|+ |b|+ |c|+ |d| ≤ 7.

6 Let n be an even positive integer. Prove that there exists a positive inter k such that

k = f(x) · (x + 1)n + g(x) · (xn + 1)

for some polynomials f(x), g(x) having integer coefficients. If k0 denotes the least such k,determine k0 as a function of n, i.e. show that k0 = 2q where q is the odd integer determinedby n = q · 2r, r ∈ N.

Note: This is variant A6’ of the three variants given for this problem.

http://www.artofproblemsolving.com/This file was downloaded from the AoPS Math Olympiad Resources Page Page 1

Page 131: យុវសិស្ ឈឹ ទីុយ IMO Shortlist 1959-2009 · IMO Shortlist 1959 1 Prove that the fraction 21n+4 14n+3 ... Let α be the acute angel subtending, from A, that

IMO Shortlist 1996

7 Let f be a function from the set of real numbers R into itself such for all x ∈ R, we have|f(x)| ≤ 1 and

f

(x +

1342

)+ f(x) = f

(x +

16

)+ f

(x +

17

).

Prove that f is a periodic function (that is, there exists a non-zero real number c suchf(x + c) = f(x) for all x ∈ R).

8 Let N0 denote the set of nonnegative integers. Find all functions f from N0 to itself such that

f(m + f(n)) = f(f(m)) + f(n) for all m,n ∈ N0.

9 Let the sequence a(n), n = 1, 2, 3, . . . be generated as follows with a(1) = 0, and for n > 1 :

a(n) = a(⌊n

2

⌋)+ (−1)

n(n+1)2 .

1.) Determine the maximum and minimum value of a(n) over n ≤ 1996 and find all n ≤ 1996for which these extreme values are attained.

2.) How many terms a(n), n ≤ 1996, are equal to 0?

http://www.artofproblemsolving.com/This file was downloaded from the AoPS Math Olympiad Resources Page Page 2

Page 132: យុវសិស្ ឈឹ ទីុយ IMO Shortlist 1959-2009 · IMO Shortlist 1959 1 Prove that the fraction 21n+4 14n+3 ... Let α be the acute angel subtending, from A, that

IMO Shortlist 1996

Combinatorics

1 We are given a positive integer r and a rectangular board ABCD with dimensions AB =20, BC = 12. The rectangle is divided into a grid of 20×12 unit squares. The following movesare permitted on the board: one can move from one square to another only if the distancebetween the centers of the two squares is

√r. The task is to find a sequence of moves leading

from the square with A as a vertex to the square with B as a vertex.

(a) Show that the task cannot be done if r is divisible by 2 or 3.

(b) Prove that the task is possible when r = 73.

(c) Can the task be done when r = 97?

2 A square (n − 1) × (n − 1) is divided into (n − 1)2 unit squares in the usual manner. Eachof the n2 vertices of these squares is to be coloured red or blue. Find the number of differentcolourings such that each unit square has exactly two red vertices. (Two colouring schemseare regarded as different if at least one vertex is coloured differently in the two schemes.)

3 Let k, m, n be integers such that 1 < n ≤ m−1 ≤ k. Determine the maximum size of a subsetS of the set 1, 2, 3, . . . , k − 1, k such that no n distinct elements of S add up to m.

4 Determine whether or nor there exist two disjoint infinite sets A and B of points in the planesatisfying the following conditions:

a.) No three points in A∪B are collinear, and the distance between any two points in A∪Bis at least 1.

b.) There is a point of A in any triangle whose vertices are in B, and there is a point of B inany triangle whose vertices are in A.

5 Let p, q, n be three positive integers with p + q < n. Let (x0, x1, · · · , xn) be an (n + 1)-tupleof integers satisfying the following conditions :

(a) x0 = xn = 0, and

(b) For each i with 1 ≤ i ≤ n, either xi − xi−1 = p or xi − xi−1 = − q.

Show that there exist indices i < j with (i, j) 6= (0, n), such that xi = xj .

6 A finite number of coins are placed on an infinite row of squares. A sequence of moves isperformed as follows: at each stage a square containing more than one coin is chosen. Twocoins are taken from this square; one of them is placed on the square immediately to the leftwhile the other is placed on the square immediately to the right of the chosen square. Thesequence terminates if at some point there is at most one coin on each square. Given some

http://www.artofproblemsolving.com/This file was downloaded from the AoPS Math Olympiad Resources Page Page 3

Page 133: យុវសិស្ ឈឹ ទីុយ IMO Shortlist 1959-2009 · IMO Shortlist 1959 1 Prove that the fraction 21n+4 14n+3 ... Let α be the acute angel subtending, from A, that

IMO Shortlist 1996

initial configuration, show that any legal sequence of moves will terminate after the samenumber of steps and with the same final configuration.

7 let V be a finitive set and g and f be two injective surjective functions from V toV .let T andS be two sets such that they are defined as following” S = w ∈ V : f(f(w)) = g(g(w))T = w ∈ V : f(g(w)) = g(f(w)) we know that S∪T = V , prove: for each w ∈ V : f(w) ∈ Sif and only if g(w) ∈ S

http://www.artofproblemsolving.com/This file was downloaded from the AoPS Math Olympiad Resources Page Page 4

Page 134: យុវសិស្ ឈឹ ទីុយ IMO Shortlist 1959-2009 · IMO Shortlist 1959 1 Prove that the fraction 21n+4 14n+3 ... Let α be the acute angel subtending, from A, that

IMO Shortlist 1996

Geometry

1 Let ABC be a triangle, and H its orthocenter. Let P be a point on the circumcircle oftriangle ABC (distinct from the vertices A, B, C), and let E be the foot of the altitude oftriangle ABC from the vertex B. Let the parallel to the line BP through the point A meetthe parallel to the line AP through the point B at a point Q. Let the parallel to the line CPthrough the point A meet the parallel to the line AP through the point C at a point R. Thelines HR and AQ intersect at some point X. Prove that the lines EX and AP are parallel.

2 Let P be a point inside a triangle ABC such that

∠APB − ∠ACB = ∠APC − ∠ABC.

Let D, E be the incenters of triangles APB, APC, respectively. Show that the lines AP ,BD, CE meet at a point.

3 Let O be the circumcenter and H the orthocenter of an acute-angled triangle ABC such thatBC > CA. Let F be the foot of the altitude CH of triangle ABC. The perpendicular to theline OF at the point F intersects the line AC at P . Prove that ]FHP = ]BAC.

4 Let ABC be an equilateral triangle and let P be a point in its interior. Let the lines AP ,BP , CP meet the sides BC, CA, AB at the points A1, B1, C1, respectively. Prove that

A1B1 ·B1C1 · C1A1 ≥ A1B ·B1C · C1A.

5 Let ABCDEF be a convex hexagon such that AB is parallel to DE, BC is parallel to EF , andCD is parallel to FA. Let RA, RC , RE denote the circumradii of triangles FAB,BCD,DEF ,respectively, and let P denote the perimeter of the hexagon. Prove that

RA + RC + RE ≥ P

2.

6 Let the sides of two rectangles be a, b and c, d, respectively, with a < c ≤ d < b andab < cd. Prove that the first rectangle can be placed within the second one if and only if

(b2 − a2

)2 ≤ (bc− ad)2 + (bd− ac)2 .

7 Let ABC be an acute triangle with circumcenter O and circumradius R. AO meets thecircumcircle of BOC at A′, BO meets the circumcircle of COA at B′ and CO meets thecircumcircle of AOB at C ′. Prove that

OA′ ·OB′ ·OC ′ ≥ 8R3.

http://www.artofproblemsolving.com/This file was downloaded from the AoPS Math Olympiad Resources Page Page 5

Page 135: យុវសិស្ ឈឹ ទីុយ IMO Shortlist 1959-2009 · IMO Shortlist 1959 1 Prove that the fraction 21n+4 14n+3 ... Let α be the acute angel subtending, from A, that

IMO Shortlist 1996

Sorry if this has been posted before since this is a very classical problem, but I failed to findit with the search-function.

8 Let ABCD be a convex quadrilateral, and let RA, RB, RC , RD denote the circumradii of thetriangles DAB, ABC, BCD, CDA, respectively. Prove that RA + RC > RB + RD if and onlyif ∠A + ∠C > ∠B + ∠D.

9 In the plane, consider a point X and a polygon F (which is not necessarily convex). Let pdenote the perimeter of F , let d be the sum of the distances from the point X to the verticesof F , and let h be the sum of the distances from the point X to the sidelines of F . Provethat d2 − h2 ≥ p2

4 .

http://www.artofproblemsolving.com/This file was downloaded from the AoPS Math Olympiad Resources Page Page 6

Page 136: យុវសិស្ ឈឹ ទីុយ IMO Shortlist 1959-2009 · IMO Shortlist 1959 1 Prove that the fraction 21n+4 14n+3 ... Let α be the acute angel subtending, from A, that

IMO Shortlist 1996

Number Theory

1 Four integers are marked on a circle. On each step we simultaneously replace each numberby the difference between this number and next number on the circle, moving in a clockwisedirection; that is, the numbers a, b, c, d are replaced by a− b, b− c, c− d, d− a. Is it possibleafter 1996 such to have numbers a, b, c, d such the numbers |bc − ad|, |ac − bd|, |ab − cd| areprimes?

2 The positive integers a and b are such that the numbers 15a + 16b and 16a − 15b are bothsquares of positive integers. What is the least possible value that can be taken on by thesmaller of these two squares?

3 A finite sequence of integers a0, a1, . . . , an is called quadratic if for each i in the set 1, 2 . . . , nwe have the equality |ai − ai−1| = i2.

a.) Prove that any two integers b and c, there exists a natural number n and a quadraticsequence with a0 = b and an = c.

b.) Find the smallest natural number n for which there exists a quadratic sequence witha0 = 0 and an = 1996.

4 Find all positive integers a and b for which⌊a2

b

⌋+

⌊b2

a

⌋=

⌊a2 + b2

ab

⌋+ ab.

5 Show that there exists a bijective function f : N0 → N0 such that for all m,n ∈ N0:

f(3mn + m + n) = 4f(m)f(n) + f(m) + f(n).

http://www.artofproblemsolving.com/This file was downloaded from the AoPS Math Olympiad Resources Page Page 7

Page 137: យុវសិស្ ឈឹ ទីុយ IMO Shortlist 1959-2009 · IMO Shortlist 1959 1 Prove that the fraction 21n+4 14n+3 ... Let α be the acute angel subtending, from A, that

IMO Shortlist 1997

1 In the plane the points with integer coordinates are the vertices of unit squares. The squaresare coloured alternately black and white (as on a chessboard). For any pair of positive integersm and n, consider a right-angled triangle whose vertices have integer coordinates and whoselegs, of lengths m and n, lie along edges of the squares. Let S1 be the total area of the blackpart of the triangle and S2 be the total area of the white part. Let f(m,n) = |S1 − S2|.a) Calculate f(m,n) for all positive integers m and n which are either both even or both odd.

b) Prove that f(m,n) ≤ 12 maxm,n for all m and n.

c) Show that there is no constant C ∈ R such that f(m,n) < C for all m and n.

2 Let R1, R2, . . . be the family of finite sequences of positive integers defined by the followingrules: R1 = (1), and if Rn1 = (x1, . . . , xs), then

Rn = (1, 2, . . . , x1, 1, 2, . . . , x2, . . . , 1, 2, . . . , xs, n).

For example, R2 = (1, 2), R3 = (1, 1, 2, 3), R4 = (1, 1, 1, 2, 1, 2, 3, 4). Prove that if n > 1, thenthe kth term from the left in Rn is equal to 1 if and only if the kth term from the right inRn is different from 1.

3 For each finite set U of nonzero vectors in the plane we define l(U) to be the length of thevector that is the sum of all vectors in U. Given a finite set V of nonzero vectors in theplane, a subset B of V is said to be maximal if l(B) is greater than or equal to l(A) for eachnonempty subset A of V.

(a) Construct sets of 4 and 5 vectors that have 8 and 10 maximal subsets respectively.

(b) Show that, for any set V consisting of n ≥ 1 vectors the number of maximal subsets isless than or equal to 2n.

4 An n×n matrix whose entries come from the set S = 1, 2, . . . , 2n−1 is called a silver matrixif, for each i = 1, 2, . . . , n, the i-th row and the i-th column together contain all elements ofS. Show that:

(a) there is no silver matrix for n = 1997;

(b) silver matrices exist for infinitely many values of n.

5 Let ABCD be a regular tetrahedron and M,N distinct points in the planes ABC and ADCrespectively. Show that the segments MN, BN, MD are the sides of a triangle.

6 (a) Let n be a positive integer. Prove that there exist distinct positive integers x, y, z suchthat

xn−1 + yn = zn+1.

http://www.artofproblemsolving.com/This file was downloaded from the AoPS Math Olympiad Resources Page Page 1

Page 138: យុវសិស្ ឈឹ ទីុយ IMO Shortlist 1959-2009 · IMO Shortlist 1959 1 Prove that the fraction 21n+4 14n+3 ... Let α be the acute angel subtending, from A, that

IMO Shortlist 1997

(b) Let a, b, c be positive integers such that a and b are relatively prime and c is relativelyprime either to a or to b. Prove that there exist infinitely many triples (x, y, z) of distinctpositive integers x, y, z such that

xa + yb = zc.

7 The lengths of the sides of a convex hexagon ABCDEF satisfy AB = BC, CD = DE,EF = FA. Prove that:

BC

BE+

DE

DA+

FA

FC≥ 3

2.

8 It is known that ∠BAC is the smallest angle in the triangle ABC. The points B and C dividethe circumcircle of the triangle into two arcs. Let U be an interior point of the arc betweenB and C which does not contain A. The perpendicular bisectors of AB and AC meet theline AU at V and W , respectively. The lines BV and CW meet at T .

Show that AU = TB + TC.

Alternative formulation:

Four different points A,B, C, D are chosen on a circle Γ such that the triangle BCD is notright-angled. Prove that:

(a) The perpendicular bisectors of AB and AC meet the line AD at certain points W and V,respectively, and that the lines CV and BW meet at a certain point T.

(b) The length of one of the line segments AD,BT, and CT is the sum of the lengths of theother two.

9 Let A1A2A3 be a non-isosceles triangle with incenter I. Let Ci, i = 1, 2, 3, be the smaller circlethrough I tangent to AiAi+1 and AiAi+2 (the addition of indices being mod 3). Let Bi, i =1, 2, 3, be the second point of intersection of Ci+1 and Ci+2. Prove that the circumcentres ofthe triangles A1B1I,A2B2I,A3B3I are collinear.

10 Find all positive integers k for which the following statement is true: If F (x) is a polynomialwith integer coeffiecients satisfying the condition 0 ≤ F (c) ≤ k for each c0, 1, . . . , k + 1,then F (0) = F (1) = . . . = F (k + 1).

11 Let P (x) be a polynomial with real coefficients such that P (x) > 0 for all x ≥ 0. Prove thatthere exists a positive integer n such that (1 + x)n · P (x) is a polynomial with nonnegativecoefficients.

12 Let p be a prime number and f an integer polynomial of degree d such that f(0) = 0, f(1) = 1and f(n) is congruent to 0 or 1 modulo p for every integer n. Prove that d ≥ p− 1.

http://www.artofproblemsolving.com/This file was downloaded from the AoPS Math Olympiad Resources Page Page 2

Page 139: យុវសិស្ ឈឹ ទីុយ IMO Shortlist 1959-2009 · IMO Shortlist 1959 1 Prove that the fraction 21n+4 14n+3 ... Let α be the acute angel subtending, from A, that

IMO Shortlist 1997

13 In town A, there are n girls and n boys, and each girl knows each boy. In town B, there are ngirls g1, g2, . . . , gn and 2n−1 boys b1, b2, . . . , b2n−1. The girl gi, i = 1, 2, . . . , n, knows the boysb1, b2, . . . , b2i−1, and no others. For all r = 1, 2, . . . , n, denote by A(r), B(r) the number ofdifferent ways in which r girls from town A, respectively town B, can dance with r boys fromtheir own town, forming r pairs, each girl with a boy she knows. Prove that A(r) = B(r) foreach r = 1, 2, . . . , n.

14 Let b, m, n be positive integers such that b > 1 and m 6= n. Prove that if bm − 1 and bn − 1have the same prime divisors, then b + 1 is a power of 2.

15 An infinite arithmetic progression whose terms are positive integers contains the square of aninteger and the cube of an integer. Show that it contains the sixth power of an integer.

16 In an acute-angled triangle ABC, let AD,BE be altitudes and AP,BQ internal bisectors.Denote by I and O the incenter and the circumcentre of the triangle, respectively. Prove thatthe points D,E, and I are collinear if and only if the points P,Q, and O are collinear.

17 Find all pairs (a, b) of positive integers that satisfy the equation: ab2 = ba.

18 The altitudes through the vertices A,B, C of an acute-angled triangle ABC meet the oppositesides at D,E, F, respectively. The line through D parallel to EF meets the lines AC and ABat Q and R, respectively. The line EF meets BC at P. Prove that the circumcircle of thetriangle PQR passes through the midpoint of BC.

19 Let a1 ≥ · · · ≥ an ≥ an+1 = 0 be real numbers. Show that√√√√ n∑k=1

ak ≤n∑

k=1

√k(√

ak −√

ak+1).

Proposed by Romania

20 Let ABC be a triangle. D is a point on the side (BC). The line AD meets the circumcircleagain at X. P is the foot of the perpendicular from X to AB, and Q is the foot of theperpendicular from X to AC. Show that the line PQ is a tangent to the circle on diameterXD if and only if AB = AC.

21 Let x1, x2, . . ., xn be real numbers satisfying the conditions:|x1 + x2 + · · ·+ xn| = 1

|xi| ≤ n + 12

for i = 1, 2, . . . , n.

http://www.artofproblemsolving.com/This file was downloaded from the AoPS Math Olympiad Resources Page Page 3

Page 140: យុវសិស្ ឈឹ ទីុយ IMO Shortlist 1959-2009 · IMO Shortlist 1959 1 Prove that the fraction 21n+4 14n+3 ... Let α be the acute angel subtending, from A, that

IMO Shortlist 1997

Show that there exists a permutation y1, y2, . . ., yn of x1, x2, . . ., xn such that

|y1 + 2y2 + · · ·+ nyn| ≤n + 1

2.

22 Does there exist functions f, g : R → R such that f(g(x)) = x2 and g(f(x)) = xk for all realnumbers x

a) if k = 3?

b) if k = 4?

23 Let ABCD be a convex quadrilateral. The diagonals AC and BD intersect at K. Show thatABCD is cyclic if and only if AK sinA + CK sinC = BK sinB + DK sinD.

24 For each positive integer n, let f(n) denote the number of ways of representing n as a sumof powers of 2 with nonnegative integer exponents. Representations which differ only in theordering of their summands are considered to be the same. For instance, f(4) = 4, becausethe number 4 can be represented in the following four ways: 4; 2+2; 2+1+1; 1+1+1+1.

Prove that, for any integer n ≥ 3 we have 2n2

4 < f(2n) < 2n2

2 .

25 Let X, Y, Z be the midpoints of the small arcs BC, CA, AB respectively (arcs of the circum-circle of ABC). M is an arbitrary point on BC, and the parallels through M to the internalbisectors of ∠B,∠C cut the external bisectors of ∠C,∠B in N,P respectively. Show thatXM,Y N,ZP concur.

26 For every integer n ≥ 2 determine the minimum value that the sum∑n

i=0 ai can take fornonnegative numbers a0, a1, . . . , an satisfying the condition a0 = 1, ai ≤ ai+1 + ai+2 fori = 0, . . . , n− 2.

http://www.artofproblemsolving.com/This file was downloaded from the AoPS Math Olympiad Resources Page Page 4

Page 141: យុវសិស្ ឈឹ ទីុយ IMO Shortlist 1959-2009 · IMO Shortlist 1959 1 Prove that the fraction 21n+4 14n+3 ... Let α be the acute angel subtending, from A, that

IMO Shortlist 1998

Algebra

1 Let a1, a2, . . . , an be positive real numbers such that a1 + a2 + · · ·+ an < 1. Prove that

a1a2 · · · an [1− (a1 + a2 + · · ·+ an)](a1 + a2 + · · ·+ an)(1− a1)(1− a2) · · · (1− an)

≤ 1nn+1

.

2 Let r1, r2, . . . , rn be real numbers greater than or equal to 1. Prove that

1r1 + 1

+1

r2 + 1+ · · ·+ 1

rn + 1≥ n

n√

r1r2 · · · rn + 1.

3 Let x, y and z be positive real numbers such that xyz = 1. Prove that

x3

(1 + y)(1 + z)+

y3

(1 + z)(1 + x)+

z3

(1 + x)(1 + y)≥ 3

4.

4 For any two nonnegative integers n and k satisfying n ≥ k, we define the number c(n, k) asfollows:

- c (n, 0) = c (n, n) = 1 for all n ≥ 0;

- c (n + 1, k) = 2kc (n, k) + c (n, k − 1) for n ≥ k ≥ 1.

Prove that c (n, k) = c (n, n− k) for all n ≥ k ≥ 0.

5 Determine the least possible value of f(1998), where f is a function from the set N of positiveintegers into itself such that for all m,n ∈ N,

f(n2f(m)

)= m [f(n)]2 .

http://www.artofproblemsolving.com/This file was downloaded from the AoPS Math Olympiad Resources Page Page 1

Page 142: យុវសិស្ ឈឹ ទីុយ IMO Shortlist 1959-2009 · IMO Shortlist 1959 1 Prove that the fraction 21n+4 14n+3 ... Let α be the acute angel subtending, from A, that

IMO Shortlist 1998

Combinatorics

1 A rectangular array of numbers is given. In each row and each column, the sum of all numbersis an integer. Prove that each nonintegral number x in the array can be changed into eitherdxe or bxc so that the row-sums and column-sums remain unchanged. (Note that dxe is theleast integer greater than or equal to x, while bxc is the greatest integer less than or equal tox.)

2 Let n be an integer greater than 2. A positive integer is said to be attainable if it is 1 or canbe obtained from 1 by a sequence of operations with the following properties:

1.) The first operation is either addition or multiplication.

2.) Thereafter, additions and multiplications are used alternately.

3.) In each addition, one can choose independently whether to add 2 or n

4.) In each multiplication, one can choose independently whether to multiply by 2 or by n.

A positive integer which cannot be so obtained is said to be unattainable.

a.) Prove that if n ≥ 9, there are infinitely many unattainable positive integers.

b.) Prove that if n = 3, all positive integers except 7 are attainable.

3 Cards numbered 1 to 9 are arranged at random in a row. In a move, one may choose anyblock of consecutive cards whose numbers are in ascending or descending order, and switchthe block around. For example, 9 1 6 5 3 2 7 4 8 may be changed to 91 3 5 6 2 7 4 8. Provethat in at most 12 moves, one can arrange the 9 cards so that their numbers are in ascendingor descending order.

4 Let U = 1, 2, . . . , n, where n ≥ 3. A subset S of U is said to be split by an arrangement ofthe elements of U if an element not in S occurs in the arrangement somewhere between twoelements of S. For example, 13542 splits 1, 2, 3 but not 3, 4, 5. Prove that for any n− 2subsets of U , each containing at least 2 and at most n− 1 elements, there is an arrangementof the elements of U which splits all of them.

5 In a contest, there are m candidates and n judges, where n ≥ 3 is an odd integer. Eachcandidate is evaluated by each judge as either pass or fail. Suppose that each pair of judgesagrees on at most k candidates. Prove that

k

m≥ n− 1

2n.

http://www.artofproblemsolving.com/This file was downloaded from the AoPS Math Olympiad Resources Page Page 2

Page 143: យុវសិស្ ឈឹ ទីុយ IMO Shortlist 1959-2009 · IMO Shortlist 1959 1 Prove that the fraction 21n+4 14n+3 ... Let α be the acute angel subtending, from A, that

IMO Shortlist 1998

6 Ten points are marked in the plane so that no three of them lie on a line. Each pair of pointsis connected with a segment. Each of these segments is painted with one of k colors, in sucha way that for any k of the ten points, there are k segments each joining two of them and notwo being painted with the same color. Determine all integers k, 1 ≤ k ≤ 10, for which thisis possible.

7 A solitaire game is played on an m×n rectangular board, using mn markers which are whiteon one side and black on the other. Initially, each square of the board contains a markerwith its white side up, except for one corner square, which contains a marker with its blackside up. In each move, one may take away one marker with its black side up, but must thenturn over all markers which are in squares having an edge in common with the square of theremoved marker. Determine all pairs (m,n) of positive integers such that all markers can beremoved from the board.

http://www.artofproblemsolving.com/This file was downloaded from the AoPS Math Olympiad Resources Page Page 3

Page 144: យុវសិស្ ឈឹ ទីុយ IMO Shortlist 1959-2009 · IMO Shortlist 1959 1 Prove that the fraction 21n+4 14n+3 ... Let α be the acute angel subtending, from A, that

IMO Shortlist 1998

Geometry

1 A convex quadrilateral ABCD has perpendicular diagonals. The perpendicular bisectors ofthe sides AB and CD meet at a unique point P inside ABCD. Prove that the quadrilateralABCD is cyclic if and only if triangles ABP and CDP have equal areas.

2 Let ABCD be a cyclic quadrilateral. Let E and F be variable points on the sides AB andCD, respectively, such that AE : EB = CF : FD. Let P be the point on the segment EFsuch that PE : PF = AB : CD. Prove that the ratio between the areas of triangles APDand BPC does not depend on the choice of E and F .

3 Let I be the incenter of triangle ABC. Let K, L and M be the points of tangency of theincircle of ABC with AB,BC and CA, respectively. The line t passes through B and isparallel to KL. The lines MK and ML intersect t at the points R and S. Prove that ∠RISis acute.

4 Let M and N be two points inside triangle ABC such that

∠MAB = ∠NAC and ∠MBA = ∠NBC.

Prove thatAM ·AN

AB ·AC+

BM ·BN

BA ·BC+

CM · CN

CA · CB= 1.

5 Let ABC be a triangle, H its orthocenter, O its circumcenter, and R its circumradius. LetD be the reflection of the point A across the line BC, let E be the reflection of the point Bacross the line CA, and let F be the reflection of the point C across the line AB. Prove thatthe points D, E and F are collinear if and only if OH = 2R.

6 Let ABCDEF be a convex hexagon such that ∠B + ∠D + ∠F = 360 and

AB

BC· CD

DE· EF

FA= 1.

Prove thatBC

CA· AE

EF· FD

DB= 1.

7 Let ABC be a triangle such that ∠ACB = 2∠ABC. Let D be the point on the side BC suchthat CD = 2BD. The segment AD is extended to E so that AD = DE. Prove that

∠ECB + 180 = 2∠EBC.

[hide=”comment”] Edited by Orl.

http://www.artofproblemsolving.com/This file was downloaded from the AoPS Math Olympiad Resources Page Page 4

Page 145: យុវសិស្ ឈឹ ទីុយ IMO Shortlist 1959-2009 · IMO Shortlist 1959 1 Prove that the fraction 21n+4 14n+3 ... Let α be the acute angel subtending, from A, that

IMO Shortlist 1998

8 Let ABC be a triangle such that ∠A = 90 and ∠B < ∠C. The tangent at A to thecircumcircle ω of triangle ABC meets the line BC at D. Let E be the reflection of A in theline BC, let X be the foot of the perpendicular from A to BE, and let Y be the midpoint ofthe segment AX. Let the line BY intersect the circle ω again at Z.

Prove that the line BD is tangent to the circumcircle of triangle ADZ.

[hide=”comment”] Edited by Orl.

http://www.artofproblemsolving.com/This file was downloaded from the AoPS Math Olympiad Resources Page Page 5

Page 146: យុវសិស្ ឈឹ ទីុយ IMO Shortlist 1959-2009 · IMO Shortlist 1959 1 Prove that the fraction 21n+4 14n+3 ... Let α be the acute angel subtending, from A, that

IMO Shortlist 1998

Number Theory

1 Determine all pairs (x, y) of positive integers such that x2y +x+ y is divisible by xy2 + y +7.

2 Determine all pairs (a, b) of real numbers such that abbnc = bbanc for all positive integers n.(Note that bxc denotes the greatest integer less than or equal to x.)

3 Determine the smallest integer n ≥ 4 for which one can choose four different numbers a, b, cand d from any n distinct integers such that a + b− c− d is divisible by 20.

4 A sequence of integers a1, a2, a3, . . . is defined as follows: a1 = 1 and for n ≥ 1, an+1 is thesmallest integer greater than an such that ai+aj 6= 3ak for any i, j and k in 1, 2, 3, . . . , n+1,not necessarily distinct. Determine a1998.

5 Determine all positive integers n for which there exists an integer m such that 2n − 1 is adivisor of m2 + 9.

6 For any positive integer n, let τ(n) denote the number of its positive divisors (including 1and itself). Determine all positive integers m for which there exists a positive integer n suchthat τ(n2)

τ(n) = m.

7 Prove that for each positive integer n, there exists a positive integer with the followingproperties: It has exactly n digits. None of the digits is 0. It is divisible by the sum of itsdigits.

8 Let a0, a1, a2, . . . be an increasing sequence of nonnegative integers such that every nonneg-ative integer can be expressed uniquely in the form ai + 2aj + 4ak, where i, j and k are notnecessarily distinct. Determine a1998.

http://www.artofproblemsolving.com/This file was downloaded from the AoPS Math Olympiad Resources Page Page 6

Page 147: យុវសិស្ ឈឹ ទីុយ IMO Shortlist 1959-2009 · IMO Shortlist 1959 1 Prove that the fraction 21n+4 14n+3 ... Let α be the acute angel subtending, from A, that

IMO Shortlist 1999

Algebra

1 Let n ≥ 2 be a fixed integer. Find the least constant C such the inequality

∑i<j

xixj

(x2

i + x2j

)≤ C

(∑i

xi

)4

holds for any x1, . . . , xn ≥ 0 (the sum on the left consists of(n2

)summands). For this constant

C, characterize the instances of equality.

2 The numbers from 1 to n2 are randomly arranged in the cells of a n × n square (n ≥ 2).For any pair of numbers situated on the same row or on the same column the ration of thegreater number to the smaller number is calculated. Let us call the characteristic of thearrangement the smallest of these n2(n− 1) fractions. What is the highest possible value ofthe characteristic ?

3 A game is played by n girls (n ≥ 2), everybody having a ball. Each of the(n2

)pairs of players,

is an arbitrary order, exchange the balls they have at the moment. The game is called nicenice if at the end nobody has her own ball and it is called tiresome if at the end everybodyhas her initial ball. Determine the values of n for which there exists a nice game and thosefor which there exists a tiresome game.

4 Prove that the set of positive integers cannot be partitioned into three nonempty subsetssuch that, for any two integers x, y taken from two different subsets, the number x2−xy + y2

belongs to the third subset.

5 Find all the functions f : R 7→ R such that

f(x− f(y)) = f(f(y)) + xf(y) + f(x)− 1

for all x, y ∈ R.

6 For n ≥ 3 and a1 ≤ a2 ≤ . . . ≤ an given real numbers we have the following instructions:

- place out the numbers in some order in a ring; - delete one of the numbers from the ring;- if just two numbers are remaining in the ring: let S be the sum of these two numbers.Otherwise, if there are more the two numbers in the ring, replace

Afterwards start again with the step (2). Show that the largest sum S which can result inthis way is given by the formula

http://www.artofproblemsolving.com/This file was downloaded from the AoPS Math Olympiad Resources Page Page 1

Page 148: យុវសិស្ ឈឹ ទីុយ IMO Shortlist 1959-2009 · IMO Shortlist 1959 1 Prove that the fraction 21n+4 14n+3 ... Let α be the acute angel subtending, from A, that

IMO Shortlist 1999

Smax =n∑

k=2

(n− 2[k2 ]− 1

)ak.

http://www.artofproblemsolving.com/This file was downloaded from the AoPS Math Olympiad Resources Page Page 2

Page 149: យុវសិស្ ឈឹ ទីុយ IMO Shortlist 1959-2009 · IMO Shortlist 1959 1 Prove that the fraction 21n+4 14n+3 ... Let α be the acute angel subtending, from A, that

IMO Shortlist 1999

Combinatorics

1 Let n ≥ 1 be an integer. A path from (0, 0) to (n, n) in the xy plane is a chain of consecutiveunit moves either to the right (move denoted by E) or upwards (move denoted by N), allthe moves being made inside the half-plane x ≥ y. A step in a path is the occurence of twoconsecutive moves of the form EN . Show that the number of paths from (0, 0) to (n, n) thatcontain exactly s steps (n ≥ s ≥ 1) is

1s

(n− 1s− 1

)(n

s− 1

).

2 If a 5× n rectangle can be tiled using n pieces like those shown in the diagram, prove that nis even. Show that there are more than 2 · 3k−1 ways to file a fixed 5× 2k rectangle (k ≥ 3)with 2k pieces. (symmetric constructions are supposed to be different.)

3 A biologist watches a chameleon. The chameleon catches flies and rests after each catch. Thebiologist notices that:

- the first fly is caught after a resting period of one minute; - the resting period before catchingthe 2mth fly is the same as the resting period before catching the mth fly and one minuteshorter than the resting period before catching the (2m+1)th fly; - when the chameleon stopsresting, he catches a fly instantly.

- How many flies were caught by the chameleon before his first resting period of 9 minutes ina row ? - After how many minutes will the chameleon catch his 98th fly ? - How many flieswere caught by the chameleon after 1999 minutes have passed ?

4 Let A be a set of N residues (mod N2). Prove that there exists a set B of of N residues(mod N2) such that A + B = a + b|a ∈ A, b ∈ B contains at least half of all the residues(mod N2).

5 Let b be an even positive integer. We say that two different cells of a n × n board areneighboring if they have a common side. Find the minimal number of cells on he n × nboard that must be marked so that any cell marked or not marked) has a marked neighboringcell.

6 Suppose that every integer has been given one of the colours red, blue, green or yellow. Let xand y be odd integers so that |x| 6= |y|. Show that there are two integers of the same colourwhose difference has one of the following values: x, y, x + y or x− y.

http://www.artofproblemsolving.com/This file was downloaded from the AoPS Math Olympiad Resources Page Page 3

Page 150: យុវសិស្ ឈឹ ទីុយ IMO Shortlist 1959-2009 · IMO Shortlist 1959 1 Prove that the fraction 21n+4 14n+3 ... Let α be the acute angel subtending, from A, that

IMO Shortlist 1999

7 Let p > 3 be a prime number. For each nonempty subset T of 0, 1, 2, 3, . . . , p− 1, let E(T )be the set of all (p−1)-tuples (x1, . . . , xp−1), where each xi ∈ T and x1+2x2+. . .+(p−1)xp−1

is divisible by p and let |E(T )| denote the number of elements in E(T ). Prove that

|E(0, 1, 3)| ≥ |E(0, 1, 2)|

with equality if and only if p = 5.

http://www.artofproblemsolving.com/This file was downloaded from the AoPS Math Olympiad Resources Page Page 4

Page 151: យុវសិស្ ឈឹ ទីុយ IMO Shortlist 1959-2009 · IMO Shortlist 1959 1 Prove that the fraction 21n+4 14n+3 ... Let α be the acute angel subtending, from A, that

IMO Shortlist 1999

Geometry

1 Let ABC be a triangle and M be an interior point. Prove that

minMA, MB, MC+ MA + MB + MC < AB + AC + BC.

2 A circle is called a separator for a set of five points in a plane if it passes through three ofthese points, it contains a fourth point inside and the fifth point is outside the circle. Provethat every set of five points such that no three are collinear and no four are concyclic hasexactly four separators.

3 A set S of points from the space will be called completely symmetric if it has at least threeelements and fulfills the condition that for every two distinct points A and B from S, theperpendicular bisector plane of the segment AB is a plane of symmetry for S. Prove that if acompletely symmetric set is finite, then it consists of the vertices of either a regular polygon,or a regular tetrahedron or a regular octahedron.

4 For a triangle T = ABC we take the point X on the side (AB) such that AX/AB = 4/5,the point Y on the segment (CX) such that CY = 2Y X and, if possible, the point Z on theray (CA such that CXZ = 180− ABC. We denote by Σ the set of all triangles T for whichXY Z = 45. Prove that all triangles from Σ are similar and find the measure of their smallestangle.

5 Let ABC be a triangle, Ω its incircle and Ωa,Ωb,Ωc three circles orthogonal to Ω passingthrough (B,C), (A,C) and (A,B) respectively. The circles Ωa and Ωb meet again in C ′; inthe same way we obtain the points B′ and A′. Prove that the radius of the circumcircle ofA′B′C ′ is half the radius of Ω.

6 Two circles Ω1 and Ω2 touch internally the circle Ω in M and N and the center of Ω2 is onΩ1. The common chord of the circles Ω1 and Ω2 intersects Ω in A and B. MA and MBintersects Ω1 in C and D. Prove that Ω2 is tangent to CD.

7 The point M is inside the convex quadrilateral ABCD, such that

MA = MC, AMB = MAD + MCD and CMD = MCB + MAB.

Prove that AB · CM = BC ·MD and BM ·AD = MA · CD.

http://www.artofproblemsolving.com/This file was downloaded from the AoPS Math Olympiad Resources Page Page 5

Page 152: យុវសិស្ ឈឹ ទីុយ IMO Shortlist 1959-2009 · IMO Shortlist 1959 1 Prove that the fraction 21n+4 14n+3 ... Let α be the acute angel subtending, from A, that

IMO Shortlist 1999

8 Given a triangle ABC. The points A, B, C divide the circumcircle Ω of the triangle ABCinto three arcs BC, CA, AB. Let X be a variable point on the arc AB, and let O1 and O2

be the incenters of the triangles CAX and CBX. Prove that the circumcircle of the triangleXO1O2 intersects the circle Ω in a fixed point.

http://www.artofproblemsolving.com/This file was downloaded from the AoPS Math Olympiad Resources Page Page 6

Page 153: យុវសិស្ ឈឹ ទីុយ IMO Shortlist 1959-2009 · IMO Shortlist 1959 1 Prove that the fraction 21n+4 14n+3 ... Let α be the acute angel subtending, from A, that

IMO Shortlist 1999

Number Theory

1 Find all the pairs of positive integers (x, p) such that p is a prime, x ≤ 2p and xp−1 is adivisor of (p− 1)x + 1.

2 Prove that every positive rational number can be represented in the forma3 + b3

c3 + d3where

a,b,c,d are positive integers.

3 Prove that there exists two strictly increasing sequences (an) and (bn) such that an(an + 1)divides b2

n + 1 for every natural n.

4 Denote by S the set of all primes such the decimal representation of 1p has the fundamental

period divisible by 3. For every p ∈ S such that 1p has the fundamental period 3r one may

write

1p

= 0, a1a2 . . . a3ra1a2 . . . a3r . . . ,

where r = r(p); for every p ∈ S and every integer k ≥ 1 define f(k, p) by

f(k, p) = ak + ak+r(p) + ak+2.r(p)

a) Prove that S is infinite. b) Find the highest value of f(k, p) for k ≥ 1 and p ∈ S

5 Let n, k be positive integers such that n is not divisible by 3 and k ≥ n. Prove that there existsa positive integer m which is divisible by n and the sum of its digits in decimal representationis k.

6 Prove that for every real number M there exists an infinite arithmetic progression such that:

- each term is a positive integer and the common difference is not divisible by 10 - the sumof the digits of each term (in decimal representation) exceeds M .

http://www.artofproblemsolving.com/This file was downloaded from the AoPS Math Olympiad Resources Page Page 7

Page 154: យុវសិស្ ឈឹ ទីុយ IMO Shortlist 1959-2009 · IMO Shortlist 1959 1 Prove that the fraction 21n+4 14n+3 ... Let α be the acute angel subtending, from A, that

IMO Shortlist 2000

Algebra

1 Let a, b, c be positive real numbers so that abc = 1. Prove that(a− 1 +

1b

) (b− 1 +

1c

) (c− 1 +

1a

)≤ 1.

2 Let a, b, c be positive integers satisfying the conditions b > 2a and c > 2b. Show that thereexists a real number λ with the property that all the three numbers λa, λb, λc have theirfractional parts lying in the interval

(12 , 2

3

].

3 Find all pairs of functions f, g : R → R such that f (x + g(y)) = x · f(y)− y · f(x) + g(x) forall real x, y.

4 The function F is defined on the set of nonnegative integers and takes nonnegative integervalues satisfying the following conditions: for every n ≥ 0,

(i) F (4n) = F (2n) + F (n), (ii) F (4n + 2) = F (4n) + 1, (iii) F (2n + 1) = F (2n) + 1.

Prove that for each positive integer m, the number of integers n with 0 ≤ n < 2m andF (4n) = F (3n) is F (2m+1).

5 Let n ≥ 2 be a positive integer and λ a positive real number. Initially there are n fleas ona horizontal line, not all at the same point. We define a move as choosing two fleas at somepoints A and B, with A to the left of B, and letting the flea from A jump over the flea fromB to the point C so that BC

AB = λ.

Determine all values of λ such that, for any point M on the line and for any initial positionof the n fleas, there exists a sequence of moves that will take them all to the position right ofM .

6 A nonempty set A of real numbers is called a B3-set if the conditions a1, a2, a3, a4, a5, a6 ∈ Aand a1 + a2 + a3 = a4 + a5 + a6 imply that the sequences (a1, a2, a3) and (a4, a5, a6) areidentical up to a permutation. Let

A = a(0) = 0 < a(1) < a(2) < . . ., B = b(0) = 0 < b(1) < b(2) < . . .

be infinite sequences of real numbers with D(A) = D(B), where, for a set X of real numbers,D(X) denotes the difference set |x− y||x, y ∈ X. Prove that if A is a B3-set, then A = B.

http://www.artofproblemsolving.com/This file was downloaded from the AoPS Math Olympiad Resources Page Page 1

Page 155: យុវសិស្ ឈឹ ទីុយ IMO Shortlist 1959-2009 · IMO Shortlist 1959 1 Prove that the fraction 21n+4 14n+3 ... Let α be the acute angel subtending, from A, that

IMO Shortlist 2000

7 For a polynomial P of degree 2000 with distinct real coefficients let M(P ) be the set of allpolynomials that can be produced from P by permutation of its coefficients. A polynomial Pwill be called n-independent if P (n) = 0 and we can get from any Q ∈ M(P ) a polynomialQ1 such that Q1(n) = 0 by interchanging at most one pair of coefficients of Q. Find allintegers n for which n-independent polynomials exist.

http://www.artofproblemsolving.com/This file was downloaded from the AoPS Math Olympiad Resources Page Page 2

Page 156: យុវសិស្ ឈឹ ទីុយ IMO Shortlist 1959-2009 · IMO Shortlist 1959 1 Prove that the fraction 21n+4 14n+3 ... Let α be the acute angel subtending, from A, that

IMO Shortlist 2000

Combinatorics

1 A magician has one hundred cards numbered 1 to 100. He puts them into three boxes, a redone, a white one and a blue one, so that each box contains at least one card. A member ofthe audience draws two cards from two different boxes and announces the sum of numberson those cards. Given this information, the magician locates the box from which no card hasbeen drawn.

How many ways are there to put the cards in the three boxes so that the trick works?

2 A staircase-brick with 3 steps of width 2 is made of 12 unit cubes. Determine all integers nfor which it is possible to build a cube of side n using such bricks.

3 Let n ≥ 4 be a fixed positive integer. Given a set S = P1, P2, . . . , Pn of n points in theplane such that no three are collinear and no four concyclic, let at, 1 ≤ t ≤ n, be the numberof circles PiPjPk that contain Pt in their interior, and let m(S) =

∑ni=1 ai. Prove that there

exists a positive integer f(n), depending only on n, such that the points of S are the verticesof a convex polygon if and only if m(S) = f(n).

4 Let n and k be positive integers such that 12n < k ≤ 2

3n. Find the least number m for whichit is possible to place m pawns on m squares of an n × n chessboard so that no column orrow contains a block of k adjacent unoccupied squares.

5 A number of n rectangles are drawn in the plane. Each rectangle has parallel sides and thesides of distinct rectangles lie on distinct lines. The rectangles divide the plane into a numberof regions. For each region R let v(R) be the number of vertices. Take the sum

∑v(R) over

the regions which have one or more vertices of the rectangles in their boundary. Show thatthis sum is less than 40n.

6 Let p and q be relatively prime positive integers. A subset S of 0, 1, 2, . . . is called ideal if0 ∈ S and for each element n ∈ S, the integers n + p and n + q belong to S. Determine thenumber of ideal subsets of 0, 1, 2, . . ..

http://www.artofproblemsolving.com/This file was downloaded from the AoPS Math Olympiad Resources Page Page 3

Page 157: យុវសិស្ ឈឹ ទីុយ IMO Shortlist 1959-2009 · IMO Shortlist 1959 1 Prove that the fraction 21n+4 14n+3 ... Let α be the acute angel subtending, from A, that

IMO Shortlist 2000

Geometry

1 In the plane we are given two circles intersecting at X and Y . Prove that there exist fourpoints with the following property:

(P) For every circle touching the two given circles at A and B, and meeting the line XY atC and D, each of the lines AC, AD, BC, BD passes through one of these points.

2 Two circles G1 and G2 intersect at two points M and N . Let AB be the line tangent to thesecircles at A and B, respectively, so that M lies closer to AB than N . Let CD be the lineparallel to AB and passing through the point M , with C on G1 and D on G2. Lines ACand BD meet at E; lines AN and CD meet at P ; lines BN and CD meet at Q. Show thatEP = EQ.

3 Let O be the circumcenter and H the orthocenter of an acute triangle ABC. Show that thereexist points D,E, and F on sides BC, CA, and AB respectively such that

OD + DH = OE + EH = OF + FH

and the lines AD,BE, and CF are concurrent.

4 Let A1A2 . . . An be a convex polygon, n ≥ 4. Prove that A1A2 . . . An is cyclic if and onlyif to each vertex Aj one can assign a pair (bj , cj) of real numbers, j = 1, 2, . . . , n, so thatAiAj = bjcibicj for all i, j with 1 ≤ i < j ≤ n.

5 Let ABC be an acute-angled triangle, and let w be the circumcircle of triangle ABC.

The tangent to the circle w at the point A meets the tangent to the circle w at C at the pointB′. The line BB′ intersects the line AC at E, and N is the midpoint of the segment BE.

Similarly, the tangent to the circle w at the point B meets the tangent to the circle w at thepoint C at the point A′. The line AA′ intersects the line BC at D, and M is the midpoint ofthe segment AD.

a) Show that ]ABM = ]BAN . b) If AB = 1, determine the values of BC and AC for thetriangles ABC which maximise ]ABM .

6 Let ABCD be a convex quadrilateral. The perpendicular bisectors of its sides AB andCD meet at Y . Denote by X a point inside the quadrilateral ABCD such that ]ADX =]BCX < 90 and ]DAX = ]CBX < 90. Show that ]AY B = 2 · ]ADX.

7 Ten gangsters are standing on a flat surface, and the distances between them are all distinct.At twelve oclock, when the church bells start chiming, each of them fatally shoots the oneamong the other nine gangsters who is the nearest. At least how many gangsters will bekilled?

http://www.artofproblemsolving.com/This file was downloaded from the AoPS Math Olympiad Resources Page Page 4

Page 158: យុវសិស្ ឈឹ ទីុយ IMO Shortlist 1959-2009 · IMO Shortlist 1959 1 Prove that the fraction 21n+4 14n+3 ... Let α be the acute angel subtending, from A, that

IMO Shortlist 2000

8 Let AH1, BH2, CH3 be the altitudes of an acute angled triangle ABC. Its incircle touchesthe sides BC, AC and AB at T1, T2 and T3 respectively. Consider the symmetric images ofthe lines H1H2,H2H3 and H3H1 with respect to the lines T1T2, T2T3 and T3T1. Prove thatthese images form a triangle whose vertices lie on the incircle of ABC.

http://www.artofproblemsolving.com/This file was downloaded from the AoPS Math Olympiad Resources Page Page 5

Page 159: យុវសិស្ ឈឹ ទីុយ IMO Shortlist 1959-2009 · IMO Shortlist 1959 1 Prove that the fraction 21n+4 14n+3 ... Let α be the acute angel subtending, from A, that

IMO Shortlist 2000

Number Theory

1 Determine all positive integers n ≥ 2 that satisfy the following condition: for all a and brelatively prime to n we have a ≡ b (mod n) if and only if ab ≡ 1 (mod n).

2 For every positive integers n let d(n) the number of all positive integers of n. Determine allpositive integers n with the property: d3(n) = 4n.

3 Does there exist a positive integer n such that n has exactly 2000 prime divisors and n divides2n + 1?

4 Find all triplets of positive integers (a,m, n) such that am + 1 | (a + 1)n.

5 Prove that there exist infinitely many positive integers n such that p = nr, where p and r arerespectively the semiperimeter and the inradius of a triangle with integer side lengths.

6 Show that the set of positive integers that cannot be represented as a sum of distinct perfectsquares is finite.

http://www.artofproblemsolving.com/This file was downloaded from the AoPS Math Olympiad Resources Page Page 6

Page 160: យុវសិស្ ឈឹ ទីុយ IMO Shortlist 1959-2009 · IMO Shortlist 1959 1 Prove that the fraction 21n+4 14n+3 ... Let α be the acute angel subtending, from A, that

IMO Shortlist 2001

Algebra

1 Let T denote the set of all ordered triples (p, q, r) of nonnegative integers. Find all functionsf : T → R satisfying

f(p, q, r) =

0 if pqr = 0,

1 + 16 (f(p + 1, q − 1, r) + f(p− 1, q + 1, r)

+f(p− 1, q, r + 1) + f(p + 1, q, r − 1)+f(p, q + 1, r − 1) + f(p, q − 1, r + 1)) otherwise

for all nonnegative integers p, q, r.

2 Let a0, a1, a2, . . . be an arbitrary infinite sequence of positive numbers. Show that the in-equality 1 + an > an−1

n√

2 holds for infinitely many positive integers n.

3 Let x1, x2, . . . , xn be arbitrary real numbers. Prove the inequality

x1

1 + x21

+x2

1 + x21 + x2

2

+ · · ·+ xn

1 + x21 + · · ·+ x2

n

<√

n.

4 Find all functions f : R → R, satisfying

f(xy)(f(x)− f(y)) = (x− y)f(x)f(y)

for all x, y.

5 Find all positive integers a1, a2, . . . , an such that

99100

=a0

a1+

a1

a2+ · · ·+ an−1

an,

where a0 = 1 and (ak+1 − 1)ak−1 ≥ a2k(ak − 1) for k = 1, 2, . . . , n− 1.

6 Prove that for all positive real numbers a, b, c,

a√a2 + 8bc

+b√

b2 + 8ca+

c√c2 + 8ab

≥ 1.

http://www.artofproblemsolving.com/This file was downloaded from the AoPS Math Olympiad Resources Page Page 1

Page 161: យុវសិស្ ឈឹ ទីុយ IMO Shortlist 1959-2009 · IMO Shortlist 1959 1 Prove that the fraction 21n+4 14n+3 ... Let α be the acute angel subtending, from A, that

IMO Shortlist 2001

Combinatorics

1 Let A = (a1, a2, . . . , a2001) be a sequence of positive integers. Let m be the number of 3-element subsequences (ai, aj , ak) with 1 ≤ i < j < k ≤ 2001, such that aj = ai + 1 andak = aj + 1. Considering all such sequences A, find the greatest value of m.

2 Let n be an odd integer greater than 1 and let c1, c2, . . . , cn be integers. For each permu-tation a = (a1, a2, . . . , an) of 1, 2, . . . , n, define S(a) =

∑ni=1 ciai. Prove that there exist

permutations a 6= b of 1, 2, . . . , n such that n! is a divisor of S(a)− S(b).

3 Define a k-clique to be a set of k people such that every pair of them are acquainted witheach other. At a certain party, every pair of 3-cliques has at least one person in common, andthere are no 5-cliques. Prove that there are two or fewer people at the party whose departureleaves no 3-clique remaining.

4 A set of three nonnegative integers x, y, z with x < y < z is called historic if z−y, y−x =1776, 2001. Show that the set of all nonnegative integers can be written as the union ofpairwise disjoint historic sets.

5 Find all finite sequences (x0, x1, . . . , xn) such that for every j, 0 ≤ j ≤ n, xj equals thenumber of times j appears in the sequence.

6 For a positive integer n define a sequence of zeros and ones to be balanced if it contains nzeros and n ones. Two balanced sequences a and b are neighbors if you can move one of the 2nsymbols of a to another position to form b. For instance, when n = 4, the balanced sequences01101001 and 00110101 are neighbors because the third (or fourth) zero in the first sequencecan be moved to the first or second position to form the second sequence. Prove that thereis a set S of at most 1

n+1

(2nn

)balanced sequences such that every balanced sequence is equal

to or is a neighbor of at least one sequence in S.

7 A pile of n pebbles is placed in a vertical column. This configuration is modified accordingto the following rules. A pebble can be moved if it is at the top of a column which containsat least two more pebbles than the column immediately to its right. (If there are no pebblesto the right, think of this as a column with 0 pebbles.) At each stage, choose a pebble fromamong those that can be moved (if there are any) and place it at the top of the column to itsright. If no pebbles can be moved, the configuration is called a final configuration. For eachn, show that, no matter what choices are made at each stage, the final configuration obtainedis unique. Describe that configuration in terms of n.

[url=http://www.mathlinks.ro/Forum/viewtopic.php?p=119189]IMO ShortList 2001, combi-natorics problem 7, alternative[/url]

http://www.artofproblemsolving.com/This file was downloaded from the AoPS Math Olympiad Resources Page Page 2

Page 162: យុវសិស្ ឈឹ ទីុយ IMO Shortlist 1959-2009 · IMO Shortlist 1959 1 Prove that the fraction 21n+4 14n+3 ... Let α be the acute angel subtending, from A, that

IMO Shortlist 2001

8 Twenty-one girls and twenty-one boys took part in a mathematical competition. It turnedout that each contestant solved at most six problems, and for each pair of a girl and a boy,there was at least one problem that was solved by both the girl and the boy. Show that thereis a problem that was solved by at least three girls and at least three boys.

http://www.artofproblemsolving.com/This file was downloaded from the AoPS Math Olympiad Resources Page Page 3

Page 163: យុវសិស្ ឈឹ ទីុយ IMO Shortlist 1959-2009 · IMO Shortlist 1959 1 Prove that the fraction 21n+4 14n+3 ... Let α be the acute angel subtending, from A, that

IMO Shortlist 2001

Geometry

1 Let A1 be the center of the square inscribed in acute triangle ABC with two vertices of thesquare on side BC. Thus one of the two remaining vertices of the square is on side AB andthe other is on AC. Points B1, C1 are defined in a similar way for inscribed squares with twovertices on sides AC and AB, respectively. Prove that lines AA1, BB1, CC1 are concurrent.

2 Consider an acute-angled triangle ABC. Let P be the foot of the altitude of triangle ABCissuing from the vertex A, and let O be the circumcenter of triangle ABC. Assume that∠C ≥ ∠B + 30. Prove that ∠A + ∠COP < 90.

3 Let ABC be a triangle with centroid G. Determine, with proof, the position of the point Pin the plane of ABC such that AP ·AG + BP ·BG + CP ·CG is a minimum, and express thisminimum value in terms of the side lengths of ABC.

4 Let M be a point in the interior of triangle ABC. Let A′ lie on BC with MA′ perpendicularto BC. Define B′ on CA and C ′ on AB similarly. Define

p(M) =MA′ ·MB′ ·MC ′

MA ·MB ·MC.

Determine, with proof, the location of M such that p(M) is maximal. Let µ(ABC) denotethis maximum value. For which triangles ABC is the value of µ(ABC) maximal?

5 Let ABC be an acute triangle. Let DAC, EAB, and FBC be isosceles triangles exterior toABC, with DA = DC, EA = EB, and FB = FC, such that

∠ADC = 2∠BAC, ∠BEA = 2∠ABC, ∠CFB = 2∠ACB.

Let D′ be the intersection of lines DB and EF , let E′ be the intersection of EC and DF ,and let F ′ be the intersection of FA and DE. Find, with proof, the value of the sum

DB

DD′ +EC

EE′ +FA

FF ′ .

6 Let ABC be a triangle and P an exterior point in the plane of the triangle. Suppose the linesAP , BP , CP meet the sides BC, CA, AB (or extensions thereof) in D, E, F , respectively.Suppose further that the areas of triangles PBD, PCE, PAF are all equal. Prove that eachof these areas is equal to the area of triangle ABC itself.

http://www.artofproblemsolving.com/This file was downloaded from the AoPS Math Olympiad Resources Page Page 4

Page 164: យុវសិស្ ឈឹ ទីុយ IMO Shortlist 1959-2009 · IMO Shortlist 1959 1 Prove that the fraction 21n+4 14n+3 ... Let α be the acute angel subtending, from A, that

IMO Shortlist 2001

7 Let O be an interior point of acute triangle ABC. Let A1 lie on BC with OA1 perpendicularto BC. Define B1 on CA and C1 on AB similarly. Prove that O is the circumcenter ofABC if and only if the perimeter of A1B1C1 is not less than any one of the perimeters ofAB1C1, BC1A1, and CA1B1.

8 Let ABC be a triangle with ∠BAC = 60. Let AP bisect ∠BAC and let BQ bisect ∠ABC,with P on BC and Q on AC. If AB + BP = AQ + QB, what are the angles of the triangle?

http://www.artofproblemsolving.com/This file was downloaded from the AoPS Math Olympiad Resources Page Page 5

Page 165: យុវសិស្ ឈឹ ទីុយ IMO Shortlist 1959-2009 · IMO Shortlist 1959 1 Prove that the fraction 21n+4 14n+3 ... Let α be the acute angel subtending, from A, that

IMO Shortlist 2001

Number Theory

1 Prove that there is no positive integer n such that, for k = 1, 2, . . . , 9, the leftmost digit (indecimal notation) of (n + k)! equals k.

2 Consider the system x + y = z + u, 2xy = zu. Find the greatest value of the real constant msuch that m ≤ x/y for any positive integer solution (x, y, z, u) of the system, with x ≥ y.

3 Let a1 = 1111, a2 = 1212, a3 = 1313, and an = |an−1−an−2|+ |an−2−an−3|, n ≥ 4. Determinea1414 .

4 Let p ≥ 5 be a prime number. Prove that there exists an integer a with 1 ≤ a ≤ p − 2 suchthat neither ap−1 − 1 nor (a + 1)p−1 − 1 is divisible by p2.

5 Let a > b > c > d be positive integers and suppose that

ac + bd = (b + d + a− c)(b + d− a + c).

Prove that ab + cd is not prime.

6 Is it possible to find 100 positive integers not exceeding 25,000, such that all pairwise sumsof them are different?

http://www.artofproblemsolving.com/This file was downloaded from the AoPS Math Olympiad Resources Page Page 6

Page 166: យុវសិស្ ឈឹ ទីុយ IMO Shortlist 1959-2009 · IMO Shortlist 1959 1 Prove that the fraction 21n+4 14n+3 ... Let α be the acute angel subtending, from A, that

IMO Shortlist 2002

Algebra

1 Find all functions f from the reals to the reals such that

f (f(x) + y) = 2x + f (f(y)− x)

for all real x, y.

2 Let a1, a2, . . . be an infinite sequence of real numbers, for which there exists a real number cwith 0 ≤ ai ≤ c for all i, such that

| ai − aj | ≥1

i + j∀i, j with i 6= j.

Prove that c ≥ 1.

3 Let P be a cubic polynomial given by P (x) = ax3 + bx2 + cx + d, where a, b, c, d are integersand a 6= 0. Suppose that xP (x) = yP (y) for infinitely many pairs x, y of integers with x 6= y.Prove that the equation P (x) = 0 has an integer root.

4 Find all functions f from the reals to the reals such that

(f(x) + f(z)) (f(y) + f(t)) = f(xy − zt) + f(xt + yz)

for all real x, y, z, t.

5 Let n be a positive integer that is not a perfect cube. Define real numbers a, b, c by

a = 3√

n , b =1

a− [a], c =

1b− [b]

,

where [x] denotes the integer part of x. Prove that there are infinitely many such integers nwith the property that there exist integers r, s, t, not all zero, such that ra + sb + tc = 0.

6 Let A be a non-empty set of positive integers. Suppose that there are positive integers b1, . . . bn

and c1, . . . , cn such that

- for each i the set biA + ci = bia + ci : a ∈ A is a subset of A, and

- the sets biA + ci and bjA + cj are disjoint whenever i 6= j

Prove that1b1

+ . . . +1bn

≤ 1.

http://www.artofproblemsolving.com/This file was downloaded from the AoPS Math Olympiad Resources Page Page 1

Page 167: យុវសិស្ ឈឹ ទីុយ IMO Shortlist 1959-2009 · IMO Shortlist 1959 1 Prove that the fraction 21n+4 14n+3 ... Let α be the acute angel subtending, from A, that

IMO Shortlist 2002

Combinatorics

1 Let n be a positive integer. Each point (x, y) in the plane, where x and y are non-negativeintegers with x + y < n, is coloured red or blue, subject to the following condition: if a point(x, y) is red, then so are all points (x′, y′) with x′ ≤ x and y′ ≤ y. Let A be the number ofways to choose n blue points with distinct x-coordinates, and let B be the number of waysto choose n blue points with distinct y-coordinates. Prove that A = B.

2 For n an odd positive integer, the unit squares of an n×n chessboard are coloured alternatelyblack and white, with the four corners coloured black. A it tromino is an L-shape formedby three connected unit squares. For which values of n is it possible to cover all the blacksquares with non-overlapping trominos? When it is possible, what is the minimum numberof trominos needed?

3 Let n be a positive integer. A sequence of n positive integers (not necessarily distinct) iscalled full if it satisfies the following condition: for each positive integer k ≥ 2, if the numberk appears in the sequence then so does the number k − 1, and moreover the first occurrenceof k− 1 comes before the last occurrence of k. For each n, how many full sequences are there?

4 Let T be the set of ordered triples (x, y, z), where x, y, z are integers with 0 ≤ x, y, z ≤ 9.Players A and B play the following guessing game. Player A chooses a triple (x, y, z) inT , and Player B has to discover A’s triple in as few moves as possible. A move consistsof the following: B gives A a triple (a, b, c) in T , and A replies by giving B the number|x + y − a− b| + |y + z − b− c| + |z + x− c− a|. Find the minimum number of moves thatB needs to be sure of determining A’s triple.

5 Let r ≥ 2 be a fixed positive integer, and let F be an infinite family of sets, each of size r, notwo of which are disjoint. Prove that there exists a set of size r− 1 that meets each set in F .

6 Let n be an even positive integer. Show that there is a permutation (x1, x2, . . . , xn) of(1, 2, . . . , n) such that for every i ∈ 1, 2, ..., n, the number xi+1 is one of the numbers2xi, 2xi−1, 2xi−n, 2xi−n−1. Hereby, we use the cyclic subscript convention, so that xn+1

means x1.

7 Among a group of 120 people, some pairs are friends. A weak quartet is a set of four peoplecontaining exactly one pair of friends. What is the maximum possible number of weak quartets?

http://www.artofproblemsolving.com/This file was downloaded from the AoPS Math Olympiad Resources Page Page 2

Page 168: យុវសិស្ ឈឹ ទីុយ IMO Shortlist 1959-2009 · IMO Shortlist 1959 1 Prove that the fraction 21n+4 14n+3 ... Let α be the acute angel subtending, from A, that

IMO Shortlist 2002

Geometry

1 Let B be a point on a circle S1, and let A be a point distinct from B on the tangent at Bto S1. Let C be a point not on S1 such that the line segment AC meets S1 at two distinctpoints. Let S2 be the circle touching AC at C and touching S1 at a point D on the oppositeside of AC from B. Prove that the circumcentre of triangle BCD lies on the circumcircle oftriangle ABC.

2 Let ABC be a triangle for which there exists an interior point F such that ∠AFB = ∠BFC =∠CFA. Let the lines BF and CF meet the sides AC and AB at D and E respectively. Provethat

AB + AC ≥ 4DE.

3 The circle S has centre O, and BC is a diameter of S. Let A be a point of S such that∠AOB < 120. Let D be the midpoint of the arc AB which does not contain C. The linethrough O parallel to DA meets the line AC at I. The perpendicular bisector of OA meetsS at E and at F . Prove that I is the incentre of the triangle CEF.

4 Circles S1 and S2 intersect at points P and Q. Distinct points A1 and B1 (not at P or Q)are selected on S1. The lines A1P and B1P meet S2 again at A2 and B2 respectively, andthe lines A1B1 and A2B2 meet at C. Prove that, as A1 and B1 vary, the circumcentres oftriangles A1A2C all lie on one fixed circle.

5 For any set S of five points in the plane, no three of which are collinear, let M(S) and m(S)denote the greatest and smallest areas, respectively, of triangles determined by three pointsfrom S. What is the minimum possible value of M(S)/m(S) ?

6 Let n ≥ 3 be a positive integer. Let C1, C2, C3, . . . , Cn be unit circles in the plane, withcentres O1, O2, O3, . . . , On respectively. If no line meets more than two of the circles, provethat ∑

1≤i<j≤n

1OiOj

≤ (n− 1)π4

.

7 The incircle Ω of the acute-angled triangle ABC is tangent to its side BC at a point K. LetAD be an altitude of triangle ABC, and let M be the midpoint of the segment AD. If N isthe common point of the circle Ω and the line KM (distinct from K), then prove that theincircle Ω and the circumcircle of triangle BCN are tangent to each other at the point N .

http://www.artofproblemsolving.com/This file was downloaded from the AoPS Math Olympiad Resources Page Page 3

Page 169: យុវសិស្ ឈឹ ទីុយ IMO Shortlist 1959-2009 · IMO Shortlist 1959 1 Prove that the fraction 21n+4 14n+3 ... Let α be the acute angel subtending, from A, that

IMO Shortlist 2002

8 Let two circles S1 and S2 meet at the points A and B. A line through A meets S1 againat C and S2 again at D. Let M , N , K be three points on the line segments CD, BC, BDrespectively, with MN parallel to BD and MK parallel to BC. Let E and F be points onthose arcs BC of S1 and BD of S2 respectively that do not contain A. Given that EN isperpendicular to BC and FK is perpendicular to BD prove that ∠EMF = 90.

http://www.artofproblemsolving.com/This file was downloaded from the AoPS Math Olympiad Resources Page Page 4

Page 170: យុវសិស្ ឈឹ ទីុយ IMO Shortlist 1959-2009 · IMO Shortlist 1959 1 Prove that the fraction 21n+4 14n+3 ... Let α be the acute angel subtending, from A, that

IMO Shortlist 2002

Number Theory

1 What is the smallest positive integer t such that there exist integers x1, x2, . . . , xt with

x31 + x3

2 + . . . + x3t = 20022002 ?

2 Let n ≥ 2 be a positive integer, with divisors 1 = d1 < d2 < . . . < dk = n. Prove thatd1d2 + d2d3 + . . . + dk−1dk is always less than n2, and determine when it is a divisor of n2.

3 Let p1, p2, . . . , pn be distinct primes greater than 3. Show that 2p1p2...pn + 1 has at least 4n

divisors.

4 Is there a positive integer m such that the equation

1a

+1b

+1c

+1

abc=

m

a + b + c

has infinitely many solutions in positive integers a, b, c?

5 Let m,n ≥ 2 be positive integers, and let a1, a2, . . . , an be integers, none of which is a multipleof mn−1. Show that there exist integers e1, e2, . . . , en, not all zero, with | ei | < m for all i,such that e1a1 + e2a2 + . . . + enan is a multiple of mn.

6 Find all pairs of positive integers m,n ≥ 3 for which there exist infinitely many positiveintegers a such that

am + a− 1an + a2 − 1

is itself an integer.

Laurentiu Panaitopol, Romania

http://www.artofproblemsolving.com/This file was downloaded from the AoPS Math Olympiad Resources Page Page 5

Page 171: យុវសិស្ ឈឹ ទីុយ IMO Shortlist 1959-2009 · IMO Shortlist 1959 1 Prove that the fraction 21n+4 14n+3 ... Let α be the acute angel subtending, from A, that

IMO Shortlist 2003

Algebra

1 Let aij (with the indices i and j from the set 1, 2, 3) be real numbers such that

aij > 0 for i = j; aij < 0 for i 6= j.

Prove the existence of positive real numbers c1, c2, c3 such that the numbers

a11c1 + a12c2 + a13c3, a21c1 + a22c2 + a23c3, a31c1 + a32c2 + a33c3

are either all negative, or all zero, or all positive.

2 Find all nondecreasing functions f : R → R such that (i) f(0) = 0, f(1) = 1; (ii) f(a)+f(b) =f(a)f(b) + f(a + b− ab) for all real numbers a, b such that a < 1 < b.

3 Consider two monotonically decreasing sequences (ak) and (bk), where k ≥ 1, and ak and bk

are positive real numbers for every k. Now, define the sequences

ck = min (ak, bk); Ak = a1 + a2 + ... + ak; Bk = b1 + b2 + ... + bk; Ck = c1 + c2 + ... + ck

for all natural numbers k.

(a) Do there exist two monotonically decreasing sequences (ak) and (bk) of positive realnumbers such that the sequences (Ak) and (Bk) are not bounded, while the sequence (Ck) isbounded?

(b) Does the answer to problem (a) change if we stipulate that the sequence (bk) must be

bk =1k

for all k ?

4 Let n be a positive integer and let x1 ≤ x2 ≤ · · · ≤ xn be real numbers. Prove that

n∑i,j=1

|xi − xj |

2

≤ 2(n2 − 1)3

n∑i,j=1

(xi − xj)2.

Show that the equality holds if and only if x1, . . . , xn is an arithmetic sequence.

5 Let R+ be the set of all positive real numbers. Find all functions f : R+ −→ R+ that satisfythe following conditions:

- f(xyz) + f(x) + f(y) + f(z) = f(√

xy)f(√

yz)f(√

zx) for all x, y, z ∈ R+;

- f(x) < f(y) for all 1 ≤ x < y.

6 Let n be a positive integer and let (x1, . . . , xn), (y1, . . . , yn) be two sequences of positivereal numbers. Suppose (z2, . . . , z2n) is a sequence of positive real numbers such that z2

i+j ≥xiyj for all 1 ≤ i, j ≤ n.

http://www.artofproblemsolving.com/This file was downloaded from the AoPS Math Olympiad Resources Page Page 1

Page 172: យុវសិស្ ឈឹ ទីុយ IMO Shortlist 1959-2009 · IMO Shortlist 1959 1 Prove that the fraction 21n+4 14n+3 ... Let α be the acute angel subtending, from A, that

IMO Shortlist 2003

Let M = maxz2, . . . , z2n. Prove that

(M + z2 + · · ·+ z2n

2n

)2

≥(

x1 + · · ·+ xn

n

)(y1 + · · ·+ yn

n

).

[hide=”comment”] Edited by Orl.

http://www.artofproblemsolving.com/This file was downloaded from the AoPS Math Olympiad Resources Page Page 2

Page 173: យុវសិស្ ឈឹ ទីុយ IMO Shortlist 1959-2009 · IMO Shortlist 1959 1 Prove that the fraction 21n+4 14n+3 ... Let α be the acute angel subtending, from A, that

IMO Shortlist 2003

Combinatorics

1 Let A be a 101-element subset of the set S = 1, 2, . . . , 1000000. Prove that there existnumbers t1, t2, . . . , t100 in S such that the sets

Aj = x + tj | x ∈ A, j = 1, 2, . . . , 100

are pairwise disjoint.

2 Let D1, D2, ..., Dn be closed discs in the plane. (A closed disc is the region limited by a circle,taken jointly with this circle.) Suppose that every point in the plane is contained in at most2003 discs Di. Prove that there exists a disc Dk which intersects at most 7 · 2003− 1 = 14020other discs Di.

3 Let n ≥ 5 be an integer. Find the maximal integer k such that there exists a polygon with nvertices (convex or not, but not self-intersecting!) having k internal 90 angles.

4 Given n real numbers x1, x2, ..., xn, and n further real numbers y1, y2, ..., yn. The elementsaij (with 1 ≤ i, j ≤ n) of an n× n matrix are defined as follows:

aij =

1 if xi + yj ≥ 0;0 if xi + yj < 0.

Further, let B be an n×n matrix whose elements are numbers from the set 0; 1 satisfying thefollowing condition: The sum of all elements of each row of B equals the sum of all elementsof the corresponding row of A; the sum of all elements of each column of B equals the sum ofall elements of the corresponding column of A. Show that in this case, A = B.

[hide=”comment”] (This one is from the ISL 2003, but in any case, [url=http://www.bundeswettbewerb-mathematik.de/imo/aufgaben/aufgaben.htm]the official problems and solutions - in German-[/url] are already online, hence I take the liberty to post it here.)

Darij

5 Regard a plane with a Cartesian coordinate system; for each point with integer coordinates,draw a circular disk centered at this point and having the radius 1

1000 .

a) Prove the existence of an equilateral triangle whose vertices lie in the interior of differentdisks;

b) Show that every equilateral triangle whose vertices lie in the interior of different disks hasa sidelength ¿ 96.

Radu Gologan, Romania [hide=”Remark”] [The ”¿ 96” in (b) can be strengthened to ”¿ 124”.By the way, part (a) of this problem is the place where I used [url=http://mathlinks.ro/viewtopic.php?t=5537]thewell-known ”Dedekind” theorem[/url].]

http://www.artofproblemsolving.com/This file was downloaded from the AoPS Math Olympiad Resources Page Page 3

Page 174: យុវសិស្ ឈឹ ទីុយ IMO Shortlist 1959-2009 · IMO Shortlist 1959 1 Prove that the fraction 21n+4 14n+3 ... Let α be the acute angel subtending, from A, that

IMO Shortlist 2003

6 Let f(k) be the number of all non-negative integers n satisfying the following conditions:

(1) The integer n has exactly k digits in the decimal representation (where the first digit isnot necessarily non-zero!), i. e. we have 0 ≤ n < 10k.

(2) These k digits of n can be permuted in such a way that the resulting number is divisibleby 11.

Show that for any positive integer number m, we have f (2m) = 10f (2m− 1).

http://www.artofproblemsolving.com/This file was downloaded from the AoPS Math Olympiad Resources Page Page 4

Page 175: យុវសិស្ ឈឹ ទីុយ IMO Shortlist 1959-2009 · IMO Shortlist 1959 1 Prove that the fraction 21n+4 14n+3 ... Let α be the acute angel subtending, from A, that

IMO Shortlist 2003

Geometry

1 Let ABCD be a cyclic quadrilateral. Let P , Q, R be the feet of the perpendiculars from Dto the lines BC, CA, AB, respectively. Show that PQ = QR if and only if the bisectors of∠ABC and ∠ADC are concurrent with AC.

2 Given three fixed pairwisely distinct points A, B, C lying on one straight line in this order.Let G be a circle passing through A and C whose center does not lie on the line AC. Thetangents to G at A and C intersect each other at a point P . The segment PB meets thecircle G at Q.

Show that the point of intersection of the angle bisector of the angle AQC with the line ACdoes not depend on the choice of the circle G.

3 Let ABC be a triangle, and P a point in the interior of this triangle. Let D, E, F be the feetof the perpendiculars from the point P to the lines BC, CA, AB, respectively. Assume that

AP 2 + PD2 = BP 2 + PE2 = CP 2 + PF 2.

Furthermore, let Ia, Ib, Ic be the excenters of triangle ABC. Show that the point P is thecircumcenter of triangle IaIbIc.

4 Let Γ1, Γ2, Γ3, Γ4 be distinct circles such that Γ1, Γ3 are externally tangent at P , and Γ2,Γ4 are externally tangent at the same point P . Suppose that Γ1 and Γ2; Γ2 and Γ3; Γ3 andΓ4; Γ4 and Γ1 meet at A, B, C, D, respectively, and that all these points are different fromP . Prove that

AB ·BC

AD ·DC=

PB2

PD2.

5 Let ABC be an isosceles triangle with AC = BC, whose incentre is I. Let P be a pointon the circumcircle of the triangle AIB lying inside the triangle ABC. The lines through Pparallel to CA and CB meet AB at D and E, respectively. The line through P parallel toAB meets CA and CB at F and G, respectively. Prove that the lines DF and EG intersecton the circumcircle of the triangle ABC.

[hide=”comment”] (According to my team leader, last year some of the countries wanted ageometry question that was even easier than this...that explains IMO 2003/4...)

[Note by Darij: This was also Problem 6 of the German pre-TST 2004, written in December03.]

Edited by Orl.

http://www.artofproblemsolving.com/This file was downloaded from the AoPS Math Olympiad Resources Page Page 5

Page 176: យុវសិស្ ឈឹ ទីុយ IMO Shortlist 1959-2009 · IMO Shortlist 1959 1 Prove that the fraction 21n+4 14n+3 ... Let α be the acute angel subtending, from A, that

IMO Shortlist 2003

6 Each pair of opposite sides of a convex hexagon has the following property: the distance

between their midpoints is equal to√

32

times the sum of their lengths. Prove that all theangles of the hexagon are equal.

7 Let ABC be a triangle with semiperimeter s and inradius r. The semicircles with diametersBC, CA, AB are drawn on the outside of the triangle ABC. The circle tangent to all ofthese three semicircles has radius t. Prove that

s

2< t ≤ s

2+

(1−

√3

2

)r.

Alternative formulation. In a triangle ABC, construct circles with diameters BC, CA, andAB, respectively. Construct a circle w externally tangent to these three circles. Let the radiusof this circle w be t. Prove: s

2 < t ≤ s2 + 1

2

(2−

√3)r, where r is the inradius and s is the

semiperimeter of triangle ABC.

http://www.artofproblemsolving.com/This file was downloaded from the AoPS Math Olympiad Resources Page Page 6

Page 177: យុវសិស្ ឈឹ ទីុយ IMO Shortlist 1959-2009 · IMO Shortlist 1959 1 Prove that the fraction 21n+4 14n+3 ... Let α be the acute angel subtending, from A, that

IMO Shortlist 2003

Number Theory

1 Let m be a fixed integer greater than 1. The sequence x0, x1, x2, . . . is defined as follows:

xi = 2i if 0 ≤ i ≤ m− 1 and xi =∑m

j=1 xi−j , if i ≥ m.

2 Each positive integer a is subjected to the following procedure, yielding the number d = d (a):

(a) The last digit of a is moved to the first position. The resulting number is called b. (b)The number b is squared. The resulting number is called c. (c) The first digit of c is movedto the last position. The resulting number is called d.

(All numbers are considered in the decimal system.) For instance, a = 2003 gives b = 3200,c = 10240000 and d = 02400001 = 2400001 = d (2003).

Find all integers a such that d (a) = a2.

3 Determine all pairs of positive integers (a, b) such that

a2

2ab2 − b3 + 1

is a positive integer.

4 Let b be an integer greater than 5. For each positive integer n, consider the number

xn = 11 · · · 1︸ ︷︷ ︸n−1

22 · · · 2︸ ︷︷ ︸n

5,

written in base b.

Prove that the following condition holds if and only if b = 10:

there exists a positive integer M such that for any integer n greater than M , the number xn

is a perfect square.

5 An integer n is said to be good if |n| is not the square of an integer. Determine all integersm with the following property: m can be represented, in infinitely many ways, as a sum ofthree distinct good integers whose product is the square of an odd integer.

6 Let p be a prime number. Prove that there exists a prime number q such that for everyinteger n, the number np − p is not divisible by q.

7 The sequence a0, a1, a2, . . . is defined as follows: a0 = 2, ak+1 = 2a2k−1 for k ≥ 0. Prove

that if an odd prime p divides an, then 2n+3 divides p2 − 1.

http://www.artofproblemsolving.com/This file was downloaded from the AoPS Math Olympiad Resources Page Page 7

Page 178: យុវសិស្ ឈឹ ទីុយ IMO Shortlist 1959-2009 · IMO Shortlist 1959 1 Prove that the fraction 21n+4 14n+3 ... Let α be the acute angel subtending, from A, that

IMO Shortlist 2003

[hide=”comment”] Hi guys ,

Here is a nice problem:

Let be given a sequence an such that a0 = 2 and an+1 = 2a2n − 1 . Show that if p is an odd

prime such that p|an then we have p2 ≡ 1 (mod 2n+3)

Here are some futher question proposed by me ¡!– s:P –¿¡img src=”SMILIESP ATH/tongue.gif”alt =” : P”title = ”Razz”/ ><!−−s : P −− > roveordisprovethat : 1)gcd(n,an) = 1 2) for everyodd prime number p we have am ≡ ±1 (mod p) where m = p2−1

2k where k = 1 or 2

Thanks kiu si u

Edited by Orl.

8 Let p be a prime number and let A be a set of positive integers that satisfies the followingconditions: (1) the set of prime divisors of the elements in A consists of p− 1 elements; (2)for any nonempty subset of A, the product of its elements is not a perfect p-th power. Whatis the largest possible number of elements in A ?

http://www.artofproblemsolving.com/This file was downloaded from the AoPS Math Olympiad Resources Page Page 8

Page 179: យុវសិស្ ឈឹ ទីុយ IMO Shortlist 1959-2009 · IMO Shortlist 1959 1 Prove that the fraction 21n+4 14n+3 ... Let α be the acute angel subtending, from A, that

IMO Shortlist 2004

Algebra

1 Let n ≥ 3 be an integer. Let t1, t2, ..., tn be positive real numbers such that

n2 + 1 > (t1 + t2 + ... + tn)(

1t1

+ 1t2

+ ... + 1tn

).

Show that ti, tj , tk are side lengths of a triangle for all i, j, k with 1 ≤ i < j < k ≤ n.

2 Let a0, a1, a2, ... be an infinite sequence of real numbers satisfying the equation an =|an+1 − an+2| for all n ≥ 0, where a0 and a1 are two different positive reals.

Can this sequence a0, a1, a2, ... be bounded?

[hide=”Remark”] This one is from the IMO Shortlist 2004, but it’s already published on the[url=http://www.bundeswettbewerb-mathematik.de/imo/aufgaben/aufgaben.htm]official BWMwebsite[/url] und thus I take the freedom to post it here:

3 Does there exist a function s : Q → −1, 1 such that if x and y are distinct rational numberssatisfying xy = 1 or x + y ∈ 0, 1, then s(x)s(y) = −1? Justify your answer.

[hide=”comment”] Edited by orl.

4 Find all polynomials f with real coefficients such that for all reals a, b, c such that ab+bc+ca =0 we have the following relations

f(a− b) + f(b− c) + f(c− a) = 2f(a + b + c).

5 If a, b ,c are three positive real numbers such that ab + bc + ca = 1, prove that

3

√1a

+ 6b + 3

√1b

+ 6c + 3

√1c

+ 6a ≤ 1abc

.

6 Find all functions f : R → R satisfying the equation

f(x2 + y2 + 2f (xy)

)= (f (x + y))2

for all x, y ∈ R.

[hide=”comment”] Edited by orl.

7 Let a1, a2, . . . , an be positive real numbers, n > 1. Denote by gn their geometric mean, andby A1, A2, . . . , An the sequence of arithmetic means defined by

Ak = a1+a2+···+akk , k = 1, 2, . . . , n.

Let Gn be the geometric mean of A1, A2, . . . , An. Prove the inequality

http://www.artofproblemsolving.com/This file was downloaded from the AoPS Math Olympiad Resources Page Page 1

Page 180: យុវសិស្ ឈឹ ទីុយ IMO Shortlist 1959-2009 · IMO Shortlist 1959 1 Prove that the fraction 21n+4 14n+3 ... Let α be the acute angel subtending, from A, that

IMO Shortlist 2004

n n

√GnAn

+ gn

Gn≤ n + 1

and establish the cases of equality.

http://www.artofproblemsolving.com/This file was downloaded from the AoPS Math Olympiad Resources Page Page 2

Page 181: យុវសិស្ ឈឹ ទីុយ IMO Shortlist 1959-2009 · IMO Shortlist 1959 1 Prove that the fraction 21n+4 14n+3 ... Let α be the acute angel subtending, from A, that

IMO Shortlist 2004

Combinatorics

1 There are 10001 students at an university. Some students join together to form several clubs(a student may belong to different clubs). Some clubs join together to form several societies(a club may belong to different societies). There are a total of k societies. Suppose that thefollowing conditions hold:

i.) Each pair of students are in exactly one club.

ii.) For each student and each society, the student is in exactly one club of the society.

iii.) Each club has an odd number of students. In addition, a club with 2m + 1 students (mis a positive integer) is in exactly m societies.

Find all possible values of k. [hide=”Remark”]In IMOTC 2005, it is given that m = 2005.

2 Let n and k be positive integers. There are given n circles in the plane. Every two of themintersect at two distinct points, and all points of intersection they determine are pairwiselydistinct (i. e. no three circles have a common point). No three circles have a point in common.Each intersection point must be colored with one of n distinct colors so that each color isused at least once and exactly k distinct colors occur on each circle. Find all values of n ≥ 2and k for which such a coloring is possible.

[hide=”comment”] This one is from the IMO Shortlist 2004, but it’s already published on the[url=http://www.bundeswettbewerb-mathematik.de/imo/aufgaben/aufgaben.htm]official BWMwebsite[/url] und thus I take the freedom to post it here.

Edited by orl.

3 In a country there are n > 3 cities. We can add a street between 2 cities A and B, iff thereexist 2 cities X and Y different from A and B such that there is no street between A and X,X and Y , and Y and B. Find the biggest number of streets one can construct!

[hide=”Official Wording”] The following operation is allowed on a finite graph: Choose anarbitrary cycle of length 4 (if there is any), choose an arbitrary edge in that cycle, and deleteit from the graph. For a fixed integer n ≥ 4, find the least number of edges of a graph thatcan be obtained by repeated applications of this operation from the complete graph on nvertices (where each pair of vertices are joined by an edge).

4 Consider a matrix of size n×n whose entries are real numbers of absolute value not exceeding1. The sum of all entries of the matrix is 0. Let n be an even positive integer. Determine theleast number C such that every such matrix necessarily has a row or a column with the sumof its entries not exceeding C in absolute value.

http://www.artofproblemsolving.com/This file was downloaded from the AoPS Math Olympiad Resources Page Page 3

Page 182: យុវសិស្ ឈឹ ទីុយ IMO Shortlist 1959-2009 · IMO Shortlist 1959 1 Prove that the fraction 21n+4 14n+3 ... Let α be the acute angel subtending, from A, that

IMO Shortlist 2004

5 3. A and B play a game, given an integer N , A writes down 1 first, then every player seesthe last number written and if it is n then in his turn he writes n + 1 or 2n, but his numbercannot be bigger than N . The player who writes N wins!, For wich values of N does B wins?

6 For an n× n matrix A, let Xi be the set of entries in row i, and Yj the set of entries incolumn j, 1 ≤ i, j ≤ n. We say that A is golden if X1, . . . , Xn, Y1, . . . , Yn are distinct sets.Find the least integer n such that there exists a 2004× 2004 golden matrix with entries inthe set 1, 2, . . . , n.[hide=”comment”] That’s problem 3 of the 2nd German TST 2005 and MoldovaTST 2005. Edited by orl.

7 Define a ”hook” to be a figure made up of six unit squares as shown below in the pic-ture, or any of the figures obtained by applying rotations and reflections to this figure.[img]http://www.mathlinks.ro/Forum/files/hook.png[/img] Determine all m × n rectanglesthat can be covered without gaps and without overlaps with hooks such that

- the rectangle is covered without gaps and without overlaps - no part of a hook covers areaoutside the rectagle.

8 For a finite graph G, let f(G) be the number of triangles and g(G) the number of tetrahedraformed by edges of G. Find the least constant c such that

g(G)3 ≤ c · f(G)4

for every graph G.

http://www.artofproblemsolving.com/This file was downloaded from the AoPS Math Olympiad Resources Page Page 4

Page 183: យុវសិស្ ឈឹ ទីុយ IMO Shortlist 1959-2009 · IMO Shortlist 1959 1 Prove that the fraction 21n+4 14n+3 ... Let α be the acute angel subtending, from A, that

IMO Shortlist 2004

Geometry

1 1. Let ABC be an acute-angled triangle with AB 6= AC. The circle with diameter BCintersects the sides AB and AC at M and N respectively. Denote by O the midpoint of theside BC. The bisectors of the angles ∠BAC and ∠MON intersect at R. Prove that thecircumcircles of the triangles BMR and CNR have a common point lying on the side BC.

2 Let Γ be a circle and let d be a line such that Γ and d have no common points. Further, letAB be a diameter of the circle Γ; assume that this diameter AB is perpendicular to the lined, and the point B is nearer to the line d than the point A. Let C be an arbitrary point onthe circle Γ, different from the points A and B. Let D be the point of intersection of the linesAC and d. One of the two tangents from the point D to the circle Γ touches this circle Γ ata point E; hereby, we assume that the points B and E lie in the same halfplane with respectto the line AC. Denote by F the point of intersection of the lines BE and d. Let the line AFintersect the circle Γ at a point G, different from A.

Prove that the reflection of the point G in the line AB lies on the line CF .

3 Let ABC be an acute-angled triangle such that ∠ABC < ∠ACB, let O be the circumcenterof triangle ABC, and let D = AO ∩BC. Denote by E and F the circumcenters of trianglesABD and ACD, respectively. Let G be a point on the extension of the segment AB beyoundA such that AG = AC, and let H be a point on the extension of the segment AC beyoundA such that AH = AB. Prove that the quadrilateral EFGH is a rectangle if and only if∠ACB − ∠ABC = 60.

[hide=”comment”] [hide=”Official version”] Let O be the circumcenter of an acute-angledtriangle ABC with ∠B < ∠C. The line AO meets the side BC at D. The circumcenters of thetriangles ABD and ACD are E and F , respectively. Extend the sides BA and CA beyond A,and choose on the respective extensions points G and H such that AG = AC and AH = AB.Prove that the quadrilateral EFGH is a rectangle if and only if ∠ACB − ∠ABC = 60.

Edited by orl.

4 In a convex quadrilateral ABCD, the diagonal BD bisects neither the angle ABC nor theangle CDA. The point P lies inside ABCD and satisfies

∠PBC = ∠DBA and ∠PDC = ∠BDA.

Prove that ABCD is a cyclic quadrilateral if and only if AP = CP .

5 Let A1A2A3...An be a regular n-gon. Let B1 and Bn be the midpoints of its sides A1A2 andAn−1An. Also, for every i ∈ 2; 3; 4; ...; n− 1, let S be the point of intersection of the

http://www.artofproblemsolving.com/This file was downloaded from the AoPS Math Olympiad Resources Page Page 5

Page 184: យុវសិស្ ឈឹ ទីុយ IMO Shortlist 1959-2009 · IMO Shortlist 1959 1 Prove that the fraction 21n+4 14n+3 ... Let α be the acute angel subtending, from A, that

IMO Shortlist 2004

lines A1Ai+1 and AnAi, and let Bi be the point of intersection of the angle bisector bisectorof the angle ]AiSAi+1 with the segment AiAi+1.

Prove that:∑n−1

i=1 ]A1BiAn = 180.

6 Let P be a convex polygon. Prove that there exists a convex hexagon that is contained in Pand whose area is at least 3

4 of the area of the polygon P .

Alternative version. Let P be a convex polygon with n ≥ 6 vertices. Prove that there existsa convex hexagon with

a) vertices on the sides of the polygon (or) b) vertices among the vertices of the polygon

such that the area of the hexagon is at least 34 of the area of the polygon.

I couldn’t solve this one, partially because I’m not quite sure of the statements’ meaning (a) or(b) ¡!– s:) –¿¡img src=”SMILIESP ATH/smile.gif”alt = ” :)”title = ”Smile”/ ><!−−s :)−− > Obviouslyifa)istruethensoisb).Foragiventriangle ABC, let XbeavariablepointonthelineBCsuchthat Cliesbetween Band Xandtheincirclesofthetriangles ABXand ACXintersectattwodistinctpointsPand Q.Provethattheline PQpassesthroughapointindependentof X.

[hide=”comment”] An extension by [url=http://www.mathlinks.ro/Forum/profile.php?mode=viewprofileu=432]DarijGrinberg[/url] can be found [url=http://www.mathlinks.ro/Forum/viewtopic.php?t=40772]here.[/url]

78 Given a cyclic quadrilateral ABCD, let M be the midpoint of the side CD, and let N bea point on the circumcircle of triangle ABM . Assume that the point N is different fromthe point M and satisfies AN

BN = AMBM . Prove that the points E, F , N are collinear, where

E = AC ∩BD and F = BC ∩DA.

http://www.artofproblemsolving.com/This file was downloaded from the AoPS Math Olympiad Resources Page Page 6

Page 185: យុវសិស្ ឈឹ ទីុយ IMO Shortlist 1959-2009 · IMO Shortlist 1959 1 Prove that the fraction 21n+4 14n+3 ... Let α be the acute angel subtending, from A, that

IMO Shortlist 2004

Number Theory

1 Let τ(n) denote the number of positive divisors of the positive integer n. Prove that thereexist infinitely many positive integers a such that the equation τ(an) = n does not have apositive integer solution n.

2 The function f from the set N of positive integers into itself is defined by the equality

f(n) =∑n

k=1 gcd(k, n), n ∈ N,

a) Prove that f(mn) = f(m)f(n) for every two relatively prime m,n ∈ N.

b) Prove that for each a ∈ N the equation f(x) = ax has a solution.

c) Find all a ∈ N such that the equation f(x) = ax has a unique solution.

[hide=”comment”] Edited by orl.

3 Find all functions f : N∗ → N∗ satisfying(f2 (m) + f (n)

)|(m2 + n

)2

for any two positive integers m and n.

Remark. The abbreviation N∗ stands for the set of all positive integers: N∗ = 1, 2, 3, .... Byf2 (m), we mean (f (m))2 (and not f (f (m))).

4 Let k be a fixed integer greater than 1, and let m = 4k2 − 5. Show that there exist positiveintegers a and b such that the sequence (xn) defined by

x0 = a, x1 = b, xn+2 = xn+1 + xn for n = 0, 1, 2, . . . ,

has all of its terms relatively prime to m.

5 We call a positive integer alternating if every two consecutive digits in its decimal represen-tation are of different parity.

Find all positive integers n such that n has a multiple which is alternating.

6 Given an integer n > 1, denote by Pn the product of all positive integers x less than n andsuch that n divides x2 − 1. For each n > 1, find the remainder of Pn on division by n.

7 Let p be an odd prime and n a positive integer. In the coordinate plane, eight distinct pointswith integer coordinates lie on a circle with diameter of length pn. Prove that there exists atriangle with vertices at three of the given points such that the squares of its side lengths areintegers divisible by pn+1.

http://www.artofproblemsolving.com/This file was downloaded from the AoPS Math Olympiad Resources Page Page 7

Page 186: យុវសិស្ ឈឹ ទីុយ IMO Shortlist 1959-2009 · IMO Shortlist 1959 1 Prove that the fraction 21n+4 14n+3 ... Let α be the acute angel subtending, from A, that

IMO Shortlist 2005

Algebra

1 Find all pairs of integers a, b for which there exists a polynomial P (x) ∈ Z[X] such thatproduct (x2 + ax + b) · P (x) is a polynomial of a form

xn + cn−1xn−1 + ... + c1x + c0

where each of c0, c1, ..., cn−1 is equal to 1 or −1.

2 We denote by R+ the set of all positive real numbers.

Find all functions f : R+ → R+ which have the property:

f (x) f (y) = 2f (x + yf (x))

for all positive real numbers x and y.

3 Four real numbers p, q, r, s satisfy p + q + r + s = 9 and p2 + q2 + r2 + s2 = 21. Prove thatthere exists a permutation (a, b, c, d) of (p, q, r, s) such that ab− cd ≥ 2.

4 Find all functions f : R → R such that f (x + y) + f (x) f (y) = f (xy) + 2xy + 1 for all realnumbers x and y.

5 Let x, y, z be three positive reals such that xyz ≥ 1. Prove that

x5 − x2

x5 + y2 + z2+

y5 − y2

x2 + y5 + z2+

z5 − z2

x2 + y2 + z5≥ 0.

Hojoo Lee, Korea

http://www.artofproblemsolving.com/This file was downloaded from the AoPS Math Olympiad Resources Page Page 1

Page 187: យុវសិស្ ឈឹ ទីុយ IMO Shortlist 1959-2009 · IMO Shortlist 1959 1 Prove that the fraction 21n+4 14n+3 ... Let α be the acute angel subtending, from A, that

IMO Shortlist 2005

Combinatorics

1 A house has an even number of lamps distributed among its rooms in such a way that thereare at least three lamps in every room. Each lamp shares a switch with exactly one otherlamp, not necessarily from the same room. Each change in the switch shared by two lampschanges their states simultaneously. Prove that for every initial state of the lamps there existsa sequence of changes in some of the switches at the end of which each room contains lampswhich are on as well as lamps which are off.

2 This ISL 2005 problem has not been used in any TST I know. A pity, since it is a nice problem,but in its shortlist formulation, it is absolutely incomprehensible. Here is a mathematicalrestatement of the problem:

A forest consists of rooted (i. e. oriented) trees. Each vertex of the forest is either a leafor has two successors. A vertex v is called an extended successor of a vertex u if there is achain of vertices u0 = u, u1, u2, ..., ut−1, ut = v such that the vertex ui+1 is a successor ofthe vertex ui for every integer i with 0 ≤ i ≤ t − 1. A vertex is called dynastic if it has twosuccessors and each of these successors has at least k extended successors.

Prove that if the forest has n vertices, then there are at most nk+2 dynastic vertices.

3 Consider a m × n rectangular board consisting of mn unit squares. Two of its unit squaresare called adjacent if they have a common edge, and a path is a sequence of unit squares inwhich any two consecutive squares are adjacent. Two parths are called non-intersecting ifthey don’t share any common squares.

Each unit square of the rectangular board can be colored black or white. We speak of acoloring of the board if all its mn unit squares are colored.

Let N be the number of colorings of the board such that there exists at least one black pathfrom the left edge of the board to its right edge. Let M be the number of colorings of theboard for which there exist at least two non-intersecting black paths from the left edge of theboard to its right edge.

Prove that N2 ≥ M · 2mn.

4 Let n ≥ 3 be a fixed integer. Each side and each diagonal of a regular n-gon is labelled with anumber from the set 1; 2; ...; r in a way such that the following two conditions are fulfilled:

1. Each number from the set 1; 2; ...; r occurs at least once as a label.

2. In each triangle formed by three vertices of the n-gon, two of the sides are labelled withthe same number, and this number is greater than the label of the third side.

(a) Find the maximal r for which such a labelling is possible.

http://www.artofproblemsolving.com/This file was downloaded from the AoPS Math Olympiad Resources Page Page 2

Page 188: យុវសិស្ ឈឹ ទីុយ IMO Shortlist 1959-2009 · IMO Shortlist 1959 1 Prove that the fraction 21n+4 14n+3 ... Let α be the acute angel subtending, from A, that

IMO Shortlist 2005

(b) Harder version (IMO Shortlist 2005): For this maximal value of r, how many suchlabellings are there?

[hide=”Easier version (5th German TST 2006) - contains answer to the harder version”]Easier version (5th German TST 2006): Show that, for this maximal value of r, there areexactly n!(n−1)!

2n−1 possible labellings.

5 There are n markers, each with one side white and the other side black. In the beginning,these n markers are aligned in a row so that their white sides are all up. In each step, ifpossible, we choose a marker whose white side is up (but not one of the outermost markers),remove it, and reverse the closest marker to the left of it and also reverse the closest markerto the right of it. Prove that, by a finite sequence of such steps, one can achieve a state withonly two markers remaining if and only if n− 1 is not divisible by 3.

6 In a mathematical competition, in which 6 problems were posed to the participants, everytwo of these problems were solved by more than 2

5 of the contestants. Moreover, no contestantsolved all the 6 problems. Show that there are at least 2 contestants who solved exactly 5problems each.

Radu Gologan and Dan Schwartz

7 Suppose that a1, a2, . . ., an are integers such that n | a1 + a2 + . . . + an. Prove that thereexist two permutations (b1, b2, . . . , bn) and (c1, c2, . . . , cn) of (1, 2, . . . , n) such that for eachinteger i with 1 ≤ i ≤ n, we have

n | ai − bi − ci

8 Suppose we have a n-gon. Some n − 3 diagonals are coloured black and some other n − 3diagonals are coloured red (a side is not a diagonal), so that no two diagonals of the samecolour can intersect strictly inside the polygon, although they can share a vertex. Find themaximum number of intersection points between diagonals coloured differently strictly insidethe polygon, in terms of n.

http://www.artofproblemsolving.com/This file was downloaded from the AoPS Math Olympiad Resources Page Page 3

Page 189: យុវសិស្ ឈឹ ទីុយ IMO Shortlist 1959-2009 · IMO Shortlist 1959 1 Prove that the fraction 21n+4 14n+3 ... Let α be the acute angel subtending, from A, that

IMO Shortlist 2005

Geometry

1 Given a triangle ABC satisfying AC +BC = 3 ·AB. The incircle of triangle ABC has centerI and touches the sides BC and CA at the points D and E, respectively. Let K and L bethe reflections of the points D and E with respect to I. Prove that the points A, B, K, L lieon one circle.

2 Six points are chosen on the sides of an equilateral triangle ABC: A1, A2 on BC, B1, B2 onCA and C1, C2 on AB, such that they are the vertices of a convex hexagon A1A2B1B2C1C2

with equal side lengths.

Prove that the lines A1B2, B1C2 and C1A2 are concurrent.

Bogdan Enescu, Romania

3 Let ABCD be a parallelogram. A variable line g through the vertex A intersects the raysBC and DC at the points X and Y , respectively. Let K and L be the A-excenters of thetriangles ABX and ADY . Show that the angle ]KCL is independent of the line g.

4 Let ABCD be a fixed convex quadrilateral with BC = DA and BC not parallel with DA. Lettwo variable points E and F lie of the sides BC and DA, respectively and satisfy BE = DF .The lines AC and BD meet at P , the lines BD and EF meet at Q, the lines EF and ACmeet at R.

Prove that the circumcircles of the triangles PQR, as E and F vary, have a common pointother than P .

5 Let 4ABC be an acute-angled triangle with AB 6= AC. Let H be the orthocenter of triangleABC, and let M be the midpoint of the side BC. Let D be a point on the side AB and Ea point on the side AC such that AE = AD and the points D, H, E are on the same line.Prove that the line HM is perpendicular to the common chord of the circumscribed circlesof triangle 4ABC and triangle 4ADE.

[url=http://www.mathlinks.ro/Forum/viewtopic.php?t=89094]All the problems from the Swiss Imo Selection Team 2006[/url]

6 Let ABC be a triangle, and M the midpoint of its side BC. Let γ be the incircle of triangleABC. The median AM of triangle ABC intersects the incircle γ at two points K and L.Let the lines passing through K and L, parallel to BC, intersect the incircle γ again in twopoints X and Y . Let the lines AX and AY intersect BC again at the points P and Q. Provethat BP = CQ.

7 In an acute triangle ABC, let D, E, F be the feet of the perpendiculars from the points A, B,C to the lines BC, CA, AB, respectively, and let P , Q, R be the feet of the perpendicularsfrom the points A, B, C to the lines EF , FD, DE, respectively.

http://www.artofproblemsolving.com/This file was downloaded from the AoPS Math Olympiad Resources Page Page 4

Page 190: យុវសិស្ ឈឹ ទីុយ IMO Shortlist 1959-2009 · IMO Shortlist 1959 1 Prove that the fraction 21n+4 14n+3 ... Let α be the acute angel subtending, from A, that

IMO Shortlist 2005

Prove that p (ABC) p (PQR) ≥ (p (DEF ))2, where p (T ) denotes the perimeter of triangle T.

http://www.artofproblemsolving.com/This file was downloaded from the AoPS Math Olympiad Resources Page Page 5

Page 191: យុវសិស្ ឈឹ ទីុយ IMO Shortlist 1959-2009 · IMO Shortlist 1959 1 Prove that the fraction 21n+4 14n+3 ... Let α be the acute angel subtending, from A, that

IMO Shortlist 2005

Number Theory

1 Determine all positive integers relatively prime to all the terms of the infinite sequence

an = 2n + 3n + 6n − 1, n ≥ 1.

2 Let a1, a2, . . . be a sequence of integers with infinitely many positive and negative terms. Sup-pose that for every positive integer n the numbers a1, a2, . . . , an leave n different remaindersupon division by n.

Prove that every integer occurs exactly once in the sequence a1, a2, . . ..

3 Let a, b, c, d, e, f be positive integers and let S = a + b + c + d + e + f . Suppose that thenumber S divides abc + def and ab + bc + ca− de− ef − df . Prove that S is composite.

4 Find all positive integers n such that there exists a unique integer a such that 0 ≤ a < n!with the following property:

n! | an + 1

5 Denote by d(n) the number of divisors of the positive integer n. A positive integer n is calledhighly divisible if d(n) > d(m) for all positive integers m < n. Two highly divisible integers mand n with m < n are called consecutive if there exists no highly divisible integer s satisfyingm < s < n.

(a) Show that there are only finitely many pairs of consecutive highly divisible integers of theform (a, b) with a | b.(b) Show that for every prime number p there exist infinitely many positive highly divisibleintegers r such that pr is also highly divisible.

6 Let a, b be positive integers such that bn + n is a multiple of an + n for all positive integersn. Prove that a = b.

7 Let P (x) = anxn + an−1xn−1 + . . . + a0, where a0, . . . , an are integers, an > 0, n ≥ 2. Prove

that there exists a positive integer m such that P (m!) is a composite number.

http://www.artofproblemsolving.com/This file was downloaded from the AoPS Math Olympiad Resources Page Page 6

Page 192: យុវសិស្ ឈឹ ទីុយ IMO Shortlist 1959-2009 · IMO Shortlist 1959 1 Prove that the fraction 21n+4 14n+3 ... Let α be the acute angel subtending, from A, that

IMO Shortlist 2006

Algebra

1 a0, a1, a2, . . . is a sequence of real numbers such that

an+1 = [an] · an

prove that exist j such that for every i ≥ j we have ai+2 = ai.

2 Let a0, a1, a2, ... be a sequence of reals such that a0 = − 1 and

an + an−1

2 + an−2

3 + ... + a1n + a0

n+1 = 0 for all n ≥ 1.

Show that an > 0 for all n ≥ 1.

3 The sequence c0, c1, ..., cn, ... is defined by c0 = 1, c1 = 0, and cn+2 = cn+1 + cn for n ≥ 0.Consider the set S of ordered pairs (x, y) for which there is a finite set J of positive integerssuch that x =

∑j∈J cj , y =

∑j∈J cj−1. Prove that there exist real numbers α, β, and M with

the following property: An ordered pair of nonnegative integers (x, y) satisfies the inequalitym < αx + βy < M if and only if (x, y) ∈ S.

Remark: A sum over the elements of the empty set is assumed to be 0.

4 Prove the inequality:∑i<j

aiaj

ai+aj≤ n

2(a1+a2+...+an) ·∑

i<j aiaj

for positive reals a1, a2, ..., an.

5 If a, b, c are the sides of a triangle, prove that

∑cyc

√a + b− c

√a +

√b−

√c≤ 3

6 Determine the least real number M such that the inequality∣∣∣ab

(a2 − b2

)+ bc

(b2 − c2

)+ ca

(c2 − a2

)| ≤ M

(a2 + b2 + c2

)2

holds for all real numbers a, b and c.

http://www.artofproblemsolving.com/This file was downloaded from the AoPS Math Olympiad Resources Page Page 1

Page 193: យុវសិស្ ឈឹ ទីុយ IMO Shortlist 1959-2009 · IMO Shortlist 1959 1 Prove that the fraction 21n+4 14n+3 ... Let α be the acute angel subtending, from A, that

IMO Shortlist 2006

Combinatorics

1 We have n ≥ 2 lamps L1, ..., Ln in a row, each of them being either on or off. Every secondwe simultaneously modify the state of each lamp as follows: if the lamp Li and its neighbours(only one neighbour for i = 1 or i = n, two neighbours for other i) are in the same state,then Li is switched off; otherwise, Li is switched on. Initially all the lamps are off exceptthe leftmost one which is on.

(a) Prove that there are infinitely many integers n for which all the lamps will eventually beoff. (b) Prove that there are infinitely many integers n for which the lamps will never be alloff.

2 Let P be a regular 2006-gon. A diagonal is called good if its endpoints divide the boundaryof P into two parts, each composed of an odd number of sides of P . The sides of P are alsocalled good. Suppose P has been dissected into triangles by 2003 diagonals, no two of whichhave a common point in the interior of P . Find the maximum number of isosceles triangleshaving two good sides that could appear in such a configuration.

3 Let S be a finite set of points in the plane such that no three of them are on a line. For eachconvex polygon P whose vertices are in S, let a(P ) be the number of vertices of P , and letb(P ) be the number of points of S which are outside P . A line segment, a point, and theempty set are considered as convex polygons of 2, 1, and 0 vertices respectively. Prove thatfor every real number x:∑

P xa(P )(1− x)b(P ) = 1, where the sum is taken over all convex polygons with vertices in S.

Alternative formulation:

Let M be a finite point set in the plane and no three points are collinear. A subset A of Mwill be called round if its elements is the set of vertices of a convex A−gon V (A). For eachround subset let r(A) be the number of points from M which are exterior from the convexA−gon V (A). Subsets with 0, 1 and 2 elements are always round, its corresponding polygonsare the empty set, a point or a segment, respectively (for which all other points that are notvertices of the polygon are exterior). For each round subset A of M construct the polynomial

PA(x) = x|A|(1− x)r(A).

Show that the sum of polynomials for all round subsets is exactly the polynomial P (x) = 1.

4 A cake has the form of an n x n square composed of n2 unit squares. Strawberries lie onsome of the unit squares so that each row or column contains exactly one strawberry; call thisarrangement A. Let B be another such arrangement. Suppose that every grid rectangle with

http://www.artofproblemsolving.com/This file was downloaded from the AoPS Math Olympiad Resources Page Page 2

Page 194: យុវសិស្ ឈឹ ទីុយ IMO Shortlist 1959-2009 · IMO Shortlist 1959 1 Prove that the fraction 21n+4 14n+3 ... Let α be the acute angel subtending, from A, that

IMO Shortlist 2006

one vertex at the top left corner of the cake contains no fewer strawberries of arrangement Bthan of arrangement A.

Prove that arrangement B can be obtained from A by performing a number of switches,defined as follows: A switch consists in selecting a grid rectangle with only two strawberries,situated at its top right corner and bottom left corner, and moving these two strawberries tothe other two corners of that rectangle.

5 An (n, k)− tournament is a contest with n players held in k rounds such that:

(i) Each player plays in each round, and every two players meet at most once. (ii) If playerA meets player B in round i, player C meets player D in round i, and player A meets playerC in round j, then player B meets player D in round j.

Determine all pairs (n, k) for which there exists an (n, k)− tournament.

6 A holey triangle is an upward equilateral triangle of side length n with n upward unit triangu-lar holes cut out. A diamond is a 60−120 unit rhombus. Prove that a holey triangle T canbe tiled with diamonds if and only if the following condition holds: Every upward equilateraltriangle of side length k in T contains at most k holes, for 1 ≤ k ≤ n.

7 Consider a convex polyheadron without parallel edges and without an edge parallel to anyface other than the two faces adjacent to it. Call a pair of points of the polyheadron antipodalif there exist two parallel planes passing through these points and such that the polyheadronis contained between these planes. Let A be the number of antipodal pairs of vertices, andlet B be the number of antipodal pairs of midpoint edges. Determine the difference A−B interms of the numbers of vertices, edges, and faces.

http://www.artofproblemsolving.com/This file was downloaded from the AoPS Math Olympiad Resources Page Page 3

Page 195: យុវសិស្ ឈឹ ទីុយ IMO Shortlist 1959-2009 · IMO Shortlist 1959 1 Prove that the fraction 21n+4 14n+3 ... Let α be the acute angel subtending, from A, that

IMO Shortlist 2006

Geometry

1 Let ABC be triangle with incenter I. A point P in the interior of the triangle satisfies

∠PBA + ∠PCA = ∠PBC + ∠PCB.

Show that AP ≥ AI, and that equality holds if and only if P = I.

2 Let ABC be a trapezoid with parallel sides AB > CD. Points K and L lie on the linesegments AB and CD, respectively, so that AK

KB = DLLC . Suppose that there are points P and

Q on the line segment KL satisfying ∠APB = ∠BCD and ∠CQD = ∠ABC. Prove thatthe points P , Q, B and C are concylic.

3 Consider a convex pentagon ABCDE such that

∠BAC = ∠CAD = ∠DAE , ∠ABC = ∠ACD = ∠ADE

Let P be the point of intersection of the lines BD and CE. Prove that the line AP passesthrough the midpoint of the side CD.

4 Let ABC be a triangle such that ACB < BAC < π2 . Let D be a point of [AC] such that

BD = BA. The incircle of ABC touches [AB] at K and [AC] at L. Let J be the center ofthe incircle of BCD. Prove that (KL) intersects [AJ ] at its middle.

5 In triangle ABC, let J be the center of the excircle tangent to side BC at A1 and to theextensions of the sides AC and AB at B1 and C1 respectively. Suppose that the lines A1B1

and AB are perpendicular and intersect at D. Let E be the foot of the perpendicular fromC1 to line DJ . Determine the angles ∠BEA1 and ∠AEB1.

6 Circles w1 and w2 with centres O1 and O2 are externally tangent at point D and internallytangent to a circle w at points E and F respectively. Line t is the common tangent of w1 andw2 at D. Let AB be the diameter of w perpendicular to t, so that A,E, O1 are on the sameside of t. Prove that lines AO1, BO2, EF and t are concurrent.

7 In a triangle ABC, let Ma, Mb, Mc be the midpoints of the sides BC, CA, AB, respectively,and Ta, Tb, Tc be the midpoints of the arcs BC, CA, AB of the circumcircle of ABC, notcontaining the vertices A, B, C, respectively. For i ∈ a, b, c, let wi be the circle with MiTi

as diameter. Let pi be the common external common tangent to the circles wj and wk (forall i, j, k = a, b, c) such that wi lies on the opposite side of pi than wj and wk do. Provethat the lines pa, pb, pc form a triangle similar to ABC and find the ratio of similitude.

http://www.artofproblemsolving.com/This file was downloaded from the AoPS Math Olympiad Resources Page Page 4

Page 196: យុវសិស្ ឈឹ ទីុយ IMO Shortlist 1959-2009 · IMO Shortlist 1959 1 Prove that the fraction 21n+4 14n+3 ... Let α be the acute angel subtending, from A, that

IMO Shortlist 2006

8 Let ABCD be a convex quadrilateral. A circle passing through the points A and D and acircle passing through the points B and C are externally tangent at a point P inside thequadrilateral. Suppose that ∠PAB + ∠PDC ≤ 90 and ∠PBA + ∠PCD ≤ 90. Prove thatAB + CD ≥ BC + AD.

9 Points A1, B1, C1 are chosen on the sides BC, CA, AB of a triangle ABC respectively. Thecircumcircles of triangles AB1C1, BC1A1, CA1B1 intersect the circumcircle of triangle ABCagain at points A2, B2, C2 respectively (A2 6= A,B2 6= B,C2 6= C). Points A3, B3, C3 aresymmetric to A1, B1, C1 with respect to the midpoints of the sides BC, CA, AB respectively.Prove that the triangles A2B2C2 and A3B3C3 are similar.

[hide=”Comment”]This is my personal favourite of the ISL Geometry problems ¡!– s:D –¿¡img src=”SMILIESP ATH/iconmrgreen.gif”alt = ” : D”title = ”Mr.Green”/ ><!−−s :D −− >

10 Assign to each side b of a convex polygon P the maximum area of a triangle that has b as aside and is contained in P . Show that the sum of the areas assigned to the sides of P is atleast twice the area of P .

http://www.artofproblemsolving.com/This file was downloaded from the AoPS Math Olympiad Resources Page Page 5

Page 197: យុវសិស្ ឈឹ ទីុយ IMO Shortlist 1959-2009 · IMO Shortlist 1959 1 Prove that the fraction 21n+4 14n+3 ... Let α be the acute angel subtending, from A, that

IMO Shortlist 2006

Number Theory

1 Determine all pairs (x, y) of integers such that

1 + 2x + 22x+1 = y2.

2 For x ∈ (0, 1) let y ∈ (0, 1) be the number whose n-th digit after the decimal point is the2n-th digit after the decimal point of x. Show that if x is rational then so is y.

3 We define a sequence (a1, a2, a3, ...) by setting

an =1n

([n

1

]+

[n

2

]+ · · ·+

[n

n

])for every positive integer n. Hereby, for every real x, we denote by [x] the integral part of x(this is the greatest integer which is ≤ x).

a) Prove that there is an infinite number of positive integers n such that an+1 > an. b) Provethat there is an infinite number of positive integers n such that an+1 < an.

4 Let P (x) be a polynomial of degree n > 1 with integer coefficients and let k be a positiveinteger. Consider the polynomial Q(x) = P (P (. . . P (P (x)) . . .)), where P occurs k times.Prove that there are at most n integers t such that Q(t) = t.

5 Prove that the equation x7−1x−1 = y5 − 1 doesn’t have integer solutions!

6 Let a > b > 1 be relatively prime positive integers. Define the weight of an integer c, denotedby w(c) to be the minimal possible value of |x| + |y| taken over all pairs of integers x andy such that ax + by = c. An integer c is called a local champion if w(c) ≥ w(c ± a) andw(c) ≥ w(c± b). Find all local champions and determine their number.

7 For all positive integers n, show that there exists a positive integer m such that n divides2m + m.

http://www.artofproblemsolving.com/This file was downloaded from the AoPS Math Olympiad Resources Page Page 6

Page 198: យុវសិស្ ឈឹ ទីុយ IMO Shortlist 1959-2009 · IMO Shortlist 1959 1 Prove that the fraction 21n+4 14n+3 ... Let α be the acute angel subtending, from A, that

IMO Shortlist 2007

Algebra

1 Real numbers a1, a2, . . ., an are given. For each i, (1 ≤ i ≤ n), define

di = maxaj | 1 ≤ j ≤ i −minaj | i ≤ j ≤ n

and let d = maxdi | 1 ≤ i ≤ n.(a) Prove that, for any real numbers x1 ≤ x2 ≤ · · · ≤ xn,

max|xi − ai| | 1 ≤ i ≤ n ≥ d

2. (∗)

(b) Show that there are real numbers x1 ≤ x2 ≤ · · · ≤ xn such that the equality holds in (*).

Author: Michael Albert, New Zealand

2 Consider those functions f : N 7→ N which satisfy the condition

f(m + n) ≥ f(m) + f(f(n))− 1

for all m,n ∈ N. Find all possible values of f(2007).

Author: unknown author, Bulgaria

3 Let n be a positive integer, and let x and y be a positive real number such that xn + yn = 1.Prove that (

n∑k=1

1 + x2k

1 + x4k

(n∑

k=1

1 + y2k

1 + y4k

)<

1(1− x) · (1− y)

.

Author: unknown author, Estonia

4 Find all functions f : R+ → R+ satisfying f (x + f (y)) = f (x + y) + f (y) for all pairs ofpositive reals x and y. Here, R+ denotes the set of all positive reals.

Author: unknown author, Thailand

5 Let c > 2, and let a(1), a(2), . . . be a sequence of nonnegative real numbers such that

a(m + n) ≤ 2 · a(m) + 2 · a(n) for all m,n ≥ 1,

and a(2k)≤ 1

(k+1)c for all k ≥ 0. Prove that the sequence a(n) is bounded.

Author: Vjekoslav Kova, Croatia

http://www.artofproblemsolving.com/This file was downloaded from the AoPS Math Olympiad Resources Page Page 1

Page 199: យុវសិស្ ឈឹ ទីុយ IMO Shortlist 1959-2009 · IMO Shortlist 1959 1 Prove that the fraction 21n+4 14n+3 ... Let α be the acute angel subtending, from A, that

IMO Shortlist 2007

6 Let a1, a2, . . . , a100 be nonnegative real numbers such that a21 + a2

2 + . . . + a2100 = 1. Prove

thata2

1 · a2 + a22 · a3 + . . . + a2

100 · a1 <1225

.

Author: Marcin Kuzma, Poland

7 Let n be a positive integer. Consider

S = (x, y, z) | x, y, z ∈ 0, 1, . . . , n, x + y + z > 0

as a set of (n+1)3−1 points in the three-dimensional space. Determine the smallest possiblenumber of planes, the union of which contains S but does not include (0, 0, 0).

Author: Gerhard Wginger, Netherlands

http://www.artofproblemsolving.com/This file was downloaded from the AoPS Math Olympiad Resources Page Page 2

Page 200: យុវសិស្ ឈឹ ទីុយ IMO Shortlist 1959-2009 · IMO Shortlist 1959 1 Prove that the fraction 21n+4 14n+3 ... Let α be the acute angel subtending, from A, that

IMO Shortlist 2007

Combinatorics

1 Let n > 1 be an integer. Find all sequences a1, a2, . . . an2+n satisfying the following conditions:

(a) ai ∈ 0, 1 for all 1 ≤ i ≤ n2 + n;

(b) ai+1 + ai+2 + . . . + ai+n < ai+n+1 + ai+n+2 + . . . + ai+2n for all 0 ≤ i ≤ n2 − n.

Author: unknown author, Serbia

2 A rectangle D is partitioned in several (≥ 2) rectangles with sides parallel to those of D.Given that any line parallel to one of the sides of D, and having common points with theinterior of D, also has common interior points with the interior of at least one rectangle ofthe partition; prove that there is at least one rectangle of the partition having no commonpoints with D’s boundary.

Author: unknown author, Japan

3 Find all positive integers n for which the numbers in the set S = 1, 2, . . . , n can be coloredred and blue, with the following condition being satisfied: The set S×S×S contains exactly2007 ordered triples (x, y, z) such that:

(i) the numbers x, y, z are of the same color, and (ii) the number x + y + z is divisible by n.

Author: Gerhard Wginger, Netherlands

4 Let A0 = (a1, . . . , an) be a finite sequence of real numbers. For each k ≥ 0, from the sequenceAk = (x1, . . . , xk) we construct a new sequence Ak+1 in the following way. 1. We choose apartition 1, . . . , n = I ∪ J , where I and J are two disjoint sets, such that the expression∣∣∣∣∣∣

∑i∈I

xi −∑j∈J

xj

∣∣∣∣∣∣attains the smallest value. (We allow I or J to be empty; in this case the corresponding sum is0.) If there are several such partitions, one is chosen arbitrarily. 2. We set Ak+1 = (y1, . . . , yn)where yi = xi + 1 if i ∈ I, and yi = xi − 1 if i ∈ J . Prove that for some k, the sequence Ak

contains an element x such that |x| ≥ n2 .

Author: Omid Hatami, Iran

http://www.artofproblemsolving.com/This file was downloaded from the AoPS Math Olympiad Resources Page Page 3

Page 201: យុវសិស្ ឈឹ ទីុយ IMO Shortlist 1959-2009 · IMO Shortlist 1959 1 Prove that the fraction 21n+4 14n+3 ... Let α be the acute angel subtending, from A, that

IMO Shortlist 2007

5 In the Cartesian coordinate plane define the strips Sn = (x, y)|n ≤ x < n + 1, n ∈ Z andcolor each strip black or white. Prove that any rectangle which is not a square can be placedin the plane so that its vertices have the same color.

[hide=”IMO Shortlist 2007 Problem C5 as it appears in the official booklet:”]In the Cartesiancoordinate plane define the strips Sn = (x, y)|n ≤ x < n + 1 for every integer n. Assumeeach strip Sn is colored either red or blue, and let a and b be two distinct positive integers.Prove that there exists a rectangle with side length a and b such that its vertices have thesame color.

Edited by Orlando Dhring

Author: Radu Gologan and Dan Schwarz, Romania

6 In a mathematical competition some competitors are friends. Friendship is always mutual.Call a group of competitors a clique if each two of them are friends. (In particular, any groupof fewer than two competitiors is a clique.) The number of members of a clique is called itssize.

Given that, in this competition, the largest size of a clique is even, prove that the competitorscan be arranged into two rooms such that the largest size of a clique contained in one roomis the same as the largest size of a clique contained in the other room.

Author: Vasily Astakhov, Russia

7 Let α < 3−√

52 be a positive real number. Prove that there exist positive integers n and

p > α · 2n for which one can select 2 · p pairwise distinct subsets S1, . . . , Sp, T1, . . . , Tp of theset 1, 2, . . . , n such that Si ∩ Tj 6= ∅ for all 1 ≤ i, j ≤ p

Author: Gerhard Wginger, Austria

8 Given is a convex polygon P with n vertices. Triangle whose vertices lie on vertices of P iscalled good if all its sides are equal in length. Prove that there are at most 2n

3 good triangles.

Author: unknown author, Ukraine

http://www.artofproblemsolving.com/This file was downloaded from the AoPS Math Olympiad Resources Page Page 4

Page 202: យុវសិស្ ឈឹ ទីុយ IMO Shortlist 1959-2009 · IMO Shortlist 1959 1 Prove that the fraction 21n+4 14n+3 ... Let α be the acute angel subtending, from A, that

IMO Shortlist 2007

Geometry

1 In triangle ABC the bisector of angle BCA intersects the circumcircle again at R, the per-pendicular bisector of BC at P , and the perpendicular bisector of AC at Q. The midpointof BC is K and the midpoint of AC is L. Prove that the triangles RPK and RQL have thesame area.

Author: Marek Pechal, Czech Republic

2 Denote by M midpoint of side BC in an isosceles triangle 4ABC with AC = AB. Take apoint X on a smaller arc MA of circumcircle of triangle 4ABM . Denote by T point insideof angle BMA such that ∠TMX = 90 and TX = BX.

Prove that ∠MTB − ∠CTM does not depend on choice of X.

Author: unknown author, Canada

3 The diagonals of a trapezoid ABCD intersect at point P . Point Q lies between the parallellines BC and AD such that ∠AQD = ∠CQB, and line CD separates points P and Q. Provethat ∠BQP = ∠DAQ.

Author: unknown author, Ukraine

4 Consider five points A, B, C, D and E such that ABCD is a parallelogram and BCED is acyclic quadrilateral. Let ` be a line passing through A. Suppose that ` intersects the interiorof the segment DC at F and intersects line BC at G. Suppose also that EF = EG = EC.Prove that ` is the bisector of angle DAB.

Author: Charles Leytem, Luxembourg

5 Let ABC be a fixed triangle, and let A1, B1, C1 be the midpoints of sides BC, CA, AB,respectively. Let P be a variable point on the circumcircle. Let lines PA1, PB1, PC1 meetthe circumcircle again at A′, B′, C ′, respectively. Assume that the points A, B, C, A′, B′,C ′ are distinct, and lines AA′, BB′, CC ′ form a triangle. Prove that the area of this triangledoes not depend on P .

Author: Christopher Bradley, United Kingdom

6 Determine the smallest positive real number k with the following property. Let ABCD be aconvex quadrilateral, and let points A1, B1, C1, and D1 lie on sides AB, BC, CD, and DA,respectively. Consider the areas of triangles AA1D1, BB1A1, CC1B1 and DD1C1; let S bethe sum of the two smallest ones, and let S1 be the area of quadrilateral A1B1C1D1. Thenwe always have kS1 ≥ S.

Author: unknown author, USA

http://www.artofproblemsolving.com/This file was downloaded from the AoPS Math Olympiad Resources Page Page 5

Page 203: យុវសិស្ ឈឹ ទីុយ IMO Shortlist 1959-2009 · IMO Shortlist 1959 1 Prove that the fraction 21n+4 14n+3 ... Let α be the acute angel subtending, from A, that

IMO Shortlist 2007

7 Given an acute triangle ABC with ∠B > ∠C. Point I is the incenter, and R the circumradius.Point D is the foot of the altitude from vertex A. Point K lies on line AD such that AK = 2R,and D separates A and K. Lines DI and KI meet sides AC and BC at E,F respectively.Let IE = IF .

Prove that ∠B ≤ 3∠C.

Author: Davoud Vakili, Iran

8 Point P lies on side AB of a convex quadrilateral ABCD. Let ω be the incircle of triangleCPD, and let I be its incenter. Suppose that ω is tangent to the incircles of triangles APDand BPC at points K and L, respectively. Let lines AC and BD meet at E, and let linesAK and BL meet at F . Prove that points E, I, and F are collinear.

Author: Waldemar Pompe, Poland

http://www.artofproblemsolving.com/This file was downloaded from the AoPS Math Olympiad Resources Page Page 6

Page 204: យុវសិស្ ឈឹ ទីុយ IMO Shortlist 1959-2009 · IMO Shortlist 1959 1 Prove that the fraction 21n+4 14n+3 ... Let α be the acute angel subtending, from A, that

IMO Shortlist 2007

Number Theory

1 Find all pairs of natural number (a, b) satisfying 7a − 3b divides a4 + b2

Author: Stephan Wagner, Austria

2 Let b, n > 1 be integers. Suppose that for each k > 1 there exists an integer ak such thatb− an

k is divisible by k. Prove that b = An for some integer A.

Author: unknown author, Canada

3 Let X be a set of 10,000 integers, none of them is divisible by 47. Prove that there existsa 2007-element subset Y of X such that a − b + c − d + e is not divisible by 47 for anya, b, c, d, e ∈ Y.

Author: Gerhard Wginger, Netherlands

4 For every integer k ≥ 2, prove that 23k divides the number(2k+1

2k

)−(

2k

2k−1

)but 23k+1 does not.

Author: unknown author, Poland

5 Find all surjective functions f : N 7→ N such that for every m,n ∈ N and every prime p, thenumber f(m + n) is divisible by p if and only if f(m) + f(n) is divisible by p.

Author: Mohsen Jamaali and Nima Ahmadi Pour Anari, Iran

6 Let k be a positive integer. Prove that the number (4 · k2 − 1)2 has a positive divisor of theform 8kn− 1 if and only if k is even.

[url=http://www.mathlinks.ro/viewtopic.php?p=894656894656]Actual IMO 2007 Problem,posed as question 5 in the contest, which was used as a lemma in the official solutions forproblem N6 as shown above.[/url]

Author: Kevin Buzzard and Edward Crane, United Kingdom

7 For a prime p and a given integer n let νp(n) denote the exponent of p in the prime factorisationof n!. Given d ∈ N and p1, p2, . . . , pk a set of k primes, show that there are infinitely manypositive integers n such that d|νpi(n) for all 1 ≤ i ≤ k.

Author: Tejaswi Navilarekkallu, India

http://www.artofproblemsolving.com/This file was downloaded from the AoPS Math Olympiad Resources Page Page 7

Page 205: យុវសិស្ ឈឹ ទីុយ IMO Shortlist 1959-2009 · IMO Shortlist 1959 1 Prove that the fraction 21n+4 14n+3 ... Let α be the acute angel subtending, from A, that

IMO Shortlist 2008

Berlin, Germany

Algebra

1 Find all functions f : (0,∞) 7→ (0,∞) (so f is a function from the positive real numbers)such that

(f(w))2 + (f(x))2

f(y2) + f(z2)=

w2 + x2

y2 + z2

for all positive real numbes w, x, y, z, satisfying wx = yz.

Author: Hojoo Lee, South Korea

2 (i) If x, y and z are three real numbers, all different from 1, such that xyz = 1, then provethat x2

(x−1)2+ y2

(y−1)2+ z2

(z−1)2≥ 1. (With the

∑sign for cyclic summation, this inequality

could be rewritten as∑ x2

(x−1)2≥ 1.)

(ii) Prove that equality is achieved for infinitely many triples of rational numbers x, y and z.

Author: Walther Janous, Austria

3 Let S ⊆ R be a set of real numbers. We say that a pair (f, g) of functions from S into S is aSpanish Couple on S, if they satisfy the following conditions: (i) Both functions are strictlyincreasing, i.e. f(x) < f(y) and g(x) < g(y) for all x, y ∈ S with x < y;

(ii) The inequality f (g (g (x))) < g (f (x)) holds for all x ∈ S.

Decide whether there exists a Spanish Couple on the set S = N of positive integers; [/*:m]on the set S = a− 1

b : a, b ∈ N[/*:m]

Proposed by Hans Zantema, Netherlands

4 For an integer m, denote by t(m) the unique number in 1, 2, 3 such that m + t(m) is amultiple of 3. A function f : Z → Z satisfies f(−1) = 0, f(0) = 1, f(1) = − 1 andf (2n + m) = f (2n − t(m)) − f(m) for all integers m, n ≥ 0 with 2n > m. Prove thatf(3p) ≥ 0 holds for all integers p ≥ 0.

Proposed by Gerhard Woeginger, Austria

5 Let a, b, c, d be positive real numbers such that abcd = 1 and a + b + c + d >a

b+

b

c+

c

d+

d

a.

Prove thata + b + c + d <

b

a+

c

b+

d

c+

a

d

Proposed by Pavel Novotn, Slovakia

http://www.artofproblemsolving.com/This file was downloaded from the AoPS Math Olympiad Resources Page Page 1

Page 206: យុវសិស្ ឈឹ ទីុយ IMO Shortlist 1959-2009 · IMO Shortlist 1959 1 Prove that the fraction 21n+4 14n+3 ... Let α be the acute angel subtending, from A, that

IMO Shortlist 2008

Berlin, Germany

6 Let f : R → N be a function which satisfies f

(x +

1f(y)

)= f

(y +

1f(x)

)for all x, y ∈ R.

Prove that there is a positive integer which is not a value of f .

Proposed by ymantas Darbnas (Zymantas Darbenas), Lithania

7 Prove that for any four positive real numbers a, b, c, d the inequality

(a− b)(a− c)a + b + c

+(b− c)(b− d)

b + c + d+

(c− d)(c− a)c + d + a

+(d− a)(d− b)

d + a + b≥ 0

holds. Determine all cases of equality.

Author: Darij Grinberg (Problem Proposal), Christian Reiher (Solution), Germany

http://www.artofproblemsolving.com/This file was downloaded from the AoPS Math Olympiad Resources Page Page 2

Page 207: យុវសិស្ ឈឹ ទីុយ IMO Shortlist 1959-2009 · IMO Shortlist 1959 1 Prove that the fraction 21n+4 14n+3 ... Let α be the acute angel subtending, from A, that

IMO Shortlist 2008

Berlin, Germany

Combinatorics

1 In the plane we consider rectangles whose sides are parallel to the coordinate axes and havepositive length. Such a rectangle will be called a box. Two boxes intersect if they have acommon point in their interior or on their boundary. Find the largest n for which there existn boxes B1, . . ., Bn such that Bi and Bj intersect if and only if i 6≡ j ± 1 (mod n).

Proposed by Gerhard Woeginger, Netherlands

2 Let n ∈ N and An set of all permutations (a1, . . . , an) of the set 1, 2, . . . , n for which

k|2(a1 + · · ·+ ak), for all 1 ≤ k ≤ n.

Find the number of elements of the set An.

3 In the coordinate plane consider the set S of all points with integer coordinates. For a positiveinteger k, two distinct points a, B ∈ S will be called k-friends if there is a point C ∈ S suchthat the area of the triangle ABC is equal to k. A set T ⊂ S will be called k-clique if everytwo points in T are k-friends. Find the least positive integer k for which there exits a k-cliquewith more than 200 elements.

Proposed by Jorge Tipe, Peru

4 Let n and k be positive integers with k ≥ n and k−n an even number. Let 2n lamps labelled1, 2, ..., 2n be given, each of which can be either on or off. Initially all the lamps are off. Weconsider sequences of steps: at each step one of the lamps is switched (from on to off or fromoff to on).

Let N be the number of such sequences consisting of k steps and resulting in the state wherelamps 1 through n are all on, and lamps n + 1 through 2n are all off.

Let M be number of such sequences consisting of k steps, resulting in the state where lamps1 through n are all on, and lamps n + 1 through 2n are all off, but where none of the lampsn + 1 through 2n is ever switched on.

Determine NM .

Author: Bruno Le Floch and Ilia Smilga, France

5 Let S = x1, x2, . . . , xk+l be a (k + l)-element set of real numbers contained in the interval[0, 1]; k and l are positive integers. A k-element subset A ⊂ S is called nice if∣∣∣∣∣∣1k

∑xi∈A

xi −1l

∑xj∈S\A

xj

∣∣∣∣∣∣ ≤ k + l

2kl

http://www.artofproblemsolving.com/This file was downloaded from the AoPS Math Olympiad Resources Page Page 3

Page 208: យុវសិស្ ឈឹ ទីុយ IMO Shortlist 1959-2009 · IMO Shortlist 1959 1 Prove that the fraction 21n+4 14n+3 ... Let α be the acute angel subtending, from A, that

IMO Shortlist 2008

Berlin, Germany

Prove that the number of nice subsets is at least2

k + l

(k + l

k

).

Proposed by Andrey Badzyan, Russia

6 For n ≥ 2, let S1, S2, . . ., S2n be 2n subsets of A = 1, 2, 3, . . . , 2n+1 that satisfy thefollowing property: There do not exist indices a and b with a < b and elements x, y, z ∈ Awith x < y < z and y, z ∈ Sa, and x, z ∈ Sb. Prove that at least one of the sets S1, S2, . . .,S2n contains no more than 4n elements.

Proposed by Gerhard Woeginger, Netherlands

http://www.artofproblemsolving.com/This file was downloaded from the AoPS Math Olympiad Resources Page Page 4

Page 209: យុវសិស្ ឈឹ ទីុយ IMO Shortlist 1959-2009 · IMO Shortlist 1959 1 Prove that the fraction 21n+4 14n+3 ... Let α be the acute angel subtending, from A, that

IMO Shortlist 2008

Berlin, Germany

Geometry

1 Let H be the orthocenter of an acute-angled triangle ABC. The circle ΓA centered at themidpoint of BC and passing through H intersects the sideline BC at points A1 and A2.Similarly, define the points B1, B2, C1 and C2.

Prove that six points A1 , A2, B1, B2, C1 and C2 are concyclic.

Author: Andrey Gavrilyuk, Russia

2 Given trapezoid ABCD with parallel sides AB and CD, assume that there exist points E online BC outside segment BC, and F inside segment AD such that ∠DAE = ∠CBF . Denoteby I the point of intersection of CD and EF , and by J the point of intersection of AB andEF . Let K be the midpoint of segment EF , assume it does not lie on line AB. Prove thatI belongs to the circumcircle of ABK if and only if K belongs to the circumcircle of CDJ .

Proposed by Charles Leytem, Luxembourg

3 Let ABCD be a convex quadrilateral and let P and Q be points in ABCD such that PQDAand QPBC are cyclic quadrilaterals. Suppose that there exists a point E on the line segmentPQ such that ∠PAE = ∠QDE and ∠PBE = ∠QCE. Show that the quadrilateral ABCDis cyclic.

Proposed by John Cuya, Peru

4 In an acute triangle ABC segments BE and CF are altitudes. Two circles passing throughthe point A anf F and tangent to the line BC at the points P and Q so that B lies betweenC and Q. Prove that lines PE and QF intersect on the circumcircle of triangle AEF .

http://www.artofproblemsolving.com/This file was downloaded from the AoPS Math Olympiad Resources Page Page 5

Page 210: យុវសិស្ ឈឹ ទីុយ IMO Shortlist 1959-2009 · IMO Shortlist 1959 1 Prove that the fraction 21n+4 14n+3 ... Let α be the acute angel subtending, from A, that

IMO Shortlist 2008

Berlin, Germany

A

B

C

E

F

P

Q

H

D

[Diagram by azjps]

Proposed by Davood Vakili, Iran

5 Let k and n be integers with 0 ≤ k ≤ n − 2. Consider a set L of n lines in the plane suchthat no two of them are parallel and no three have a common point. Denote by I the set ofintersections of lines in L. Let O be a point in the plane not lying on any line of L. A pointX ∈ I is colored red if the open line segment OX intersects at most k lines in L. Prove that

I contains at least12(k + 1)(k + 2) red points.

Proposed by Gerhard Woeginger, Netherlands

6 There is given a convex quadrilateral ABCD. Prove that there exists a point P inside thequadrilateral such that

∠PAB + ∠PDC = ∠PBC + ∠PAD = ∠PCD + ∠PBA = ∠PDA + ∠PCB = 90

if and only if the diagonals AC and BD are perpendicular.

Proposed by Dukan Dukic, Serbia

7 Let ABCD be a convex quadrilateral with BA different from BC. Denote the incircles oftriangles ABC and ADC by k1 and k2 respectively. Suppose that there exists a circle ktangent to ray BA beyond A and to the ray BC beyond C, which is also tangent to the linesAD and CD.

Prove that the common external tangents to k1 and k2 intersects on k.

Author: Vladimir Shmarov, Russia

http://www.artofproblemsolving.com/This file was downloaded from the AoPS Math Olympiad Resources Page Page 6

Page 211: យុវសិស្ ឈឹ ទីុយ IMO Shortlist 1959-2009 · IMO Shortlist 1959 1 Prove that the fraction 21n+4 14n+3 ... Let α be the acute angel subtending, from A, that

IMO Shortlist 2008

Berlin, Germany

Number Theory

1 Let n be a positive integer and let p be a prime number. Prove that if a, b, c are integers(not necessarily positive) satisfying the equations

an + pb = bn + pc = cn + pa

then a = b = c.

Proposed by Angelo Di Pasquale, Australia

2 Let a1, a2, . . ., an be distinct positive integers, n ≥ 3. Prove that there exist distinct indicesi and j such that ai + aj does not divide any of the numbers 3a1, 3a2, . . ., 3an.

Proposed by Mohsen Jamaali, Iran

3 Let a0, a1, a2, . . . be a sequence of positive integers such that the greatest common divisor ofany two consecutive terms is greater than the preceding term; in symbols, gcd(ai, ai+1) > ai−1.Prove that an ≥ 2n for all n ≥ 0.

Proposed by Morteza Saghafian, Iran

4 Let n be a positive integer. Show that the numbers(2n − 1

0

),

(2n − 1

1

),

(2n − 1

2

), . . . ,

(2n − 1

2n−1 − 1

)are congruent modulo 2n to 1, 3, 5, . . ., 2n − 1 in some order.

Proposed by Duskan Dukic, Serbia

5 For every n ∈ N let d(n) denote the number of (positive) divisors of n. Find all functionsf : N → N with the following properties: d (f(x)) = x for all x ∈ N.[/*:m] f(xy) divides(x− 1)yxy−1f(x) for all x, y ∈ N.[/*:m]

Proposed by Bruno Le Floch, France

6 Prove that there are infinitely many positive integers n such that n2 + 1 has a prime divisorgreater than 2n +

√2n.

Author: Kestutis Cesnavicius, Lithuania

http://www.artofproblemsolving.com/This file was downloaded from the AoPS Math Olympiad Resources Page Page 7

Page 212: យុវសិស្ ឈឹ ទីុយ IMO Shortlist 1959-2009 · IMO Shortlist 1959 1 Prove that the fraction 21n+4 14n+3 ... Let α be the acute angel subtending, from A, that

IMO Shortlist 2009

Algebra

1 Find the largest possible integer k, such that the following statement is true: Let 2009arbitrary non-degenerated triangles be given. In every triangle the three sides are coloured,such that one is blue, one is red and one is white. Now, for every colour separately, let ussort the lengths of the sides. We obtain

b1 ≤ b2 ≤ . . . ≤ b2009 the lengths of the blue sidesr1 ≤ r2 ≤ . . . ≤ r2009 the lengths of the red sides

and w1 ≤ w2 ≤ . . . ≤ w2009 the lengths of the white sides

Then there exist k indices j such that we can form a non-degenerated triangle with sidelengths bj , rj , wj .

Proposed by Michal Rolinek, Czech Republic

2 Let a, b, c be positive real numbers such that1a

+1b

+1c

= a + b + c. Prove that:

1(2a + b + c)2

+1

(a + 2b + c)2+

1(a + b + 2c)2

≤ 316

Proposed by Juhan Aru, Estonia

3 Determine all functions f from the set of positive integers to the set of positive integers suchthat, for all positive integers a and b, there exists a non-degenerate triangle with sides oflengths

a, f(b) and f(b + f(a)− 1).

(A triangle is non-degenerate if its vertices are not collinear.)

Proposed by Bruno Le Floch, France

4 Let a, b, c be positive real numbers such that ab + bc + ca ≤ 3abc. Prove that√a2 + b2

a + b+

√b2 + c2

b + c+

√c2 + a2

c + a+ 3 ≤

√2

(√a + b +

√b + c +

√c + a

)Proposed by Dzianis Pirshtuk, Belarus

5 Let f be any function that maps the set of real numbers into the set of real numbers. Provethat there exist real numbers x and y such that

f (x− f(y)) > yf(x) + x

Proposed by Igor Voronovich, Belarus

http://www.artofproblemsolving.com/This file was downloaded from the AoPS Math Olympiad Resources Page Page 1

Page 213: យុវសិស្ ឈឹ ទីុយ IMO Shortlist 1959-2009 · IMO Shortlist 1959 1 Prove that the fraction 21n+4 14n+3 ... Let α be the acute angel subtending, from A, that

IMO Shortlist 2009

6 Suppose that s1, s2, s3, . . . is a strictly increasing sequence of positive integers such thatthe sub-sequences ss1 , ss2 , ss3 , . . . and ss1+1, ss2+1, ss3+1, . . . are both arithmetic progressions.Prove that the sequence s1, s2, s3, . . . is itself an arithmetic progression.

Proposed by Gabriel Carroll, USA

7 Find all functions f from the set of real numbers into the set of real numbers which satisfyfor all x, y the identity

f (xf(x + y)) = f (yf(x)) + x2

Proposed by Japan

http://www.artofproblemsolving.com/This file was downloaded from the AoPS Math Olympiad Resources Page Page 2

Page 214: យុវសិស្ ឈឹ ទីុយ IMO Shortlist 1959-2009 · IMO Shortlist 1959 1 Prove that the fraction 21n+4 14n+3 ... Let α be the acute angel subtending, from A, that

IMO Shortlist 2009

Combinatorics

1 Consider 2009 cards, each having one gold side and one black side, lying on parallel on a longtable. Initially all cards show their gold sides. Two player, standing by the same long side ofthe table, play a game with alternating moves. Each move consists of choosing a block of 50consecutive cards, the leftmost of which is showing gold, and turning them all over, so thosewhich showed gold now show black and vice versa. The last player who can make a legalmove wins. (a) Does the game necessarily end? (b) Does there exist a winning strategy forthe starting player?

Proposed by Michael Albert, Richard Guy, New Zealand

2 For any integer n ≥ 2, let N(n) be the maxima number of triples (ai, bi, ci), i = 1, . . . , N(n),consisting of nonnegative integers ai, bi and ci such that the following two conditions aresatisfied: ai + bi + ci = n for all i = 1, . . . , N(n),[/*:m] If i 6= j then ai 6= aj , bi 6= bj andci 6= cj [/*:m] Determine N(n) for all n ≥ 2.

Proposed by Dan Schwarz, Romania

3 Let n be a positive integer. Given a sequence ε1, . . . , εn−1 with εi = 0 or εi = 1 for eachi = 1, . . . , n − 1, the sequences a0, . . . , an and b0, . . . , bn are constructed by the followingrules: a0 = b0 = 1, a1 = b1 = 7,

ai+1 =

2ai−1 + 3ai, if εi = 0,3ai−1 + ai, if εi = 1,

for each i = 1, . . . , n− 1,

bi+1 =

2bi−1 + 3bi, if εn−i = 0,3bi−1 + bi, if εn−i = 1,

for each i = 1, . . . , n− 1. Prove that an = bn.

Proposed by Ilya Bogdanov, Russia

4 For an integer m ≥ 1, we consider partitions of a 2m×2m chessboard into rectangles consistingof cells of chessboard, in which each of the 2m cells along one diagonal forms a separaterectangle of side length 1. Determine the smallest possible sum of rectangle perimeters insuch a partition.

Proposed by Gerhard Woeginger, Netherlands

http://www.artofproblemsolving.com/This file was downloaded from the AoPS Math Olympiad Resources Page Page 3

Page 215: យុវសិស្ ឈឹ ទីុយ IMO Shortlist 1959-2009 · IMO Shortlist 1959 1 Prove that the fraction 21n+4 14n+3 ... Let α be the acute angel subtending, from A, that

IMO Shortlist 2009

5 Five identical empty buckets of 2-liter capacity stand at the vertices of a regular pentagon.Cinderella and her wicked Stepmother go through a sequence of rounds: At the beginning ofevery round, the Stepmother takes one liter of water from the nearby river and distributesit arbitrarily over the five buckets. Then Cinderella chooses a pair of neighbouring buckets,empties them to the river and puts them back. Then the next round begins. The Stepmothergoal’s is to make one of these buckets overflow. Cinderella’s goal is to prevent this. Can thewicked Stepmother enforce a bucket overflow?

Proposed by Gerhard Woeginger, Netherlands

6 On a 999 × 999 board a limp rook can move in the following way: From any square it canmove to any of its adjacent squares, i.e. a square having a common side with it, and everymove must be a turn, i.e. the directions of any two consecutive moves must be perpendicular.A non-intersecting route of the limp rook consists of a sequence of pairwise different squaresthat the limp rook can visit in that order by an admissible sequence of moves. Such a non-intersecting route is called cyclic, if the limp rook can, after reaching the last square of theroute, move directly to the first square of the route and start over. How many squares doesthe longest possible cyclic, non-intersecting route of a limp rook visit?

Proposed by Nikolay Beluhov, Bulgaria

7 Let a1, a2, . . . , an be distinct positive integers and let M be a set of n−1 positive integers notcontaining s = a1 + a2 + . . . + an. A grasshopper is to jump along the real axis, starting atthe point 0 and making n jumps to the right with lengths a1, a2, . . . , an in some order. Provethat the order can be chosen in such a way that the grasshopper never lands on any point inM.

Proposed by Dmitry Khramtsov, Russia

8 For any integer n ≥ 2, we compute the integer h(n) by applying the following procedureto its decimal representation. Let r be the rightmost digit of n. If r = 0, then the decimalrepresentation of h(n) results from the decimal representation of n by removing this rightmostdigit 0.[/*:m] If 1 ≤ r ≤ 9 we split the decimal representation of n into a maximal right partR that solely consists of digits not less than r and into a left part L that either is empty orends with a digit strictly smaller than r. Then the decimal representation of h(n) consists ofthe decimal representation of L, followed by two copies of the decimal representation of R−1.For instance, for the number 17, 151, 345, 543, we will have L = 17, 151, R = 345, 543 andh(n) = 17, 151, 345, 542, 345, 542.[/*:m] Prove that, starting with an arbitrary integer n ≥ 2,iterated application of h produces the integer 1 after finitely many steps.

Proposed by Gerhard Woeginger, Austria

http://www.artofproblemsolving.com/This file was downloaded from the AoPS Math Olympiad Resources Page Page 4

Page 216: យុវសិស្ ឈឹ ទីុយ IMO Shortlist 1959-2009 · IMO Shortlist 1959 1 Prove that the fraction 21n+4 14n+3 ... Let α be the acute angel subtending, from A, that

IMO Shortlist 2009

Geometry

1 Let ABC be a triangle with AB = AC . The angle bisectors of ∠CAB and ∠ABC meetthe sides BC and CA at D and E , respectively. Let K be the incentre of triangle ADC.Suppose that ∠BEK = 45 . Find all possible values of ∠CAB .

Jan Vonk, Belgium, Peter Vandendriessche, Belgium and Hojoo Lee, Korea

2 Let ABC be a triangle with circumcentre O. The points P and Q are interior points of thesides CA and AB respectively. Let K, L and M be the midpoints of the segments BP, CQand PQ. respectively, and let Γ be the circle passing through K, L and M . Suppose that theline PQ is tangent to the circle Γ. Prove that OP = OQ.

Proposed by Sergei Berlov, Russia

3 Let ABC be a triangle. The incircle of ABC touches the sides AB and AC at the points Zand Y , respectively. Let G be the point where the lines BY and CZ meet, and let R and Sbe points such that the two quadrilaterals BCY R and BCSZ are parallelogram. Prove thatGR = GS.

Proposed by Hossein Karke Abadi, Iran

4 Given a cyclic quadrilateral ABCD, let the diagonals AC and BD meet at E and the linesAD and BC meet at F . The midpoints of AB and CD are G and H, respectively. Showthat EF is tangent at E to the circle through the points E, G and H.

Proposed by David Monk, United Kingdom

5 Let P be a polygon that is convex and symmetric to some point O. Prove that for someparallelogram R satisfying P ⊂ R we have

|R||P |

≤√

2

where |R| and |P | denote the area of the sets R and P , respectively.

Proposed by Witold Szczechla, Poland

6 Let the sides AD and BC of the quadrilateral ABCD (such that AB is not parallel toCD) intersect at point P . Points O1 and O2 are circumcenters and points H1 and H2 areorthocenters of triangles ABP and CDP , respectively. Denote the midpoints of segmentsO1H1 and O2H2 by E1 and E2, respectively. Prove that the perpendicular from E1 on CD,the perpendicular from E2 on AB and the lines H1H2 are concurrent.

Proposed by Ukraine

http://www.artofproblemsolving.com/This file was downloaded from the AoPS Math Olympiad Resources Page Page 5

Page 217: យុវសិស្ ឈឹ ទីុយ IMO Shortlist 1959-2009 · IMO Shortlist 1959 1 Prove that the fraction 21n+4 14n+3 ... Let α be the acute angel subtending, from A, that

IMO Shortlist 2009

7 Let ABC be a triangle with incenter I and let X, Y and Z be the incenters of the trianglesBIC, CIA and AIB, respectively. Let the triangle XY Z be equilateral. Prove that ABC isequilateral too.

Proposed by Mirsaleh Bahavarnia, Iran

8 Let ABCD be a circumscribed quadrilateral. Let g be a line through A which meets thesegment BC in M and the line CD in N . Denote by I1, I2 and I3 the incenters of 4ABM ,4MNC and 4NDA, respectively. Prove that the orthocenter of 4I1I2I3 lies on g.

Proposed by Nikolay Beluhov, Bulgaria

http://www.artofproblemsolving.com/This file was downloaded from the AoPS Math Olympiad Resources Page Page 6

Page 218: យុវសិស្ ឈឹ ទីុយ IMO Shortlist 1959-2009 · IMO Shortlist 1959 1 Prove that the fraction 21n+4 14n+3 ... Let α be the acute angel subtending, from A, that

IMO Shortlist 2009

Number Theory

1 Let n be a positive integer and let a1, a2, a3, . . . , ak (k ≥ 2) be distinct integers in the set1, 2, . . . , n such that n divides ai(ai+1−1) for i = 1, 2, . . . , k−1. Prove that n does not divideak(a1 − 1).

Proposed by Ross Atkins, Australia

2 A positive integer N is called balanced, if N = 1 or if N can be written as a product of aneven number of not necessarily distinct primes. Given positive integers a and b, consider thepolynomial P defined by P (x) = (x + a)(x + b). (a) Prove that there exist distinct positiveintegers a and b such that all the number P (1), P (2),. . ., P (50) are balanced. (b) Prove thatif P (n) is balanced for all positive integers n, then a = b.

Proposed by Jorge Tipe, Peru

3 Let f be a non-constant function from the set of positive integers into the set of positiveinteger, such that a − b divides f(a) − f(b) for all distinct positive integers a, b. Prove thatthere exist infinitely many primes p such that p divides f(c) for some positive integer c.

Proposed by Juhan Aru, Estonia

4 Find all positive integers n such that there exists a sequence of positive integers a1, a2,. . .,an satisfying:

ak+1 =a2

k + 1ak−1 + 1

− 1

for every k with 2 ≤ k ≤ n− 1.

Proposed by North Korea

5 Let P (x) be a non-constant polynomial with integer coefficients. Prove that there is nofunction T from the set of integers into the set of integers such that the number of integers xwith Tn(x) = x is equal to P (n) for every n ≥ 1, where Tn denotes the n-fold application ofT .

Proposed by Jozsef Pelikan, Hungary

6 Let k be a positive integer. Show that if there exists a consequence a0, a1,. . . of integerssatisfying the condition

an =an−1 + nk

nfor all n ≥ 1,

then k − 2 is divisible by 3.

Proposed by Turkey

http://www.artofproblemsolving.com/This file was downloaded from the AoPS Math Olympiad Resources Page Page 7

Page 219: យុវសិស្ ឈឹ ទីុយ IMO Shortlist 1959-2009 · IMO Shortlist 1959 1 Prove that the fraction 21n+4 14n+3 ... Let α be the acute angel subtending, from A, that

IMO Shortlist 2009

7 Let a and b be distinct integers greater than 1. Prove that there exists a positive integer nsuch that (an − 1)(bn − 1) is not a perfect square.

Proposed by Mongolia

http://www.artofproblemsolving.com/This file was downloaded from the AoPS Math Olympiad Resources Page Page 8